Naked Science Forum

Non Life Sciences => Physics, Astronomy & Cosmology => Topic started by: Sarah Raphaella Rodgers on 26/10/2009 09:30:10

Title: Why don't an atom's electrons fall into the nucleus and stick to the protons?
Post by: Sarah Raphaella Rodgers on 26/10/2009 09:30:10
Sarah Raphaella Rodgers asked the Naked Scientists:
   
I'm a 16 year old Chemistry student. My Chemistry class has been focusing on the periodic table recently. I know that protons are positively charged, neutrons are neutral, electrons are  negatively charged and that atoms are mostly empty space. I also know for magnets opposites attract.

So why don't electrons stick to protons instead of flying around the nucleus? Magnets do it, so why can't atoms?

What do you think?
Title: Re: Why don't an atom's electrons fall into the nucleus and stick to the protons?
Post by: Mr. Scientist on 26/10/2009 10:58:54
It has to do with the uncertainty principle. Because the electron cannot have a defined position in the nuclei of atoms means that it must occupy every other space within the atom in a wave of possibilities. If the electron was positioned with great certainty within the nuclei of atoms, their momenta becomes infinitely uncertaint. But instead, they seem to have energy-orbits inside of atoms which determine the chemical struture of the universe. Another interesting thing to note is that electrons could not be in the center of atoms, because if they where, matter would drastically sink in size.

We already know of nature objects which undergo this process, and they go by the name of neutron stars. In classical mechanics, electrons couple so strongly with protons that they should collapse all the time; and would in classical physics mean that every nucleus of every atom would gobble up the electrons in about 100 microseconds.
Title: Re: Why don't an atom's electrons fall into the nucleus and stick to the protons?
Post by: Vern on 27/10/2009 11:29:02
When I was 16 people like Einstein and Schrodinger were reassuring us that we need not worry; the Copenhagen interpretation and Quantum Theory were too weird to make any real difference. Now we are a half century into Physics becoming Quantum theory to the exclusion of reality even. Causality was abandoned because Quantum theory can't survive if we insist upon it.

There is a cause for quantum phenomena just as there is a cause for uncertainty.  Philosopher David Hume trashed causality with his view that no matter the times we observe an event and its precursor we can never be certain that such an event will follow a future precursor of the same nature. Philosopher Emanual Kant insisted that there is a cause for every event, however; it is just that we may never know that cause with certainty.

Edit: The cause of all quantum phenomena is that the electric and magnetic amplitude that space can support is a finite value; all photons peak at this value. Max Planck observed this. But because we did not demand causality, we imagined the Quantum nature of the universe without even considering its cause.

Uncertainty has been boiled down to the statement that (http://photontheory.com/uncertain.html) it is impossible to know both the position and the momentum of anything absolutely. The more you know about the position of something, the less you can know about its momentum.

Books have been written about the implications of this. The link describes the cause of uncertainty. The quote below is the meat of it.

Edit: I should point out that the causes mentioned are my speculation; you won't find them in physics books. [:)]

Quote from: the link
The electromagnetic fields that comprise a photon are in a state of constant change. This change drives the central point of a photon forward through space. We measure the photon's path to be that of the central point, but the fields exist spatially around the photon at an amplitude that is greatest close to the point and diminishes as the square of distance away from the point.
When this photon nears its target, churning electrons belonging to atoms in the target begin to sense the photon's approach. Some electromagnetic fields in the electrons will be in good phase relation with the approaching photon. Among this huge jumble of moving electrons, some will be more inclined to absorb the photon's fields than others. Those most inclined will probably not be dead centre in the photon's path.
Title: Re: Why don't an atom's electrons fall into the nucleus and stick to the protons?
Post by: Vern on 27/10/2009 13:47:02
Quote from: Sarah Raphaella Rodgers
So why don't electrons stick to protons instead of flying around the nucleus? Magnets do it, so why can't atoms?
The present state of physical science does not allow "why" questions. Any answer will have to be speculative. I have an answer to the question that works well for me.

There has never been found any substance of an electron that is smaller than its electromagnetic radius. This radius is much larger than a proton. So if observations are correct, and electrons only exist at their electromagnetic radius, they would consist of a hollow shell about 12 times larger than a proton. The electron would engulf the proton and form a dynamic dance with the proton's charges.

This is speculative, but it explains the observations.
Title: Re: Why don't an atom's electrons fall into the nucleus and stick to the protons?
Post by: Homely Physicist on 01/11/2009 02:32:13
It has to do with the uncertainty principle.

I'm afraid not. It's actually a result of two physical phenomena.


1) Pauli exclusion principle

This states that two fermions must be distinguishable i.e. you can always tell them apart. In practice, this means they must have at least one different quantum number. This restricts electrons into their shell structure. For example, consider hydrogen. The first shell (s- shell) has quantum numbers (1,1,1) and (1,1,-1). This is why two electrons, at most, can occupy the s- shell. These number combinations are easily derivable by solving the Schrodinger wave equation for hydrogen.


2) Entropy

Processes in physics tend to increase the entropy of the universe. Energy likes to go from ordered states to disordered (like how a ball wants to roll down a slope). A proton and an electron is more energetically favourable than a neutron. The decay of neutrons this way is known as beta decay. In order to 'squash' together a proton and an electron into a neutron you need to supply a large amount of energy, as well as overcome the electron degeneracy force (as you're probably going to try it with a large collection of atoms rather than waiting millions of years for a single electron to pair up). This occurs inside neutron stars.
Title: Re: Why don't an atom's electrons fall into the nucleus and stick to the protons?
Post by: Vern on 01/11/2009 03:40:45
Principles do not cause things; principles merely describe the happenings. We tend to think of principles and theories as causes; they can not be causes; their use is in describing the happenings. [:)] I'm just trying to keep folks honest.
Title: Re: Why don't an atom's electrons fall into the nucleus and stick to the protons?
Post by: Mr. Scientist on 01/11/2009 15:33:42
It has to do with the uncertainty principle.

I'm afraid not. It's actually a result of two physical phenomena.


1) Pauli exclusion principle

This states that two fermions must be distinguishable i.e. you can always tell them apart. In practice, this means they must have at least one different quantum number. This restricts electrons into their shell structure. For example, consider hydrogen. The first shell (s- shell) has quantum numbers (1,1,1) and (1,1,-1). This is why two electrons, at most, can occupy the s- shell. These number combinations are easily derivable by solving the Schrodinger wave equation for hydrogen.


2) Entropy

Processes in physics tend to increase the entropy of the universe. Energy likes to go from ordered states to disordered (like how a ball wants to roll down a slope). A proton and an electron is more energetically favourable than a neutron. The decay of neutrons this way is known as beta decay. In order to 'squash' together a proton and an electron into a neutron you need to supply a large amount of energy, as well as overcome the electron degeneracy force (as you're probably going to try it with a large collection of atoms rather than waiting millions of years for a single electron to pair up). This occurs inside neutron stars.

I am sure Hawking himself said the Uncertainty Principle had something to do with it.

Either way you're wrong, the pauli explusion principle has nothing to do with electrons falling into the nuclei of atoms. It's a process which eliminates one fermion energy level to another. This happens everywhere, not only inside an atom. And entropy also has nothing to do with it.
Title: Re: Why don't an atom's electrons fall into the nucleus and stick to the protons?
Post by: Mr. Scientist on 01/11/2009 15:36:37
Just in case you would like an example of the exclusionary principle ordinary in nature, it even happens when two electrons come close to each other in space. It's closely related to the wave function, which is actually one main reason why the electron does not fall into the nuclei of atoms; specifically because they are not located to any particular region of space, which would induce a collapse of their superpositioned states. They are ''arranged'' within their superpositioning because of energy levels. But the exclusion principle is not the prime cause of either the wave function or the fundemental reason why particles do not fall into the nuclei of atoms.
Title: Re: Why don't an atom's electrons fall into the nucleus and stick to the protons?
Post by: Mr. Scientist on 01/11/2009 15:45:03
I knew i was right. I came across this convo on the net:

If these particles are attracted to one another, shouldn't electrons be pulled into the nucleus? I gather the reasoning is because of the strong force? If thats the case i need to understand this "strong force" better..

Mizzuno

This question is actually addressed in the Feynmann lectures, which are linked to in the physics napster thread in the General Physics forum. The answer is:

What keeps the electrons from simply falling in? [The uncertainty principle]: If they were in the nucleus, we would know their position precisely, which would require them to have a very large, but uncertain, momentum, i.e., a very large kinetic energy. This would cause them to break away from the nucleus. They make a compromise: they leave themselves a little room for this uncertainty and then jiggle with a certain amount of minimum motion in accordance with this rule.

It wasn't really the answer I was expecting. I was previously under the impression that the uncertainty relations were only an expression of our own limitation as subjective observers of a subatomic event, but apparently they are actually an expression of a fundamental principle governing the behavior of small particles. If you're curious, the relation used here is:

\Delta x \Delta \rho \geq \frac{h}{2\pi}

Where
x = the position of the particle,
\rho = the momentum of the particle, and
h = Planck's constant
[\i][\b]
Title: Re: Why don't an atom's electrons fall into the nucleus and stick to the protons?
Post by: Vern on 01/11/2009 16:49:00
If you like to think that Quantum theory represents reality you have to invent excuses. Quarks can not exist outside nuclei, for example. Electrons dance to the uncertainty tune, etc. To me it is much easier just to accept reality as it presents itself.
Title: Re: Why don't an atom's electrons fall into the nucleus and stick to the protons?
Post by: Mr. Scientist on 01/11/2009 16:55:29
But it seems that we hve experimental evidence for these conclusions. If anything, i think reality has shaped physics for the larger part, not so much intentionally the other the way.
Title: Re: Why don't an atom's electrons fall into the nucleus and stick to the protons?
Post by: Vern on 01/11/2009 18:13:39
Quote from: Mr. Scientist
But it seems that we hve experimental evidence for these conclusions.
But we really don't. I started looking for experimental evidence for wave function collapse years ago. I'm still looking. None found. We have a habit of reporting our conclusions as experimental results. Sometimes it is hard to find the actual results that led the experimenters to their reported conclusions.

In every case where I have searched out the actual experiment the evidence was not there. The POS thought experiment Einstein and company proposed is still valid.
Title: Re: Why don't an atom's electrons fall into the nucleus and stick to the protons?
Post by: Mr. Scientist on 01/11/2009 18:21:56
We can measure decoherence, which is the gradual collapse of the wave function in wave-states of matter. We may not be able to directly observe the transformation because in doing so we disturb the p-field ''probability-field''. But, we know the collapse must occur as an actual transition from having matter acts as waves and then suddenly not.
Title: Re: Why don't an atom's electrons fall into the nucleus and stick to the protons?
Post by: Vern on 01/11/2009 18:35:25
Quote from: Mr. Scientist
But, we know the collapse must occur as an actual transition from having matter acts as waves and then suddenly not.
But this is what we don't know; this is the idea in contention. Does the observed state happen at the time of observation as in wave function collapse, or does the observed state happen at the time of creation of the particles, as in the POS experiment?
Title: Re: Why don't an atom's electrons fall into the nucleus and stick to the protons?
Post by: Mr. Scientist on 01/11/2009 18:38:08
But there is very little else that can happen. Given the intantaneous change from wave to particle-nature means that there is little room other than to say there is a sudden collapse. All models have agreed with observation.
Title: Re: Why don't an atom's electrons fall into the nucleus and stick to the protons?
Post by: Vern on 01/11/2009 18:56:47
We don't know that there is an instantaneous transition from wave to particle. We know that there is an instantaneous transition of a previously unknown state to a known state at the time of observation. We have not yet figured out how to know the state of the previously unknown state. 

In the simple case of a photon striking a target, my speculative model has changing fields driving two points of maxima of the fields. Interaction always occurs very close to the points of maxima; the fields determine the trajectory.

Edit: Bolded text was edited for clarification.
Title: Re: Why don't an atom's electrons fall into the nucleus and stick to the protons?
Post by: Mr. Scientist on 01/11/2009 19:00:32
Though, we have what we need to know about this state, and that is it acts in every way like a particle when its not being observed.
Title: Re: Why don't an atom's electrons fall into the nucleus and stick to the protons?
Post by: Vern on 01/11/2009 19:06:28
I can't argue with the success of Quantum theory. It is the only theory I know that demands a change in reality when reality does not agree with it. [:)]
Title: Re: Why don't an atom's electrons fall into the nucleus and stick to the protons?
Post by: Mr. Scientist on 01/11/2009 19:14:39
I meant a wave by the way in the passage above - oops.
Title: Re: Why don't an atom's electrons fall into the nucleus and stick to the protons?
Post by: Vern on 02/11/2009 12:02:19
I don't mean to be contrary. [:)] I just need to explore every possibility that might offer experimental evidence that my vision of a photon is not reality. As far as I can determine the double slit experiment supports the vision. If I did not have the photon defined so that it must produce the observed results by cause and effect, I might fantasize some magical wave-particle duality.

The anatomy of a photon: A photon consists of two half cycles of electric and magnetic fields that drive points of maxima through space. The fields exist in a spatial area around the points. The changing amplitude of the fields drive the points and determine their path through space. Photon interaction happens at the points of maxima. So any observation will see the points. Edit: It is not my definition; it is Maxwell's definition.

What perplexes me is that folks don't seem to understand that. Is it that I just can't put the right words together?

Here's a schematic of the vision. It looks just like those that were in the text books when I studied electronics and nuclear instrumentation back in the 50's.

(https://www.thenakedscientists.com/forum/proxy.php?request=http%3A%2F%2Fphotontheory.com%2Fphoton-01.jpg&hash=fdb8df20e4b0531ffc4c59573374c8c0)
Title: Re: Why don't an atom's electrons fall into the nucleus and stick to the protons?
Post by: Mr. Scientist on 02/11/2009 12:13:23
I wouldn't be as bold as to suggest you cannot explain physics, if indeed it is the correct description of a photon. Physics is not easy to explain, whether it being a pet-theory or not.
Title: Re: Why don't an atom's electrons fall into the nucleus and stick to the protons?
Post by: Vern on 02/11/2009 16:51:24
I have to keep reminding myself what my goal is here in this forum. It is not to point out weaknesses in Quantum theory, and it is not to promote my pet concepts. It is simply to remind folks when common misconceptions are promoted. In this case it was the misconception that there is experimental evidence that quantum states occur at observation time. [:)]
Title: Re: Why don't an atom's electrons fall into the nucleus and stick to the protons?
Post by: Mr. Scientist on 03/11/2009 16:13:03
Fair do's.
Title: Re: Why don't an atom's electrons fall into the nucleus and stick to the protons?
Post by: Vern on 03/11/2009 17:14:01
It would really be interesting if there was experimental evidence; maybe a last instant change in one of the states that is reflected in the other. I know that has been tried. All the attempts I know about failed.
Title: Re: Why don't an atom's electrons fall into the nucleus and stick to the protons?
Post by: litespeed on 03/11/2009 19:41:45
Vern - You wrote: "There is a cause for quantum phenomena just as there is a cause for uncertainty."

I agree. SOMETHING caused an individual Uranium atom to decay. We just do not know what the hell it is. Perhaps it is a simply some sort of harmonic in the electron field that works a bit like "The Buterfly" effect.

Personally, I have become increasingly convinced our four dimensional world is entangled with one, or probably several other "Dimensions". In our universe NOTHING transits from point A to point B through an infinite number of points. I am unaware of ANY motion that does not pop in and out of our universe according to the various Plank Units.

Perhaps our universe has time movement, but not particle movement. As time progresses paricles move in and out of a timeless 'holding' dimension producing an effect something like a motion picture.
Title: Re: Why don't an atom's electrons fall into the nucleus and stick to the protons?
Post by: litespeed on 03/11/2009 19:55:35
Vern

I have a couple of observations concerning Quantum Mechanics that may or may not be relevant.  First, the Drake Equation shows that entangled particles are not a local phenomena.  That means that entangled particle A and entangled particle B do not change polarity symultaneously because they were both 'programed' at the time of separation.

I see no possible way to explain this other then to accept some sort of extra dimensional involvement, that theoretically could communicate faster the the speed of light. I send a series of entangled particles in your direction, followed by a similar sequence of non entagle particles. You notice the diference, and work out some sort of Morse code with the senders. Almost instantaneously you have joined a universeal communications exchange.  Of course the signal SENT to you travels at light speed. The subsequent communications is instantaneous.
Title: Re: Why don't an atom's electrons fall into the nucleus and stick to the protons?
Post by: Vern on 03/11/2009 19:56:14
Quote from: litespeed
Personally, I have become increasingly convinced our four dimensional world is entangled with one, or probably several other "Dimensions". In our universe NOTHING transits from point A to point B through an infinite number of points. I am unaware of ANY motion that does not pop in and out of our universe according to the various Plank Units.

I think you're making a huge assumption here. I know of no experimental evidence that movement is quantized.
Title: Re: Why don't an atom's electrons fall into the nucleus and stick to the protons?
Post by: Vern on 03/11/2009 19:59:58
Quote from: litespeed
I see no possible way to explain this other then to accept some sort of extra dimensional involvement, that theoretically could communicate faster the the speed of light. I send a series of entangled particles in your direction, followed by a similar sequence of non entagle particles. You notice the diference, and work out some sort of Morse code with the senders. Almost instantaneously you have joined a universeal communications exchange.  Of course the signal SENT to you travels at light speed. The subsequent communications is instantaneous.

I have seen several attempts to show that this could happen. As far as I know they have all failed. I have never seen a proof for wave function collapse in the Copenhagen sense.
Title: Re: Why don't an atom's electrons fall into the nucleus and stick to the protons?
Post by: litespeed on 03/11/2009 20:03:29
Vern,

I agree it is a big leap. Howver, the Heisenberg Uncertanty priciple seems to support the notion. Further, I would like your discussion on Plank Units. IMHO, these seem to support a kind of granularity in our universe.  For instance, there is a minimum distance between A and B that can not be subdevided.  Similarly, Plank time seems to support a minimum unit of time that can not be subdevided.

Of course my understanding of Plank Units is very likely flawed.  However, I have actually seen explanations of the Big Bang that include things like Plank Zero is null, Plank Two is such and such proportion of the inflation etc etc.

My basic point is that it seems to me nothing in our universe EVER moves. It simply moves in and out of time.  Just me rambling....
Title: Re: Why don't an atom's electrons fall into the nucleus and stick to the protons?
Post by: Vern on 03/11/2009 21:42:16
I have not yet signed on to quantum units other than the quantum of light. That is because I have a speculative cause for How Come The Quantum (http://www.thenakedscientists.com/forum/index.php?topic=26456.0) that assigns the cause to a property of the photon. I guess when you dwell on a subject for a long time it kinda sets in your mind and makes it difficult to contemplate another scenario for the action in mind.

Quote from: the link
Quantum Phenomena:
How come the quantum then is because empty space has limits on the amount of electric and magnetic amplitude it can support. These limits cause Planck's constant. These limits therefore cause the quantum nature of the universe. We have not invented anything new for this realization. We just noticed the obvious cause for a well known effect. But we only noticed it because we looked for a cause for the quantum effect.
Title: Re: Why don't an atom's electrons fall into the nucleus and stick to the protons?
Post by: thedoc on 04/11/2009 11:50:44
Listen to the answer to this question on our podcast. (http://www.thenakedscientists.com/HTML/podcasts/show/2009.11.01/)
Title: Re: Why don't an atom's electrons fall into the nucleus and stick to the protons?
Post by: Jarek Duda on 02/02/2013 17:03:46
Heisenberg uncertainty principle restricts measurement capabilities, not what objectively happens there - I completely disagree with such explanation by eye shutting ...
Quantum phenomenas are much more subtle (like interference), for example we can make expansion around extremely small Planck's constant (semiclassical WKB approximation (http://en.wikipedia.org/wiki/WKB_approximation)) and in zeroth order we start with the classical mechanics.
So there should be already a classical explanation of such brutal property like not falling against Coulomb attraction ... and indeed there is - it is enough to remind that electron is not only a charge, but has also very strong magnetic dipole moment - is tiny magnet. So if they would try to fall into each other, while placing reference frame in the electron, proton/nucleus is moving in magnetic field of electron - there appears perpendicular Lorentz force bending the trajectory, so even classically they would have to miss each other.

The complete Lagrangian including electron's magnetic dipole moment (c9faf6ead2cd2c2187bd943488de1d0a.gif) looks like that ( http://en.wikipedia.org/wiki/Free-fall_atomic_model ):
73b63e5489a49ea9598199081fe7cc1a.gif
Title: Re: Why don't an atom's electrons fall into the nucleus and stick to the protons?
Post by: yor_on on 02/02/2013 20:01:56
Yeah, reading it I agree with homely physicist. You can't ignore the Pauli exclusion principle as that is what defined matter macroscopically. Although the Heisenberg exclusion principle is also important, but there depending on how far you want to take it. As a way of thinking or as a real property of the universe.
Title: Re: Why don't an atom's electrons fall into the nucleus and stick to the protons?
Post by: Jarek Duda on 02/02/2013 20:29:15
Schrodigner picture represents complex electron dynamics as a simple wavefunction - Pauli exclusion principle only says that there cannot be multiple repelling particles in the same dynamical state.
This principle doesn't need to be artificially included - it is already there in Schrodinger equation alone: if we don't treat electrons independently, but include their interaction - use 4aedcdf1249f085b09617897d51d58a8.gif with repulsive potential, such two-electron wavefuntion has to vanish on diagonal: when potential goes to infinity.
And this principle doesn't work for attracting particles, like electron-positon pair would just annihilate ... we cannot use Pauli principle in proton-electron case.

The Heisenberg principle, on the other hand, says that measurements influence the system - affect eventual additional measurements of noncommuting observables - it concerns only extremely subtle category of phenomenas: measurements (projections - not unitary!).
But atom "works" even without measurements - without applying Heisenberg principle ...

Quantum mechanics gave physicists universal answers when they don't understand: "it's quantum", "it's uncertainty" ... but maybe we can search for the real understanding, concrete answers ... understand the underlying dynamics (like in Couder's picture (http://www.thenakedscientists.com/forum/index.php?topic=46639.0)).
Title: Re: Why don't an atom's electrons fall into the nucleus and stick to the protons?
Post by: yor_on on 03/02/2013 00:53:54
Doesn't matter (ahem:) if they vanish meeting Jarek, well, as i see it :)
It's about each particle of rest mass craving a unique space-time position, not willingly sharing it with others. There is more to it naturally as with helium4 etc, but that's how I see it from a simplified definition. And without that principle matter should become chaotic as I think, and the chair might become?? (possibly :) Anti matter or matter, they are still  defined as rest mass, as I understands it.

And yeah, you hit a very delicate point there discussing HUP.

What is a 'observer'?
Does it need consciousness to be defined as such?
Or is it enough with something, interacting with something else?
Title: Re: Why don't an atom's electrons fall into the nucleus and stick to the protons?
Post by: yor_on on 03/02/2013 01:12:10
But there are some weird effects to it, thinking of it from the probability of finding a electron in a atom. The electron (in its orbital inside the atom) is from the point of probability 'smeared out' as I understands it. The measurement alone must then be the definition of 'where it is/was'

And that is not the exact same as defining a unique 'place' to/for each particle of rest mass. But macroscopically I find the Pauli exclusion principle to be what keeps us existent.
Title: Re: Why don't an atom's electrons fall into the nucleus and stick to the protons?
Post by: evan_au on 03/02/2013 04:49:25
The short answer is that a "proton and electron stuck together" does happen, in a neutron.

However, a neutron is unstable, and will break down in about 15 minutes, releasing an electron (beta particle) and proton, plus a ghost-like particle called a neutrino. This decay releases a lot of energy. So, a hydrogen atom (=proton+electron) is much more stable than a isolated neutron.

Neutrons can be stable, if they are combined into an atomic nucleus with protons in the right ratio. In this case, the strong nuclear force provides the binding force to keep the nucleus stable.
Title: Re: Why don't an atom's electrons fall into the nucleus and stick to the protons?
Post by: Jarek Duda on 03/02/2013 08:10:42
Quote
It's about each particle of rest mass craving a unique space-time position, not willingly sharing it with others. There is more to it naturally as with helium4 etc, but that's how I see it from a simplified definition. And without that principle matter should become chaotic as I think, and the chair might become?? (possibly :) Anti matter or matter, they are still  defined as rest mass, as I understands it.
But electron and positron do will to share the same position ... while electrons avoid themselves because of Coulomb repulsion itself, and protons similarly.
Just imagine two-electron wavefunction 4aedcdf1249f085b09617897d51d58a8.gif. It is extremely difficult to calculate it so usually they only consider energy corrections, but the wavefunction is also very different from for two noninteracting electrons because of the 1/|x-y| repulsive potential in this 3+3 dimensional space - it has infinite potential barrier on the diagonal: meaning electrons avoid themselves because of repulsion ("exclusion principle").

The only missing is understanding why against Coulomb attraction, electron doesn't fall into proton, but exclusion principle doesn't help for attracting particles (e.g. electron+proton->neutron).
And it doesn't have to - as I have written, it is enough to remember that electron is also relatively strong tiny magnet - it creates Lorentz force while trying to fall into proton - bending trajectory such that it will always miss.

About helium4 superfluid, shouldn't you rather say that they are bosons so these atoms should be all in the same quantum state? :)
But in fact it is just nonzero volume fluid ...
Quantum "smart sounding phrases" are great when you don't understand but need to say something ... but these are simplifications - bosons doesn't exactly choose the same state, exclusion of repulsive fermions is already there in Schrodinger equation ... we shouldn't be satisfied with such mystical answers, but need to get deeper ...
Quote
And yeah, you hit a very delicate point there discussing HUP.
What is a 'observer'?
Does it need consciousness to be defined as such?
Or is it enough with something, interacting with something else?
But I didn't want to - I have only pointed out that using it to explain why electron doesn't fall into nucleus is a nonsense...
Indeed "conscious?" observer seems to be extremely delicate point ... but he is made of the same atoms governed by the same physics, so extending the system to include him, the problem disappears ... and vanishes completely when we think of the whole universe as the system - the wavefunction of the universe. It no longer has an external observer, exterior to interact with, so there are no longer wavefunction collapses like measurements - we have unique objective unitary evolution.
And so we can consider objective physics to understand why the atom works - without external observers, measurements, Heisenberg principle ...
Quote
But there are some weird effects to it, thinking of it from the probability of finding a electron in a atom. The electron (in its orbital inside the atom) is from the point of probability 'smeared out' as I understands it. The measurement alone must then be the definition of 'where it is/was'.
This smearing tries to forget about the particle part of wave-particle duality. But now we can measure where exactly was the electron before leaving the orbital - here is such picture (http://prb.aps.org/abstract/PRB/v80/i16/e165404) made by averaging positions of many single electrons:
(https://www.thenakedscientists.com/forum/proxy.php?request=http%3A%2F%2Fblogs.knoxnews.com%2Fknx%2Fmunger%2Fatomsphotos.jpg&hash=1d84f284050539b4c7edd1a4d6dea5fc)
So physically the electron was somewhere in the orbital in the moment of being stripped off - remain both wave and particle like Couder's droplets (http://www.thenakedscientists.com/forum/index.php?topic=46639.0) and wavefunction describes only its averaged position density and relative phases of its wave nature.
Quote
The short answer is that a "proton and electron stuck together" does happen, in a neutron.
Indeed ... and Pauli exclusion principle doesn't prevent such sticking together of two parts of matter :)
However, as you have written, because of strong interaction this state has much higher energy ... so what ground state hydrogen atom is, is just the lowest energetic state of electron-proton pair (excluding proton decay): electron cannot fall into the nucleus just because it would increase energy.
Title: Re: Why don't an atom's electrons fall into the nucleus and stick to the protons?
Post by: lightarrow on 03/02/2013 14:19:09
Part of the hydrogen electron's life *is* lived in the nucleus: its wavefunction square modulus is non-zero there; it has even the greatest value, there!
Infact |a11bd56a0ff5973a5604bb3fc9142b1d.gif|2 goes as e-r where r is the electron distance from the nucleus' centre.

For example, look for R(r) here:
http://hyperphysics.phy-astr.gsu.edu/hbase/quantum/hydwf.html

and look at the first picture here:
http://users.aber.ac.uk/ruw/teach/237/shape.php
<<The radial solutions of the Schrödinger equation of the hydrogen atom, R(r), are plotted on the right. Each time the quantum number n increases, an additional node is created. At n=1, the radial function is all positive. Its maximum is at r=0, i.e. the point in space with the highest probability density of finding the electron is actually inside the nucleus! That is why the term probability density is used: As we move outward along the radius, the volume of a shell of equal thickness is getting larger and larger, thereby spreading out the probability over a larger volume. >>
Title: Re: Why don't an atom's electrons fall into the nucleus and stick to the protons?
Post by: Jarek Duda on 03/02/2013 15:21:12
Indeed the simplest Schrodinger equation leads to that the maximum of electron density is exactly where the proton is ... but this model is just one point charge in potential of another fixed point charge - greatly simplifies the real physics. In the real world electron being in the same place as proton would mean that they create neutron, but it would require relatively huge energy: 782keV.  So including strong force holding baryons together would rather remove this density maximum from the Schrödinger's ground state.

This simplest Schrödinger picture misses much more, like magnetic dipole moments, relativistic corrections, interaction with environment ... it is rather surprising that it works so well, especially as Nuclear shell model (http://en.wikipedia.org/wiki/Nuclear_shell_model) where they model this unbelievably complex internal structure of large nucleus with just a simple potential well.
Connecting with independence of environment behavior, which should be seen as thermal noise, we see how unbelievably strong this universality of Schrödinger's ground state is ...

... and indeed it should be - if we make "classical" thermodynamical considerations of corpuscular entities, it turns out that models based on the fundamental in statistical physics: maximal uncertainty principle - Maximal Entropy Random Walk (http://prl.aps.org/abstract/PRL/v102/i16/e160602), in opposite to standard "generic random walk" only approximating this principle, also leads to stationary probability density being exactly squares of coordinates of dominant eigenvector of corresponding Hamiltonian: the quantum ground state. Here is comparison of such "classical"(approximated) and "quantum"(corrected) random walks on defected lattice - the second has strong (Anderson's) localization properties:
(https://www.thenakedscientists.com/forum/proxy.php?request=http%3A%2F%2Fdl.dropbox.com%2Fu%2F12405967%2Fconf.jpg&hash=eb22d7e63981757f1e12c115e530cdae)
Title: Re: Why don't an atom's electrons fall into the nucleus and stick to the protons?
Post by: yor_on on 03/02/2013 15:32:43
Be that how it might Jarek, but your question about measurements is one that has been on my mind too, but in the form of 'observers', and of course 'consciousness'. And it is important to define it I think. My own view of it is that as long as you define a 'observer' as 'something' being in a interaction with 'something else' the Copenhagen interpretation makes sense, and HUp seems then to be a sort of ultimate answer on the very small plane. If you on the other hand define it such as a 'measurement' always must involve something conscious, deciding to make that measurement? Then all of your objections hold water to me.
Title: Re: Why don't an atom's electrons fall into the nucleus and stick to the protons?
Post by: yor_on on 03/02/2013 16:06:44
As for bosons and fermions?

Helium 4 has a rest mass, but is defined as a boson according to Bose-Einstein statistics. Its nucleus has a atomic mass (u) of 4.0026 u. That makes it a member of the Pauli exclusion principle at normal temperatures as I understand, although acting (much) as a boson when as a condensate. The definition of a boson is hinging on the spin, and there the physics differ between a even (bosons) or uneven (fermions) amount of 'spins', counted up all together (net nuclear spin + electrons spin etc etc)  for whatever atom/particle under discussion, defining how the particle will act, as a boson, or as a fermion following Fermi-Dirac theory. But there are differences to 'bosons' too, or you might otherwise be able to expect helium4 to be massless, time less, and move at 'c' :)
Title: Re: Why don't an atom's electrons fall into the nucleus and stick to the protons?
Post by: Jarek Duda on 03/02/2013 16:43:00
The need of consciousness of observer makes it extremely mystical ...
I think the best experiment to understand measurements is the Stern-Gerlach (http://en.wikipedia.org/wiki/Stern%E2%80%93Gerlach_experiment) - it doesn't need any consciousness and we can get intuitions classically.
So imagine a particle with a randomly chosen direction of spin goes through such conditions (gradient of magnetic field) making it align in a line: to have spin up or down. The nearest to "up" the initial spin was, the larger probability - so this is measurement for "Pauli z matrix" observable - with two eigenvectors: spin up or down.
This measurement definitely modify the state: from a random one into one of two ... placing a few of them in different directions behaves accordingly to their order as Pauli matrices don't commute ...
Anyway, while we used to see them as something basic, measurements are physically quite subtle and complex phenomenas ...

Ok, let us look also at conscious observer situation - e.g. Schrödinger's cat.
So imagine there is a cat killed by practically random incident like nuclear decay and two observers: one near the cat, and the other separated - for simplicity let us imagine he is spatially separated, like a light year away.
Now after accidentally killing the cat, he will immediately become dead for the knowledge of nearby observer ... but for the knowledge of far observer, he will be in superposition of life and death ...
It seems there is a conflict here - while objectively cat is dead xor alive, it looks like these two observers use different quantum mechanics ... suggesting that QM only represents their knowledge ...
In fact accordingly to QM of far observer, the situation is rather:
(|cat is dead, near observer knows that cat is dead> + |cat is alive, near observer knows that cat is alive>)/sqrt(2)
so the atoms building the "conscious observer" becomes just part of physics around the cat ...

----------------------
About superfluid helium4 - indeed it is seen as made of bosons because of even multiplicity of 1/2 spin, but being all in the same quantum state is huge approximation here as it is just a liquid which can have practically any volume - liquid of electromagnetically binded alphas and electrons loosing the viscosity.
The situation is better for not composed bosons like photons, what is used in lasers for stimulated deexcitation ... but the essence here is to understand why the presence of photons makes it easier to release energy from excited atoms - understand their internal dynamics instead of just saying that photons are bosons ...
Title: Re: Why don't an atom's electrons fall into the nucleus and stick to the protons?
Post by: lightarrow on 03/02/2013 22:12:03
Indeed the simplest Schrodinger equation leads to that the maximum of electron density is exactly where the proton is ... but this model is just one point charge in potential of another fixed point charge - greatly simplifies the real physics. In the real world electron being in the same place as proton would mean that they create neutron, but it would require relatively huge energy: 782keV.  So including strong force holding baryons together would rather remove this density maximum from the Schrödinger's ground state.

This simplest Schrödinger picture misses much more, like magnetic dipole moments, relativistic corrections, interaction with environment ... it is rather surprising that it works so well, especially as Nuclear shell model (http://en.wikipedia.org/wiki/Nuclear_shell_model) where they model this unbelievably complex internal structure of large nucleus with just a simple potential well.
Connecting with independence of environment behavior, which should be seen as thermal noise, we see how unbelievably strong this universality of Schrödinger's ground state is ...

... and indeed it should be - if we make "classical" thermodynamical considerations of corpuscular entities, it turns out that models based on the fundamental in statistical physics: maximal uncertainty principle - Maximal Entropy Random Walk (http://prl.aps.org/abstract/PRL/v102/i16/e160602), in opposite to standard "generic random walk" only approximating this principle, also leads to stationary probability density being exactly squares of coordinates of dominant eigenvector of corresponding Hamiltonian: the quantum ground state. Here is comparison of such "classical"(approximated) and "quantum"(corrected) random walks on defected lattice - the second has strong (Anderson's) localization properties:
(https://www.thenakedscientists.com/forum/proxy.php?request=http%3A%2F%2Fdl.dropbox.com%2Fu%2F12405967%2Fconf.jpg&hash=eb22d7e63981757f1e12c115e530cdae)
I have to admit to have understood nothing of what you have written, maybe it's outside of my knowledge possibilities.
Just for the news, you are the same J.Duda of the Phys. Rev. article you linked and of this: http://arxiv.org/abs/0910.2724 ?
Title: Re: Why don't an atom's electrons fall into the nucleus and stick to the protons?
Post by: Jarek Duda on 03/02/2013 22:38:26
What you don't understand? I have only pointed out that this density maximum in the center is a nonsense from the point of particle physics (binding proton with electron would cost m_n-m_p-m_e=782keV). This simple Schrödinger equation ignores much more physical aspects, but still gives impressively good agreement, even in nuclear shell model - it is because the quantum ground state is something extremely universal, also from thermodynamical point of view as Maximal Entropy Random Walk shows (these papers and my last PhD thesis (http://dl.dropbox.com/u/12405967/phdphys.pdf) was about) ...
Title: Re: Why don't an atom's electrons fall into the nucleus and stick to the protons?
Post by: yor_on on 04/02/2013 01:36:22
Jarek, what lightarrow mean is that you have had a long hard thinking about this, with friends presumably. We are new to the subject, and it might well be that we miss what you consider obvious. You keep coming back to the quantum ground state btw, can you expand on how you see that? One universal ground state, is that what you mean? And yes, spin states are a mystery to me :) How they can define matter from 'bosons'. So does your model simplify it, or explain them?
=

This random walk you're describing, would that then be a mechanism that we can foresee? It's what sets the spin states, if I get you right? But it would still be governed by probability, or are you saying that your model give us a tool for a 'classical explanation' that is predictable?
=

I'm probably jumping to conclusions here, but there is one more thing that intrigue me with your ideas. You refer to particles as possibly having 'internal clocks'. If I now assume that a particle, not atom, but let's say a electron can't be split in more parts, what does a internal clock means? That the arrow becomes a 'force' of sorts too? It seems to me that if I assumed a intrinsic time keeping for particles I also lift up time as a real 'dimension'?
Title: Re: Why don't an atom's electrons fall into the nucleus and stick to the protons?
Post by: lightarrow on 04/02/2013 20:27:45
What you don't understand? I have only pointed out that this density maximum in the center is a nonsense from the point of particle physics (binding proton with electron would cost m_n-m_p-m_e=782keV). This simple Schrödinger equation ignores much more physical aspects, but still gives impressively good agreement, even in nuclear shell model - it is because the quantum ground state is something extremely universal,
...and this is quite simple to understand.
Quote
also from thermodynamical point of view as Maximal Entropy Random Walk shows (these papers and my last PhD thesis (http://dl.dropbox.com/u/12405967/phdphys.pdf) was about) ...
...and this is the less simple part  [:)]
Title: Re: Why don't an atom's electrons fall into the nucleus and stick to the protons?
Post by: Jarek Duda on 04/02/2013 21:08:19
Probability density of the quantum ground state is universal from QM point of view because other states are excited - have higher energy and so want to release this energy, deexcitating down to the ground state - so this is kind of thermal equilibrium state (in 0K).
Stochastic models also predict some probability densities for these situations, but standard models predict different from QM (much weaker localization properties). Maximal Entropy Random Walks(MERW) allows to understand this conflict - it is because standard models only approximate the basic for thermodynamics: maximal uncertainty principle. If we do it right, there is no longer conflict - MERW also leads exactly to the ground state probability density.
It also gives natural intuition of the Born rules that probability is square of amplitude (leading to violation of Bell inequalities): in this model amplitude is probability on the end of past and simultaneously on the beginning of future - to get real probability in given moment we have to multiply them. This "fourdimensional understanding" allows also to get intuitive understanding of why quantum computers are stronger than classical: because they can "mount" qbit trajectories in both past (initialization) and future (measurements). Here is schematic picture of Shor's algoritm (description (http://www.thenakedscientists.com/forum/index.php?topic=46630.0)):
(https://www.thenakedscientists.com/forum/proxy.php?request=http%3A%2F%2Fdl.dropbox.com%2Fu%2F12405967%2Ffqcomp.jpg&hash=666701825bac414278d49213ff106ed1)
Quote
This random walk you're describing, would that then be a mechanism that we can foresee? It's what sets the spin states, if I get you right? But it would still be governed by probability, or are you saying that your model give us a tool for a 'classical explanation' that is predictable?
MERW is thermodynamical model, that means predicting the most probable evolution. It is obtained for the maximal uncertainty principle, what basically means that if there is no reason to emphasize any scenario, we should assume uniform probability distribution. So it is not about foreseeing some concrete scenario, but operating on our knowledge - like what stationary probability we should assume, or if we know where it is in one moment, what probability density we should assume after some time (propagator).
These are completely general considerations - spin is something much more subtle (approximately the direction of magnetic dipole moment).
Please ask if you have some questions ... here are slides about MERW (http://dl.dropbox.com/u/12405967/phdsem.pdf).

Quote
I'm probably jumping to conclusions here, but there is one more thing that intrigue me with your ideas. You refer to particles as possibly having 'internal clocks'
It's extremely offtopic here ... while there is topic about it (http://www.thenakedscientists.com/forum/index.php?topic=46639.0), maybe let us take it there - it was de Broglie's idea (see Hestenes paper), and the Couder's droplets give great intuition of such view on wave-particle duality and basic quantum phenomenas from this point of view ...
Title: Re: Why don't an atom's electrons fall into the nucleus and stick to the protons?
Post by: yor_on on 05/02/2013 10:27:13
So you do this from assuming a even probability 'density' to the universe? "we should assume uniform probability distribution". So I got it all wrong :) when I wondered if you were trying for a 'classical' (Newtonian?) definition. I will have to read up on the maximal uncertainty principle, it's new to me. As for a uniform probability it makes sense to me, as long as we ignore interactions, if that is how you mean? I'm good at jumping to conclusions :) And I like new ideas, and yours are new to me.
==

Btw, anything that can simplify or visualize my understanding of quantum logic and their effects, as you seem to imply in the other thread comparing macroscopic systems to quantum effects, are welcome to me :)

I'm still stuck on the simple experiment where we split a photon in two (down converting its energy) Getting either the 'spooky action at a distance', or 'hidden variable(s)' defining the outcome. Because I see no way identical photons, whose polarization you can't predict (50% chance either way) before the measurement, as proven experimentally, still always result in the other photon 'knowing' which way the polarization was, and setting the opposite polarization.

How would you describe that from your view? Or maybe that is outside the subject?
Title: Re: Why don't an atom's electrons fall into the nucleus and stick to the protons?
Post by: Jarek Duda on 05/02/2013 10:50:14
In terms of random walk, one of equivalent formulation of maximal uncertainty principle says that having absolutely no information about what trajectory the object will chose, we should assume uniform probability distribution among all possible paths - it is MERW. Another formulation is by maximizing entropy.
In physics we emphasize some scenarios by assigning them energy - replacing uniform distribution with Boltzmann distribution ... we can also consider multiple particles with interactions between them through potential (in analogous way as in quantum mechanics) ... please at least look at sources like slides (http://dl.dropbox.com/u/12405967/phdsem.pdf) before asking further (especially we are offtopic)..
Title: Re: Why don't an atom's electrons fall into the nucleus and stick to the protons?
Post by: yor_on on 05/02/2013 10:57:00
K will do.
Title: Re: Why don't an atom's electrons fall into the nucleus and stick to the protons?
Post by: jccc on 11/05/2014 20:20:18
 
This is my own atomic structure theory. I think the space is negative charged elastic fluid . A positive particle such as proton will attract space to form a negative ball field around it. When an electron closing to a proton, this ball field pushes it away. The balance point is the diameter of the atom. Electrons does not fly around nuclear but bond by the ball field and proton forces. Since the space itself is charged, it conducts electromagnetic force such as light waves. Light wave is coming from electron vibrating in space.
Title: Re: Why don't an atom's electrons fall into the nucleus and stick to the protons?
Post by: jccc on 12/05/2014 02:45:59
This magnet toy is a good demo for a hydrogen atom.
Title: Re: Why don't an atom's electrons fall into the nucleus and stick to the protons?
Post by: PmbPhy on 06/06/2014 07:20:28
Quote from: Sarah Raphaella
I know that protons are positively charged, neutrons are neutral, electrons are  negatively charged and that atoms are mostly empty space. I also know for magnets opposites attract. … So why don't electrons stick to protons instead of flying around the nucleus?
The laws of physics at the atomic level is based on what is known as Quantum Mechanics and not on the physics that you’ve probably only learned to date, i.e. Newtonian Physics which is now known as Classical Mechanics. In the case of the atom, electrons can only exist in certain states. In those states the electrons don’t move on classical trajectories, as you might otherwise think of them moving. They are found in regions of space according to what we call the Wave Function. For the hydrogen atom the wave function contains everything that can be known about the state of the electron in the hydrogen atom. The square of the magnitude of the wave function is the probability density which is used to determine the probability of finding the electron in a particular region of space. Described in this way electrons don’t move the way you’d expect them to using classical physics. The states that the electrons can exist in are described by the wave function. For each allowed state there is an associated wave function known as an eigenstate. Each eigenstate is defined by certain numbers called quantum numbers. These numbers describe things like energy, angular momentum, spin, etc.. In chemistry these eigenstates are referred to as an Atomic Orbital. You can read about them online at
http://en.wikipedia.org/wiki/Atomic_orbital

The shapes of these orbitals are shown in this link. For the lowest energy level the electron can actually come as close to the nucleus as it wants to. I.e. the probability density at r= 0 is non-zero.

Quote from: Mr. Scientist
It has to do with the uncertainty principle. …I am sure Hawking himself said the Uncertainty Principle had something to do with it.
That is incorrect. The reason is as I just described it. The reason is not for the reason you give. Also it’s quite wrong and contrary to quantum mechanics to assert that electrons “must occupy every other space within the atom.” Such a thing is quite wrong in quantum mechanical terms. No electron can be said to exist in more than one place at one time. No electron can even be said to be at a place unless its position is measured and the electron is found to be there. And no. Hawking would not say such a thing. However I do agree that the Pauli exclusion principle has nothing to do with (and it’s called the “exclusion” principle, not the “expulsion” principle).

Quote from: Vern
The present state of physical science does not allow "why" questions.
[/quote
That is incorrect. When someone asks a question whose answer is a description in terms other than stating postulates then science can indeed address “why” questions. For example: the question Why is the sky blue? has a very definite answer to it.



Nasa Answers the question Why Is the Sky Blue at
http://spaceplace.nasa.gov/blue-sky/en/
Quote
Sunlight reaches Earth's atmosphere and is scattered in all directions by all the gases and particles in the air. Blue light is scattered in all directions by the tiny molecules of air in Earth's atmosphere. Blue is scattered more than other colors because it travels as shorter, smaller waves. This is why we see a blue sky most of the time.
and this answer is far from being speculative as you claim it must be.


Quote from: Vern
If you like to think that Quantum theory represents reality you have to invent excuses.
Nonsense.

Quote from: Vern
Quarks can not exist outside nuclei, for example. Electrons dance to the uncertainty tune, etc. To me it is much easier just to accept reality as it presents itself.
Those aren’t excuses. And we do things because of their logical consistency, correspondence with experiment, etc. Not because we want things to be “easy.” If you want easy become an auto mechanic.

Quote from: Vern
But we really don't.
Wrong. We absolutely do.

Quote from: Vern
I started looking for experimental evidence for wave function collapse years ago.
Any physicist worth his salt could have and would have told you that such a search is a waste of time. The wave function and the notion of the collapse of the wave function are merely mathematical intermediaries, not physical entities. E.g. we don’t measure the wave function in the lab. We don’t directly measure a probability either. What we measure are things like The particle detector at (x=2, y=4) “clicked” and thus registered the presence of an electron at 3:33:29pm. We keep repeating that kind of thing and then add these numbers up. We then calculate a probability density. Etc.

Quote from: Vern
I'm still looking. None found.
If that’s true then it’s because you didn’t understand the theory and thus didn’t know what to look for or how to look for it. We can certainly observer nature and conclude that nature is consistent with the concept of wave function collapse.

Quote from: Vern
We have a habit of reporting our conclusions as experimental results.
[/quotes]
Who is “we”? I know of nobody that ignorant.


Title: Re: Why don't an atom's electrons fall into the nucleus and stick to the protons?
Post by: PmbPhy on 06/06/2014 07:26:52
Quote from: evan_au
The short answer is that a "proton and electron stuck together" does happen, in a neutron.
That is quite incorrect. The neutron cannot be thought of that way, It can be shown that an electron cannot exist inside a neutron and exist as a neutron/electron system. I can't  recall where I came across that fact but no matter. It's a well-known fact. I had to prove it as part of my studies of quantum  mechanics.
Title: Re: Why don't an atom's electrons fall into the nucleus and stick to the protons?
Post by: chiralSPO on 06/06/2014 20:25:58
Quote from: evan_au
The short answer is that a "proton and electron stuck together" does happen, in a neutron.
That is quite incorrect. The neutron cannot be thought of that way, It can be shown that an electron cannot exist inside a neutron and exist as a neutron/electron system. I can't  recall where I came across that fact but no matter. It's a well-known fact. I had to prove it as part of my studies of quantum  mechanics.

There is no discrete electron-proton pair within a neutron--it is a single particle. However a neutron is the result of a proton "capturing" an electron: p+ + e →  n  +  νe
Title: Re: Why don't an atom's electrons fall into the nucleus and stick to the protons?
Post by: alancalverd on 06/06/2014 23:36:12
The fact that atoms don't collapse just shows that the classical electron-proton model is inadequate. There is no "why" in nature: stuff happens, and the best we can do is to generate predictive models of what happens. The classical model of electrostatics works pretty well for widely separated charges but just doesn't describe the behaviour of electrons in an atom - and there's no reason why it should.

The test of quantum theory is whether it describes what we see at a very small scale, and reduces to the classical continuum description at the mesoscopic scale: and it does. The reverse test, attempting to describe small objects form the behaviour of large ones just doesn't work.   
Title: Re: Why don't an atom's electrons fall into the nucleus and stick to the protons?
Post by: PmbPhy on 07/06/2014 00:29:27
Quote from: chiralSPO
There is no discrete electron-proton pair within a neutron--it is a single particle.
Not according to the Stadard Model. In particle physics the neutron is not a single particle but a system of three particles calledquarks of which there are several types. The neutron is composed of two down quarks and one up quark. When Murry Gell-Mann developed the theory of quarks it was just a nice gimmick to help describe what was being observed. Later on Gell-Mann decided to accept the reality of them as being "real" particles. Deep inelastic scattering shows that in the case of the proton the evidence suggests three lumps of charge instead of one. This is strong support for the quark model.

See see http://en.wikipedia.org/wiki/Neutron

Quote from: chiralSPO
However a neutron is the result of a proton "capturing" an electron: p+ + e →  n  +  νe
Just because a proton can be created that way it doesn't mean that's the only way and it doesn't mean that's what a neutron "is." There are other ways to create neutrons. It is therefore wrong to identify a neutron as "that which results when a proton captures an electron."
Title: Re: Why don't an atom's electrons fall into the nucleus and stick to the protons?
Post by: PmbPhy on 07/06/2014 01:38:23
Quote from: jccc
Let's pretend Enertron is real, ...
That's equivalent to saying []Let's pretend that nature does not behave the way that we observe that it does and see what happens.[/i] That can result in anything that you'd like because what you're describing goes by another name, i.e. magic.

Magic is like anything that you'd like it to be so you're now free to create anything that you'd like. Have fun.
Title: Re: Why don't an atom's electrons fall into the nucleus and stick to the protons?
Post by: PmbPhy on 07/06/2014 02:40:28
Quote from: jccc
Enertron should play a roll in many things including energy transfer, energy density, temperature etc,.
Why?

Each particle that exists is able to exist because it has all the properties which allow it to exist according to the laws of physics. Now you come along and say "The Enertron particle exists." which quite literally means that it exists outside the range of normal experience. That’s the ramification of assuming what you’re telling us to assume. Do you know what that’s called? I.e. do you know what we call a phenomenon that exists outside the range of normal range of experience? It has a very particular name. It’s know as the Paranormal. See http://en.wikipedia.org/wiki/Paranormal

Magic is the attempt to control or otherwise work with the paranormal. See
http://en.wikipedia.org/wiki/Magic_(paranormal)

Quote from: jccc
Open mind, watch and think, predict and test. Isn't that science?
If you’re doing it with respect to those things that exist in nature than it’s called science. If you’re creating things out of thin air which in doing so violate the laws of nature by their very existence then no. That’s not science. That’s magic.

Quote from: jccc
I suggest a model …
No you didn’t. At least not yet. All you said was Let's pretend Enertron is real,.. which is the furthest thing from a model that you can get.

Quote from: jccc
...to explain atomic structure, created enertron sub particle idea, not magic. Not as magicle as QM.
There’s nothing in any of your recent posts which explains anything, never mind atomic structure. You never created an “enertron”. You merely pretended it existed. When you did so and did so outside the laws of nature then you’re talking about the paranormal.

Please find a dictionary and look up the term “paranormal.”

Perhaps you have no training whatsoever in particle physics. If so then that’d explain a few things. Particle physics is a theory of elementary particles. It tells us what particles can exist and what the properties of those particles are. In this thread I’m assuming the idea situation by which there exists a theory of particles whose properties we fully know. Also in this thread I’m assuming that the theory which we’re assuming that we have is able to fully account for the existence of all particles that either exist now or can be created. I’m also assuming in this thread that this “enertron” is something you dreamed up which is something which is not on any list of currently known particles. If it is then please tell me where to find the list on which this particle exists.
Title: Re: Why don't an atom's electrons fall into the nucleus and stick to the protons?
Post by: PmbPhy on 07/06/2014 04:32:05
Quote from: jccc
I haven't think out a way to detect it to proof or disproof its existence, can you help?
You'd do it the same way that you'd prove or disprove the existance of unicorns.

Quote from: jccc
Physics laws are created by men, men don't create particles.
What's your point? The laws of physics are indeed created by man but they describe nature whose existance man does not dictate.

Please do yourself a major favor. Study the following articles very carefully;

http://home.comcast.net/~peter.m.brown/ref/philosophy_physics.pdf
http://home.comcast.net/~peter.m.brown/ref/what_is_science.pdf
Title: Re: Why don't an atom's electrons fall into the nucleus and stick to the protons?
Post by: JP on 07/06/2014 06:09:28
You didn't come out anything you knew besides from books.

Can you proof enertron is not there? How you explain electrons not stick to proton?
Wait few more years, learn more new theories and discoveries. Science is advancing.

Since this is a science forum, we're all here to discuss what science currently knows.  If you'd like to discuss what you think might be discovered in the future, please keep the discussion to the New Theories forum.
Title: Re: Why don't an atom's electrons fall into the nucleus and stick to the protons?
Post by: jccc on 07/06/2014 07:02:46
Quote from: Sarah Raphaella Rodgers
So why don't electrons stick to protons instead of flying around the nucleus? Magnets do it, so why can't atoms?
The present state of physical science does not allow "why" questions. Any answer will have to be speculative.

Why not? Science should be always allows why questions?
Title: Re: Why don't an atom's electrons fall into the nucleus and stick to the protons?
Post by: alancalverd on 07/06/2014 07:14:48
Why not? Science should be always allows why questions?

No. "Why" implies an ulterior purpose. There is no evidence of one, nor that fundamental particles have any knowledge of such a purpose. Purpose is a construct of living things, not a property of their constituent atoms.

Science is concerned with "how" - though biologists may occasionally ask "why" as long as they are wary of excessive anthropomorphism. Hence the truly scientific answer to why the chicken crossed the road.
Title: Re: Why don't an atom's electrons fall into the nucleus and stick to the protons?
Post by: JP on 07/06/2014 08:06:43
Science is all about applying the scientific method to come up with models of nature.  The key term (for this discussion) is models.  Science doesn't deal with coming up with a fundamental "real" cause for everything.  All we as scientists can do it to come up with accurate models and then leave it to philosophers to argue over whether the model itself is reality or whether it is simply a model of some deeper underlying reality. 

The problem with posting ideas like "I think an electron can stick to a proton" is that there is no science to back that up.  It may or may not be a good idea, but unless you can show:

a) The theory is consistent with other existing measurements
b) The theory is testable and falsifiable

It is not even on the track to being a scientific theory.
Title: Re: Why don't an atom's electrons fall into the nucleus and stick to the protons?
Post by: jccc on 07/06/2014 08:57:42
I never posted ideas like "I think an electron can stick to a proton".

I posted ideas like this.

"If the speed of force is c, then the speed limit is c.

A 100 miles per hour train cannot push a man move faster than 100 miles per hour.

The force we use is electromagnetic force, its speed is c. Therefore, we can never travel at light speed."

What's your comment? Do you think force has a speed?
Title: Re: Why don't an atom's electrons fall into the nucleus and stick to the protons?
Post by: JP on 07/06/2014 09:39:25
Force is caused by a change in momentum which is in turn caused by the exchange of particles or fields.  The speed at which one object can exert a force on another is determined by the speed of those particles or fields.  The upper speed limit to anything we know of is the speed of light, since both fields and particles obey special relativity.  Of course, forces can travel slower.

So no, there is no "speed of force."  There is speed of objects which can transfer momentum.
Title: Re: Why don't an atom's electrons fall into the nucleus and stick to the protons?
Post by: alancalverd on 07/06/2014 10:20:53

I am very confused. Isn't Newton ask why apple falls so to discover gravitation? Isn't scientist ask why there is red shift so to make big bang theory?

Why can't we ask why to any thing we don't understand?

Seriously, what is science all about?

You can indeed ask why, but the best a scientist can tell you is how.

I am being very pedantic, but for a good reason: words in science all have very precise and noninterchangeable meanings. To a journalist, force, energy and power are all the same thing but they are entirely different in physics, and the difference is crucial to understanding and describing how things work.

It is arguable that Newton was still labouring under the illusion of a created universe with a purpose, and his work certainly predated the word "scientist" which was invented in 1833. It has been suggested that belief in a purposeful creator was the reason why natural philosophers  like Newton sought rational and consistent explanations, hence the theistic "why" was entangled with "how" in their minds, and actually laid the foundation for systematic investigation of what was presumed to be a systematic universe. But an atheistic view cannot assume an ultimate purpose, indefinitely consistent systematics, or that common logic, applied ad infinitum, will explain everything: you may have to accept from time to time that "that's just the way it is", and that certainly applies to quantum mechanics. 

So we observe red shift and ask how it can be explained. You can look at the known phenomena of doppler shift and general relativity, and deduce that distant objects in general are moving away from each other, which suggest that at some time they were closer together (or that space was smaller) hence there must have been a starting point before which the universe bore no resemblance to its present state. No "why" because no need for an ultimate purpose - it just is, and apparently was, so let's untangle the mechanism of "how" it got from there to here.   
Title: Re: Why don't an atom's electrons fall into the nucleus and stick to the protons?
Post by: PmbPhy on 07/06/2014 17:35:22
Quote from: alancalverd
You can indeed ask why, but the best a scientist can tell you is how.
I used to think that was true but experience has shown me that it's not. After I took the time to sit down and look at all the questions that scientists have been asking and answering over the last hundred years I came to see how wrong I was and how wrong your assertion is. And I'm far from being the only physicists to think so too.

From The Inflationary Universe by Alan H. Guth. On page ix, the Foreword, written by Alan Lightman, reads
Quote
In the 1970’s, the study of cosmology went through a major conceptual change. Prior to this time, modern cosmologists asked such questions as; What is the composition of galaxies and where are they located in space? How rapidly is the universe expanding? What is the average density of matter in the cosmos? After this time, in the “new cosmology,” cosmologists began seriously asking questions like: Why does matter exist at all, and where did it come from? Why is the universe as homogenous as it is over such vast distances? Why is the cosmic density of matter such that the energy of expansion of the universe is almost exactly balanced by its energy gravitational attraction? In other words, the nature of the questions changed. “Why?” was added to “What?” and “How? and “Where?”. Alan Guth was one of the young pioneers of the new cosmology, asking the Whys, and his Inflationary Universe theory provided  many answers.

Quote from: alancalverd
So we observe red shift and ask how it can be explained. You can look at the known phenomena of doppler shift and general relativity, and deduce that distant objects in general are moving away from each other, which suggest that at some time they were closer together (or that space was smaller) hence there must have been a starting point before which the universe bore no resemblance to its present state. No "why" because no need for an ultimate purpose - it just is, and apparently was, so let's untangle the mechanism of "how" it got from there to here.   
Please don't take this the wrong way, but I hope that you're not trying to discourage people from asking questions in a way that feels natural to them. People want to know why certain things are the way they are. E.g. if somone wishes to ask "If the entire floor in my house has the same temperature then why does the ceramic tiled floor of the bathroom colder than the wooden floor in the kitchen?" then you shouldn't try to get them to change the way they phrase it because it has a very simple answer.

The flaw in your argument is that you're only using examples which are consistent with your assertion and are igoring those which demonstrate that you're wrong. If you used the questions that Alan Guth was asking during the research which led to the inflationary theory of the universe then your argument falls apart.

I gave an example of a "Why" question recently in this forum - Why is the sky blue? is a very legitimate question which has a very direct and valid answer. The question which started this thread is also a valid question which also has a very direct and valid answer which I also gave.

Take a look at all the usolved problems in physics at
http://en.wikipedia.org/wiki/List_of_unsolved_problems_in_physics

Notice how they're phrased:
- Why is the distant universe so homogeneous when the Big Bang theory seems to predict larger measurable anisotropies of the night sky than those observed?

- Why aren't there obvious large-scale discontinuities in the electroweak vacuum if distant parts of the observable universe were causally separate when the electroweak epoch ended?

- Why is there far more matter than antimatter in the observable universe?

- Why does the zero-point energy of the vacuum not cause a large cosmological constant?

- Why is the energy density of the dark energy component of the same magnitude as the density of matter at present when the two evolve quite differently over time;

- Why does the predicted mass of the quantum vacuum have little effect on the expansion of the universe?

- Why is gravity such a weak force? It

- Why are there three generations of quarks and leptons?

etc

There are basically two different kinds of "Why?" questions. There are the kinds which are seeking deep spiritual meaning like "Why am I the person I am rather than someone else?" or "Why did I have to get cancer?"

See - http://www.merriam-webster.com/dictionary/why

The kinds that science can address are those of "cause." The kind that science cannot answer is "reason." Therefore

Science can answer the question - "What causes the sky to be blue?"

Science cannot answer the question - "What is the reason that the sky to be blue?" if by "reason" one is asking why "God" didn't make the sky red or purple
Title: Re: Why don't an atom's electrons fall into the nucleus and stick to the protons?
Post by: lightarrow on 03/07/2014 08:02:39
After 4 pages of discussion, I still don't understand this question - Why don't an atom's electrons fall into the nucleus and stick to the protons?

Please someone help me to understand, or let me know where else to find answer.
And I don't understand why you don't want to understand that electrons in an atom are not little balls and so to describe their behaviour you have to use quantum mechanics.

--
lightarrow
Title: Re: Why don't an atom's electrons fall into the nucleus and stick to the protons?
Post by: evan_au on 03/07/2014 17:26:30
how can they not attract each other and stick together?

A very crude analogy: Imagine you had an airport, with a slight indentation down towards a drain hole that is perhaps 1 inch across.
- If you place a jumbo jet on this indentation, will it be attracted towards the drain-hole? Yes, because there is an energy gradient.
- Why doesn't the jumbo jet stick there? It will stick there, unless a greater external force moves it away.
- Why doesn't the jumbo jet go down the drainhole? It's too big, and it takes a considerable amount of energy to squash it down and turn it into a drainhole plug.
Title: Re: Why don't an atom's electrons fall into the nucleus and stick to the protons?
Post by: jeffreyH on 04/07/2014 00:24:04
The short answer is that a "proton and electron stuck together" does happen, in a neutron.

However, a neutron is unstable, and will break down in about 15 minutes, releasing an electron (beta particle) and proton, plus a ghost-like particle called a neutrino. This decay releases a lot of energy. So, a hydrogen atom (=proton+electron) is much more stable than a isolated neutron.

Neutrons can be stable, if they are combined into an atomic nucleus with protons in the right ratio. In this case, the strong nuclear force provides the binding force to keep the nucleus stable.
  • Too many neutrons, and one could decay (releasing an electron, as described above)
  • Too few neutrons, and an inner electron can be captured, forming a neutron, just as you asked
  • There are other nuclear decay paths too; for more details: http://en.wikipedia.org/wiki/Stable_nuclei

I have only just picked up on this thread and haven't read it all the way through. The fact that we can even have a particle like the neutron shows that the electron can become part of the nucleus. However as this requires energy the electron cannot become part of a proton so is forced to orbit due to the spin of the proton and the spin of the electron. If it had enough energy it would form a neutron. However if this were easier as in a property defined at the big bang every particle would be neutral and no solid matter would form as atoms would not exist. The question should not be why doesn't it fall into the nucleus but why doesn't this combination more easily form neutrons. It actually has fallen as far as it can towards the nucleus.
Title: Re: Why don't an atom's electrons fall into the nucleus and stick to the protons?
Post by: evan_au on 04/07/2014 02:57:37
The energy involved in electrostatic interactions between electrons and protons is around a few electron-Volts - the level of energy that is involved in chemical reactions.
The energy involved in the nuclear force between protons or protons & neutrons is typically around a million times stronger, at Mega-electron-Volts.

So it takes an immense amount of energy to get electrons to interact with a nucleus - an amount of energy that cannot come from normal chemical reactions. For humans to achieve such an interaction, the energy would have to come from a high-powered particle accelerator.

There is another source of the necessary energy - a nucleus which is already severely stressed by having too many protons for the number of neutrons - this stress represents MeV of potential energy. There is a very small probability that such a nucleus will capture an electron, releasing the pent-up energy in a neutrino, to balance the necessary nuclear equations.

So three reasons protons don't routinely capture electrons:
- Electrons have far too little energy (by a factor of a million or so)
- Fundamental properties must balance before and after the interaction, which does not happen without production of an energetic neutrino.
- This interaction involves the weak nuclear force, which means it can take a long time...
Title: Re: Why don't an atom's electrons fall into the nucleus and stick to the protons?
Post by: acsinuk on 04/07/2014 11:21:39
"So three reasons protons don't routinely capture electrons:
- Electrons have far too little energy (by a factor of a million or so)
- Fundamental properties must balance before and after the interaction,"
 Evan you are refering to energy balance but the only stuff inside a molecule is the electrostatic charge and magnetic forces and these need to balanced.
As the inside of the molecule must balance perfectly electrically; a strong nuclear force of 10^38 G will be needed to force the like charges in the proton bundles together. Same force applies to the neutron bundle as well.  The proton bundle must be pushed away from the neutron bundle electro-magnetic by an EMF curling force of 10^ 36 G which also pushes away the electron enclosure.  As the normal repulsion of same charges force is weaker at 10^ 25 G ; this pushes the complex molecules proton bundles away from the other compound proton bundles thus the molecule is electrostatically and electromagnetically stable.
CliveS
Title: Re: Why don't an atom's electrons fall into the nucleus and stick to the protons?
Post by: jeffreyH on 04/07/2014 21:32:34
This brings up a question about neutron stars. Do they have a magnetic field? If so then is it simply electrons that cause it? The neutrons being neutrally charged are unlikely candidates. This would make the field mono-polar.
Title: Re: Why don't an atom's electrons fall into the nucleus and stick to the protons?
Post by: chiralSPO on 04/07/2014 22:05:17
I don't know how important this is for neutron stars, but neutrons in atomic nuclei are certainly magnetic--both protons and neutrons have "nuclear spin" that results in a small magnetic field. I don't know to what extent the spins would arrange themselves to cancel out in a neutron star, but it would only take a small imbalance to have a fairly large magnetic moment. (A neutron has a spin of magnitude 1/2, some of the most out-of balance, but still stable nucleons have a spin of 7/2. Some more extreme nuclear states only last a short while (110Ag has a 12/2 nuclear spin, but a half life of only 253 days, and 43Sc has a 19/2 nuclear spin, but a half-life of only 450 ns!)
Title: Re: Why don't an atom's electrons fall into the nucleus and stick to the protons?
Post by: evan_au on 05/07/2014 01:55:50
the only stuff inside a molecule is the electrostatic charge and magnetic forces and these need to balanced.
There are 4 known forces, of which the "Strong Nuclear Force" is the strongest one we know, and it provides most of the glue that holds the nucleus together against electrostatic replulsion of the protons.

As you suggest, Gravity is the weakest, and its effect can almost be ignored on a scale any less than a neutron star.

The proton bundle must be pushed away from the neutron bundle electro-magnetic
As I understand it, the protons and neutrons are mixed together in the nucleus of an atom. The neutrons do not repel each other electrostatically, but they contribute to the strong nuclear force which holds the protons together against the electrostatic repulsion of the protons.

which also pushes away the electron enclosure
The electrons enclosing the nucleus are attracted to the protons in the nucleus, due to their opposite charge. As far as I know, the effect of neutrons on electrons is much weaker than the impact of protons on electrons.

It is the wave nature of the electrons which prevent them collapsing into the nucleus.

the only stuff inside a molecule is the electrostatic charge and magnetic forces
There are other factors which must be balanced when considering nuclear interactions.

Like macroscopic interactions between particles, mass/energy and momentum are conserved.

Interactions involving the Strong Nuclear Force also must  obey CP symmetry, which permits some interactions, and forbids others.

The Weak Nuclear force has some slightly looser constraints on which interactions are permitted and sometimes violates CP symmetry, but these interactions have much lower probability, like a nucleus consuming an inner electron.

See: http://en.wikipedia.org/wiki/CP_violation#CP-symmetry
Title: Re: Why don't an atom's electrons fall into the nucleus and stick to the protons?
Post by: jeffreyH on 05/07/2014 09:31:52
I don't know how important this is for neutron stars, but neutrons in atomic nuclei are certainly magnetic--both protons and neutrons have "nuclear spin" that results in a small magnetic field. I don't know to what extent the spins would arrange themselves to cancel out in a neutron star, but it would only take a small imbalance to have a fairly large magnetic moment. (A neutron has a spin of magnitude 1/2, some of the most out-of balance, but still stable nucleons have a spin of 7/2. Some more extreme nuclear states only last a short while (110Ag has a 12/2 nuclear spin, but a half life of only 253 days, and 43Sc has a 19/2 nuclear spin, but a half-life of only 450 ns!)

In the case of densely packed neutrons the strong nuclear force and gravitation probably act together to bind the mass. I think it is likely that an electron cloud produces the magnetic field and without a well defined positive component to the field would swamp the gravitational effect. In black holes this would become important. I am following any observations of G2 and Sag A* as this would confirm this if the cloud remains mainly intact. It would not matter if the gas cloud contains a host star.
Title: Re: Why don't an atom's electrons fall into the nucleus and stick to the protons?
Post by: jccc on 06/07/2014 01:40:34
I rather to be a neutron than an electron, how about you?
Title: Re: Why don't an atom's electrons fall into the nucleus and stick to the protons?
Post by: McKay on 06/07/2014 13:07:14
Well, here is something to read about this: http://www.wired.com/2014/06/the-new-quantum-reality
And in my mind, something like this provides a much better understanding of how things work - much better than: "an equation [or a principle] says so" ..
Title: Re: Why don't an atom's electrons fall into the nucleus and stick to the protons?
Post by: jeffreyH on 06/07/2014 16:15:20
Well, here is something to read about this: http://www.wired.com/2014/06/the-new-quantum-reality
And in my mind, something like this provides a much better understanding of how things work - much better than: "an equation [or a principle] says so" ..

I vote de Broglie. He seems to have been sidelined and never made any further significant contributions to physics. It would be a testament to his foresight and give him more significance in the history of physics. I have for years thought quantum mechanics lacking clarity. No wonder quantum gravity has eluded physics for 100 years.
Title: Re: Why don't an atom's electrons fall into the nucleus and stick to the protons?
Post by: jeffreyH on 06/07/2014 16:21:56
From the previously mentioned article.

"The experiments began a decade ago, when Yves Couder and colleagues at Paris Diderot University discovered that vibrating a silicon oil bath up and down at a particular frequency can induce a droplet to bounce along the surface. The droplet’s path, they found, was guided by the slanted contours of the liquid’s surface generated from the droplet’s own bounces — a mutual particle-wave interaction analogous to de Broglie’s pilot-wave concept."

In a modified form this describes gravity.
Title: Re: Why don't an atom's electrons fall into the nucleus and stick to the protons?
Post by: evan_au on 07/07/2014 04:27:31
Quote from: jccc
If we can force electron and proton in hydrogen atoms to marry each other, the nucleus force field will break down and release all the energy stored
You can't really use an electron to release energy from a nucleus.
- For one thing, the electron very rarely interacts with the nucleus of an atom.
- And in those rare instances where an interaction does occur, much of the energy is carried away by the ghostly neutrino, which we can't capture as an energy source.

Combining two deuterium nuclei (or a dueterium & a tritium) to make a Helium nucleus does release a usable amount of energy.
- However, the wavelength of an electron is too long to draw the two nuclei together close enough for a nuclear reaction to occur at a usable rate.
- The wavelength of a negative muon is short enough to react deuterium nuclei together; unfortunately, the lifetime of the muon is too short to generate usable amounts of energy. (Or fortunately - if the muon were much better at catalysing a nuclear reaction between hydrogen nuclei, much of the hydrogen in the universe would already have fused to Helium...)
Title: Re: Why don't an atom's electrons fall into the nucleus and stick to the protons?
Post by: evan_au on 08/07/2014 08:33:53
Quote
If energy is stored within nucleus force field, it should be negative charged in nature.

I don't think that conclusion is correct. The way I understand it:
Title: Re: Why don't an atom's electrons fall into the nucleus and stick to the protons?
Post by: chiralSPO on 10/07/2014 23:33:06
How do you calculate "10^6 nucleus energy" out for e + p+ → n ??

The equation E = keQ1Q2(1/rinitial–1/rfinal) gives no sensible answer if rfinal = 0 (as one might expect for the merger of the two point charges). Some theories predict charge separation within the neutron, but as far as I know, this has not been verified experimentally in any way. If I try values of about 1–2 fm as the final charge separation (approximate radius of a neutron) energy release is on the order of 2x10–13 J...  What was your calculation? What is nucleus energy? If the whole neutron were converted to energy (E = mc2), I calculate that as 1.5x10–10 J, which is not a factor of 106 different from anything... I am confused.

At any rate, we know experimentally that neutrons spontaneously decay exothermically (releasing energy) to produce electrons and protons, so unless the first law of thermodynamics doesn't apply here for some reason, I wouldn't expect the reverse reaction, your "neusion," to be exothermic. Am I missing something?
Title: Re: Why don't an atom's electrons fall into the nucleus and stick to the protons?
Post by: jccc on 11/07/2014 02:17:59
I don't have calculation. I am thinking energy is stored within atoms, not nucleus. To release all the energy, we must break down nucleus force field to destroy the atoms.

Nucleus reactions only released a fiction of the energy stored within atoms. Because by products are still atoms.

Experiment should not be too hard, if I have a lab I'll be very busy.
Title: Re: Why don't an atom's electrons fall into the nucleus and stick to the protons?
Post by: evan_au on 11/07/2014 12:36:24
Quote from: jccc
The nucleus force field is larger than atom radius, so that atoms can form matter.

The smallest atoms are Hydrogen (http://en.wikipedia.org/wiki/Hydrogen) (covalent radius around 30,000 femtometers) and Helium (http://en.wikipedia.org/wiki/Helium) (radius 28,000 femtometers).

The nucleus is primarily held together by the strong nuclear force (http://en.wikipedia.org/wiki/Strong_nuclear_force), which has a range of 1-3 femtometers. This is far less than the radius of even the smallest atom.

The distance which forms the chemical compounds in matter is determined by the quantum nature of electrons, not the nuclear force. (For gases, the Van Der Waals radius of a Hydrogen molecule is 120,000 femtometers.)

Jccc, please provide a hyperlink to the website which gave you the idea that the range of the nuclear force field is similar to the radius of an atom. (Note: "I just made it up" is not a good reference; experimental results are the best...)
Title: Re: Why don't an atom's electrons fall into the nucleus and stick to the protons?
Post by: chiralSPO on 11/07/2014 17:04:30
JCC, I think some of the confusion here has to do with trying to think classically about things on the atomic scale (and especially at the nuclear scale). Picturing protons and electrons as little charged balls that move like macroscopic balls will naturally lead to paradoxical and contradictory predictions.
Title: Re: Why don't an atom's electrons fall into the nucleus and stick to the protons?
Post by: evan_au on 11/07/2014 20:46:20
The experimental result which first showed the distribution of charge within the atom was conducted by Geiger & Marsden (http://en.wikipedia.org/wiki/Rutherford_experiment) in a 5-year period around 1910.

They fired alpha particles (positive charge) at a gold foil (electrically neutral). The results can be explained by classical replusion of electric charges. This experiment was not sensitive enough to measure the strong nuclear force, which occurs on much smaller scales.

Up to that time, there were a variety of models for charge within an atom; one of them (developed by J J Thomson) looks a bit like the Jccc model.

However, as a result of these experiments, an alternative model developed by Rutherford was accepted, where the positive charge is concentrated in a very small volume at the very center of the atom. (Rutherford was the director Geiger's lab).

The idea that "The nucleus force field is larger than atom radius" was discarded about 100 years ago, based on experimental evidence.

Science does need theories, but the most useful output of a theory is identifying a method to disprove that theory.

There have been many theories proposed, and anything you can think of has probably already been tried in some form by someone else - that's why they hand out Nobel prizes for genuinely new discoveries in certain fields. So, Jccc, it's useful to look at some of the history of science, so you don't resurrect theories which are already disproven. You are welcome to propose new theories, but identify them as a "maybe", not state them as a "fact"; do post them in the "New Theories" section, and try to suggest a way that will disprove your new theory.

That just leaves this thread with finding a simple explanation of the more complex quantum theory,  which was developed over the subsequent 40 years, partly to explain why the negative electrons don't collapse into the positive nucleus. I suggest you start your historical catch-up here: http://en.wikipedia.org/wiki/Quantum_mechanics
Title: Re: Why don't an atom's electrons fall into the nucleus and stick to the protons?
Post by: Bill S on 11/07/2014 22:47:45
 For some time I was puzzled by what seemed to be a reciprocity failure between electron jumps away from and towards the nucleus.  I reasoned that if an electron was attracted towards the nucleus, it should require more energy to move it away from the nucleus than towards it.  The conclusion I eventually reached was that as this was a quantum leap, the electron could not be said to be anywhere when it was not occupying an energy level.  If the electron could not be said to be anywhere, it could not be using or exchanging energy.  Thus it made no difference which way it was going.  The energy necessary to accomplish the jump would be the same.

I would appreciate comments on my reasoning, please.
Title: Re: Why don't an atom's electrons fall into the nucleus and stick to the protons?
Post by: jccc on 12/07/2014 01:39:00
The experimental result which first showed the distribution of charge within the atom was conducted by Geiger & Marsden (http://en.wikipedia.org/wiki/Rutherford_experiment) in a 5-year period around 1910.

They fired alpha particles (positive charge) at a gold foil (electrically neutral). The results can be explained by classical replusion of electric charges. This experiment was not sensitive enough to measure the strong nuclear force, which occurs on much smaller scales.

Up to that time, there were a variety of models for charge within an atom; one of them (developed by J J Thomson) looks a bit like the Jccc model.

However, as a result of these experiments, an alternative model developed by Rutherford was accepted, where the positive charge is concentrated in a very small volume at the very center of the atom. (Rutherford was the director Geiger's lab).

The idea that "The nucleus force field is larger than atom radius" was discarded about 100 years ago, based on experimental evidence.

Science does need theories, but the most useful output of a theory is identifying a method to disprove that theory.

There have been many theories proposed, and anything you can think of has probably already been tried in some form by someone else - that's why they hand out Nobel prizes for genuinely new discoveries in certain fields. So, Jccc, it's useful to look at some of the history of science, so you don't resurrect theories which are already disproven. You are welcome to propose new theories, but identify them as a "maybe", not state them as a "fact"; do post them in the "New Theories" section, and try to suggest a way that will disprove your new theory.

That just leaves this thread with finding a simple explanation of the more complex quantum theory,  which was developed over the subsequent 40 years, partly to explain why the negative electrons don't collapse into the positive nucleus. I suggest you start your historical catch-up here: http://en.wikipedia.org/wiki/Quantum_mechanics

Thanks for the comment and link! I read that page few times before, wish I understand the math. Also read this http://en.wikipedia.org/wiki/J._J._Thomson. I am far behind every one here, pretty depressed.  It's all your fault.
Title: Re: Why don't an atom's electrons fall into the nucleus and stick to the protons?
Post by: jccc on 12/07/2014 03:15:39
For some time I was puzzled by what seemed to be a reciprocity failure between electron jumps away from and towards the nucleus.  I reasoned that if an electron was attracted towards the nucleus, it should require more energy to move it away from the nucleus than towards it.  The conclusion I eventually reached was that as this was a quantum leap, the electron could not be said to be anywhere when it was not occupying an energy level.  If the electron could not be said to be anywhere, it could not be using or exchanging energy.  Thus it made no difference which way it was going.  The energy necessary to accomplish the jump would be the same.

I would appreciate comments on my reasoning, please.


This might help, not QM stuff.
It takes more force to push in than pull away.
Title: Re: Why don't an atom's electrons fall into the nucleus and stick to the protons?
Post by: jccc on 12/07/2014 05:05:48
Quote from: evan_au

The idea that "The nucleus force field is larger than atom radius" was discarded about 100 years ago, based on experimental evidence.

Please explain the evidence, thanks.

Quote from: evan_au

Science does need theories, but the most useful output of a theory is identifying a method to disprove that theory.

How about as I suggested, beam high speed/energy electron into water, measure the energy used and the heat produced, they should be always the same according to present knowledge. If experiments show extra energy?
Title: Re: Why don't an atom's electrons fall into the nucleus and stick to the protons?
Post by: jccc on 12/07/2014 15:02:06
2 H2o + 4e = o2 + E   

Oxygen atom nucleus has 8 protons, it attracts more energy into force field, the energy ball around it is denser than the energy ball around hydrogen atom nucleus.  Therefore, it takes higher energy electron to break down oxygen nucleus.

This give us control to only break down hydrogen atoms in the water. We don't want to break down oxygen atoms, it might produce new atoms and radiations.

Imagine a flash light type device which shoots high speed electron beams, point to a pond, dry it in minutes.

Point it to a watermelon, there is no more.

Do not let Pete have it.
Title: Re: Why don't an atom's electrons fall into the nucleus and stick to the protons?
Post by: chiralSPO on 12/07/2014 16:24:11
Quote from: jccc
The nucleus force field is larger than atom radius, so that atoms can form matter.

The smallest atoms are Hydrogen (http://en.wikipedia.org/wiki/Hydrogen) (covalent radius around 30,000 femtometers) and Helium (http://en.wikipedia.org/wiki/Helium) (radius 28,000 femtometers).

The nucleus is primarily held together by the strong nuclear force (http://en.wikipedia.org/wiki/Strong_nuclear_force), which has a range of 1-3 femtometers. This is far less than the radius of even the smallest atom.

The distance which forms the chemical compounds in matter is determined by the quantum nature of electrons, not the nuclear force. (For gases, the Van Der Waals radius of a Hydrogen molecule is 120,000 femtometers.)

Jccc, please provide a hyperlink to the website which gave you the idea that the range of the nuclear force field is similar to the radius of an atom. (Note: "I just made it up" is not a good reference; experimental results are the best...)
I remember we been discussing the possibility to pull the moon closer by charging it. We all agree that electrostatic force do work at long distance.

From F=kq1q2/r^2, also showed force between charged particles is always there even in long distance.

I said the nucleus force field is bigger than atom radius, means the positive charges within the nucleus produces a force field that is beyond atom radius. We can charge electron into neutral matter shows nucleus force attracts electron even out side the atom.

So what's wrong with this concept? The nucleus force field is larger than atom radius, so that atoms can form matter.

In your opinion, how big is nucleus force field? Or proton force field in a hydrogen atom?

It is widely accepted that the electrostatic force acts over long distances. I don't think anyone on this thread disputes that. When it was mentioned that the "nuclear force" exerts no significant effect at even reasonably small distances, what was meant was that the "strong force" which holds the positively charged nucleus together only acts over very, very short distances (femtometer scale)... This strong force is entirely different in nature and mechanism from the electrostatic force.

I am probably wrong here, but my intuition on this is that the strong force is related to an exchange energy of the quarks between the nucleons of the nucleus, so the effect decays exponentially with distance (e–k*r, where k is some constant; analogous to tunneling or exchanging electrons) in addition to the 1/r^2 law which arises from the fact that it spreads out in 3 dimensions. Any thoughts on this from the physicists out there? (this is just a chemist's hand-waving answer that allows me to sleep at night...)
Title: Re: Why don't an atom's electrons fall into the nucleus and stick to the protons?
Post by: jccc on 14/07/2014 00:28:24
2 H2o + 4e = o2 + E   

Oxygen atom nucleus has 8 protons, it attracts more energy into force field, the energy ball around it is denser than the energy ball around hydrogen atom nucleus.  Therefore, it takes higher energy electron to break down oxygen nucleus.

This give us control to only break down hydrogen atoms in the water. We don't want to break down oxygen atoms, it might produce new atoms and radiations.


Any thoughts on this?

I think electron is not as easy to accelerate/energize as proton, maybe beam protons into negative charged water, hoping some head on action to happen.

The net reaction we want is p + e = E. Assume all mass converted.

Never thought I will dream such a small dream, thanks for this palace!

Our future is not in the stars but in the stardusts.
Title: Re: Why don't an atom's electrons fall into the nucleus and stick to the protons?
Post by: jccc on 14/07/2014 02:32:57
2 H2o + 4e = o2 + E   

Oxygen atom nucleus has 8 protons, it attracts more energy into force field, the energy ball around it is denser than the energy ball around hydrogen atom nucleus.  Therefore, it takes higher energy electron to break down oxygen nucleus.

This give us control to only break down hydrogen atoms in the water. We don't want to break down oxygen atoms, it might produce new atoms and radiations.


Any thoughts on this?

I think electron is not as easy to accelerate/energize as proton, maybe beam protons into negative charged water, hoping some head on action to happen.

The net reaction we want is p + e = E. Assume all mass converted.

Never thought I will dream such a small dream, thanks for this palace!

Our future is not in the stars but in the stardusts.

This is my proof, try it.

Put e on top of p, what do you see/get? 
Title: Re: Why don't an atom's electrons fall into the nucleus and stick to the protons?
Post by: chiralSPO on 14/07/2014 03:43:48


The net reaction we want is p + e = E. Assume all mass converted.


Our future is not in the stars but in the stardusts.

Positrons and electrons will annihilate each other to release much energy. Protons and electrons will not convert. There are no positrons in water.
Title: Re: Why don't an atom's electrons fall into the nucleus and stick to the protons?
Post by: jccc on 14/07/2014 08:46:33

Protons and electrons will not convert.


Electrons and protons will not convert? Any proof?

Logically any opposite charges will interact.

The goal is to break down protons positive charge/force field to release stored energy.

It's like to break a bloom to release pressure air. Or cut a robber band to release bonding stuff.

The trigger can be other than electron, maybe laser beam or other particle.

This is just a theory to be tested, looks too good to be true for now.
Title: Re: Why don't an atom's electrons fall into the nucleus and stick to the protons?
Post by: evan_au on 14/07/2014 10:17:57
Quote
Electrons and protons will not convert [into Energy]? Any proof?

Proof: Electron+Proton=Hydrogen atom (or two of each will make a Hydrogen molecule, H2)

The Hydrogen atom 1H is stable, with a lifetime of at least tens of billions of years (ie much longer than Uranium - too long for us to measure in a laboratory).

So a proton & electron do not annihilate to release energy.
Quote
The goal is to break down protons positive charge/force field to release stored energy.
A Proton and an electron do interact (to form Hydrogen), and they do release energy, in the form of an ultraviolet photon from the Lyman Series (http://en.wikipedia.org/wiki/Lyman_series#The_Lyman_series).

The maximum energy released in this photon is 13.6 eV (electron Volts).

The energy remaining in the Hydrogen atom is around 940,000,000 eV.

So the interaction of a proton and an electron releases about 13.6/940,000,000 = 0.000001% of the energy in the interacting particles.

But you can't use this 13.6eV as a source of unlimited energy, as you must supply more energy than this to the equipment which rips the electron off the Hydrogen atom in the first place, and you can't convert ultraviolet photons into electricity with 100% efficiency.
Title: Re: Why don't an atom's electrons fall into the nucleus and stick to the protons?
Post by: Bill S on 14/07/2014 20:25:10
Quote from: jccc
... not QM stuff.

Interesting link, but it might be better to stick to the QM stuff to answer the OP's question.
Title: Re: Why don't an atom's electrons fall into the nucleus and stick to the protons?
Post by: jccc on 14/07/2014 20:36:28
Quote from: jccc
... not QM stuff.

Interesting link, but it might be better to stick to the QM stuff to answer the OP's question.

What's QM answer to what force canceled out the attraction force between nucleus and electrons so electrons able to levitate at atom radius?

Been asked few times, haven't seen any comment on that. Why?

Title: Re: Why don't an atom's electrons fall into the nucleus and stick to the protons?
Post by: chiralSPO on 14/07/2014 22:48:30
The electrons aren't levitating at an atom's radius, they're just that big. Consider them as waves rather than particles--the wave just can't fit in a space as small as the nucleus. For the simplest case, a single proton with a single electron in the lowest energy level (a 1s orbital), the density of the electron *is* greatest right near the proton, but there is significant electron density as far as ~1 Å from the proton. Try not to think of it as a tiny particle that is either close to the nucleus OR far from it, but a smooshed-out particle cloud that is close to AND far from the nucleus.

Does that help?
If visualizing this is difficult, look at the diagrams:
http://wps.prenhall.com/wps/media/objects/3081/3155040/blb0606/6.20.gif
http://0.tqn.com/d/np/einstein/176-1.png
or do a google image search for "electron density orbitals" or something along those lines (that's how I found these)
Title: Re: Why don't an atom's electrons fall into the nucleus and stick to the protons?
Post by: Bill S on 14/07/2014 23:33:40
Another way of visualising the scenario is to think of the electron as a standing wave and the orbital as a line on which the wave sits.  Only a fixed number of half wavelengths will fit on that line.  Thus the electron remains in that orbital unless/until it loses/gains just the right amount of energy to shed/gain a half wavelength, and thus jump either down or up to an orbital that will accommodate its new number of half wavelengths.

I know this is an oversimplification that doesn’t include the whole picture and that experts might shoot it down in flames, but it helped me.
Title: Re: Why don't an atom's electrons fall into the nucleus and stick to the protons?
Post by: jccc on 15/07/2014 02:26:05
You guys didn't answer the question at all.

The question is about force, what force balanced out 10^34g between proton and electron in hydrogen atoms?

But appreciate is the same, have a good one!

Title: Re: Why don't an atom's electrons fall into the nucleus and stick to the protons?
Post by: chiralSPO on 15/07/2014 03:28:00
If the electron is uniformly distributed around the nucleus, the force balances out. In this sense "orbit" is somewhat of an accurate analogy. What "force" keeps the moon away from the Earth?
Title: Re: Why don't an atom's electrons fall into the nucleus and stick to the protons?
Post by: jccc on 15/07/2014 03:51:47
If the electron is uniformly distributed around the nucleus, the force balances out. In this sense "orbit" is somewhat of an accurate analogy. What "force" keeps the moon away from the Earth?

A proton and an electron make a hydrogen atom. The two charges are separated by atom radius. I see the attraction force, where/what is the force to balance it?

The moon is circling the Earth, centrifugal force balanced out gravitation.

Use the same mechanism/principle, atoms will not work/form at all.

Atom structure is not similar like solar system at all, between electron and proton there are two forces at work, attraction and repelling. Much like bound by a spring.

Seems you have no clue about my ideas posted. Or you don't agree with. It's all good. 
Title: Re: Why don't an atom's electrons fall into the nucleus and stick to the protons?
Post by: jccc on 15/07/2014 04:03:29
Quote from: jccc
... not QM stuff.

Interesting link, but it might be better to stick to the QM stuff to answer the OP's question.

You really feel/think so? Why? Because of QM is mainstream or more logical?
Title: Re: Why don't an atom's electrons fall into the nucleus and stick to the protons?
Post by: chiralSPO on 15/07/2014 04:31:02
Imagine for a moment that a proton is a ball the size of a grain of sand, and the electron is a cloud the size of a zeppelin. They are both charged to the same magnitude but with opposite charge. The proton's charge is highly localized, the electron's is diffuse but somewhat "squishy." They will be attracted to each other, and eventually the two will have the exact same center. The electron cloud totally surrounds the nuclear grain and is so symmetrically distributed that every pull on one end of the cloud is exactly counteracted by the identical pull of the nucleus on the other end (both are pulled toward the nucleus, but the force is equal and opposite in Cartesian coordinates)

Any attempt to move the electron's center away from the proton's center will be opposed by the electrostatic attraction, but the two literally can't get any closer to each other, so they don't.

You may ask what is the "pressure" that keeps the electron inflated/keeps the cloud from collapsing down to a point the size of the proton. That has already been explained on this thread. Near the beginning. This is one of the many hideous faces of uncertainty. If the proton were somehow able to constrain the electron to a smaller volume, the electron's velocity would be more variable, and it would ultimately go fast enough to return to a larger area of occupancy.
Title: Re: Why don't an atom's electrons fall into the nucleus and stick to the protons?
Post by: jccc on 15/07/2014 04:42:01
Imagine for a moment that a proton is a ball the size of a grain of sand, and the electron is a cloud the size of a zeppelin. They are both charged to the same magnitude but with opposite charge. The proton's charge is highly localized, the electron's is diffuse but somewhat "squishy." They will be attracted to each other, and eventually the two will have the exact same center. The electron cloud totally surrounds the nuclear grain and is so symmetrically distributed that every pull on one end of the cloud is exactly counteracted by the identical pull of the nucleus on the other end (both are pulled toward the nucleus, but the force is equal and opposite in Cartesian coordinates)

Any attempt to move the electron's center away from the proton's center will be opposed by the electrostatic attraction, but the two literally can't get any closer to each other, so they don't.

You may ask what is the "pressure" that keeps the electron inflated/keeps the cloud from collapsing down to a point the size of the proton. That has already been explained on this thread. Near the beginning. This is one of the many hideous faces of uncertainty. If the proton were somehow able to constrain the electron to a smaller volume, the electron's velocity would be more variable, and it would ultimately go fast enough to return to a larger area of occupancy.

 A cloud the size of a zeppelin. Hit by a photon, jump out atom to become an electron?  Give me some of your lsd watered weed please.
Title: Re: Why don't an atom's electrons fall into the nucleus and stick to the protons?
Post by: chiralSPO on 15/07/2014 12:42:08
I am not suggesting that the electron cloud turns into a point-charge after interacting with a photon. I am also not claiming that it is easy to wrap one's mind around the first time through. However, I do think that the QM model of atoms, molecules and light is a very good one, and yes, it even becomes intuitive once you think about it enough. (lsd watered weed helps, of course)
Title: Re: Why don't an atom's electrons fall into the nucleus and stick to the protons?
Post by: jccc on 15/07/2014 19:14:34
I am not suggesting that the electron cloud turns into a point-charge after interacting with a photon. I am also not claiming that it is easy to wrap one's mind around the first time through. However, I do think that the QM model of atoms, molecules and light is a very good one, and yes, it even becomes intuitive once you think about it enough. (lsd watered weed helps, of course)

Finally you summed it up, good job!
Title: Re: Why don't an atom's electrons fall into the nucleus and stick to the protons?
Post by: jccc on 16/07/2014 00:24:24
If we  break proton in hydrogen atom into quarks, hydrogen atom will be no more. Left about 1% mass - electron and 3 quarks.

The missing mass should be converted to energy.

Agree?
Title: Re: Why don't an atom's electrons fall into the nucleus and stick to the protons?
Post by: UltimateTheory on 16/07/2014 01:22:59
If we  break proton in hydrogen atom into quarks, hydrogen atom will be no more. Left about 1% mass - electron and 3 quarks.

The missing mass should be converted to energy.

Agree?

This would break baryon number conservation.

Currently the only known way to extract energy stored in proton, is annihilation with antiproton.

f.e.
p+ + p- -> pion0 + pion0 + pion0 + kinetic energy spread across these particles.

Proton and antiproton have together 2 * 938.272 MeV = 1876.544 MeV
Pion0 rest mass is 135 MeV, so 3*135 MeV = 405 MeV
The rest is in kinetic energy of newly created particles.

pion0 is unstable particle and quickly decaying to

pion0 -> y + y
or
pion0 -> y + e+ + e-

Other sources mention up to 9 pions (also charged) (theoretically up to 13 mesons can be created)
So if we will have 2 pion+ and pion- and 5x pion0 it's 2*139 MeV + 5*135 MeV = 953 MeV energy in rest mass of newly created particles. The rest in kinetic energy.
Title: Re: Why don't an atom's electrons fall into the nucleus and stick to the protons?
Post by: jccc on 16/07/2014 02:03:07
If we  break proton in hydrogen atom into quarks, hydrogen atom will be no more. Left about 1% mass - electron and 3 quarks.

The missing mass should be converted to energy.

Agree?


Currently the only known way to extract energy stored in proton, is annihilation with antiproton.

f.e.
p+ + p- -> pion0 + pion0 + pion0 + kinetic energy spread across these particles.

Proton and antiproton have together 2 * 938.272 MeV = 1876.544 MeV
Pion0 rest mass is 135 MeV, so 3*135 MeV = 405 MeV
The rest is in kinetic energy of newly created particles.


Find a better way, antiproton is too pricy.
Title: Re: Why don't an atom's electrons fall into the nucleus and stick to the protons?
Post by: jccc on 16/07/2014 02:38:52
Electron carries 1 negative charge, proton carries 1 positive charge.

Why proton carries 939 Mev but electron only carries so little energy?

Is that suggesting that energy is negatively charged? Stored within protons force field?

All the energy released since big bang filled space and connected it into an EM field.

All things in space/universe is connected by this EM field.

When a charged particle accelerates, its force field produce a pressure wave into surrounding EM field, travel away at light speed.

So far, do you agree the logic?




 
Title: Re: Why don't an atom's electrons fall into the nucleus and stick to the protons?
Post by: UltimateTheory on 16/07/2014 03:27:29
Electron carries 1 negative charge, proton carries 1 positive charge.

Why proton carries 939 Mev but electron only carries so little energy?

938.272 MeV proton
0.5109989 MeV electron

Is that suggesting that energy is negatively charged? Stored within protons force field?

There is no such thing as negative energy..

Antiparticle has positive energy. Dirac was mistaken. He plug Einstein equation E=m*c^2 to Planck E=h*f, and though that antielectron will have negative frequency, negative mass, etc.
But the all people who can calculate decay energy and decay modes of unstable isotopes will tell you it's wrong. Positron (antielectron) has positive energy.

All the energy released since big bang filled space and connected it into an EM field.

All things in space/universe is connected by this EM field.

When a charged particle accelerates, its force field produce a pressure wave into surrounding EM field, travel away at light speed.

So far, do you agree the logic?

No.

Pressure wave? In front of particle? There is no experiment confirming such behavior.

Charged particle is emitting photons when it's changing direction or is decelerated, or intercepted by other charged particle (like f.e. electron intercepted by proton).

f.e. if I will use Cockcroft-Walton generator to produce 10 kV, and plug electrodes to vacuum tube, electrons kinetic energy inside of tube will be 10 keV,
if such highly accelerated electron will collide with something between electrodes like piece of metal, there will be emitted photon up to 10 keV (x-ray)
and electron will be slowed down, decelerated.
X-ray will fly through tube, then through walls, then will ionize air or other material behind it, leaving trace in Cloud Chamber.
You can see it on your own eyes, if you will build Cloud Chamber and Cockcroft-Walton generator for less than $100
http://www.ultimate-theory.com/en/2014/6/8/how-to-build-cloud-chamber-particle-detector
Title: Re: Why don't an atom's electrons fall into the nucleus and stick to the protons?
Post by: jccc on 16/07/2014 04:06:08
Electron carries 1 negative charge, proton carries 1 positive charge.

Why proton carries 939 Mev but electron only carries so little energy?

938.272 MeV proton
0.5109989 MeV electron

Is that suggesting that energy is negatively charged? Stored within protons force field?

There is no such thing as negative energy..

Antiparticle has positive energy. Dirac was mistaken. He plug Einstein equation E=m*c^2 to Planck E=h*f, and though that antielectron will have negative frequency, negative mass, etc.
But the all people who can calculate decay energy and decay modes of unstable isotopes will tell you it's wrong. Positron (antielectron) has positive energy.

All the energy released since big bang filled space and connected it into an EM field.

All things in space/universe is connected by this EM field.

When a charged particle accelerates, its force field produce a pressure wave into surrounding EM field, travel away at light speed.

So far, do you agree the logic?

No.

Pressure wave? In front of particle? There is no experiment confirming such behavior.

Charged particle is emitting photons when it's changing direction or is decelerated, or intercepted by other charged particle (like f.e. electron intercepted by proton).

f.e. if I will use Cockcroft-Walton generator to produce 10 kV, and plug electrodes to vacuum tube, electrons kinetic energy inside of tube will be 10 keV,
if such highly accelerated electron will collide with something between electrodes like piece of metal, there will be emitted photon up to 10 keV (x-ray)
and electron will be slowed down, decelerated.
X-ray will fly through tube, then through walls, then will ionize air or other material behind it, leaving trace in Cloud Chamber.
You can see it on your own eyes, if you will build Cloud Chamber and Cockcroft-Walton generator for less than $100
http://www.ultimate-theory.com/en/2014/6/8/how-to-build-cloud-chamber-particle-detector

Awesome! Thanks!

Please comment on the following thoughts.

The strongest positive force field in nature is very next to a proton.

The strongest negative force field is also very next to a proton, composed by negative charged energy/enertron.

Energy is thought to be infinity small negative charged particles each carries a fiction of an electron's charge.

In nature, proton attracts all negative charges, electrons and enertrons compete accelerating to proton. Enertrons win the race, it has more charge to volume ratio. 

Enertrons form a ball around proton, density = 1/r^3 from proton. Electron levitate at atom radius where protons attracting force equals to enertron ball repulsion force.

Title: Re: Why don't an atom's electrons fall into the nucleus and stick to the protons?
Post by: UltimateTheory on 16/07/2014 04:45:34
Electron you can see in Cloud Chamber as trace made by particle, especially when it has large kinetic energy (f.e. electron from beta decay-).

Unlike your particle.
Title: Re: Why don't an atom's electrons fall into the nucleus and stick to the protons?
Post by: jeffreyH on 17/07/2014 22:21:42
If you were to have a pole mounted in the centre of a pool and were to generate a wave around the periphery that moved toward the centre it would be interesting to see what the effect would be. Water is a good medium for modeling wave-like phenomena.
Title: Re: Why don't an atom's electrons fall into the nucleus and stick to the protons?
Post by: jeffreyH on 17/07/2014 22:33:24
This site has some interesting representations of what matter waves may look like.

http://www.cyclesresearchinstitute.org/cycles-wave-structure/wave-structure-la-freniere.shtml
Title: Re: Why don't an atom's electrons fall into the nucleus and stick to the protons?
Post by: JP on 18/07/2014 04:02:54
This site has some interesting representations of what matter waves may look like.

http://www.cyclesresearchinstitute.org/cycles-wave-structure/wave-structure-la-freniere.shtml

Unfortunately, those plots are nonsense.
Title: Re: Why don't an atom's electrons fall into the nucleus and stick to the protons?
Post by: jeffreyH on 19/07/2014 19:50:51
This site has some interesting representations of what matter waves may look like.

http://www.cyclesresearchinstitute.org/cycles-wave-structure/wave-structure-la-freniere.shtml

Unfortunately, those plots are nonsense.

I didn't say they were valid. What interests me are inward traveling waves and what effects they would produce. Throughout the universe we have converging gravitational waves. This could be represented by a perfect sphere with a perfect spherical cavity at its centre.
Title: Re: Why don't an atom's electrons fall into the nucleus and stick to the protons?
Post by: PmbPhy on 19/07/2014 22:00:28
Quote from: jccc
Please comment on the following thoughts.

The strongest positive force field in nature is very next to a proton.

The strongest negative force field is also very next to a proton, composed by negative charged energy/enertron.

Energy is thought to be infinity small negative charged particles each carries a fiction of an electron's charge.

In nature, proton attracts all negative charges, electrons and enertrons compete accelerating to proton. Enertrons win the race, it has more charge to volume ratio. 

Enertrons form a ball around proton, density = 1/r^3 from proton. Electron levitate at atom radius where protons attracting force equals to enertron ball repulsion force.
You already have my thoughts on some of this. I.e. first off you're talking about something that doesn't exist so it's either imaginary, like a unicorn, or it belongs to a New Theory. If it's the later then this is the wrong forum. If indeed it is a new theory and this is a new particle from that theory then you could at least tell us where this notion came from. I.e. what made you start talking about a thing called an "enertron" and then refuse to tell us what it is every time I've asked you what it is? It's hard to help you when you do things like that. And you know me my friend. I do want to help you. :)

I'll go as far as I can with what I have; you wrote
Quote from: jccc
Please comment on the following thoughts.

The strongest positive force field in nature is very next to a proton.
What is the source of this force? Are you talking about the electric force? If so then that's not the strongest force in nature. The strong force is much stronger. Also since you said that it's the strongest positive force then the sign being positive means that its repulsive. However the strong force is attractive for all nucleons, both protons and neutrons. So you have a contradiction here.

Quote from: jccc
The strongest negative force field is also very next to a proton, composed by negative charged energy/enertron.
Here's where you've lost connection with mainstream science. You're asking about things that don't exist and expecting physics to give you an appropriate and correct answer. Why are you doing this my friend?
Title: Re: Why don't an atom's electrons fall into the nucleus and stick to the protons?
Post by: evan_au on 21/07/2014 10:29:14
Quote
inside nucleus, gravity plays a big deal due to f=m1m2/r^2

Newton's theory of gravity (http://en.wikipedia.org/wiki/Newton%27s_law_of_universal_gravitation) states that f=Gm1m2/r2

The missing term is G, which is approximately equal to 6.7×10−11 N m2 kg−2

So this comparison between gravity and the electrical field is off by about 11 orders of magnitude.
Title: Re: Why don't an atom's electrons fall into the nucleus and stick to the protons?
Post by: chiralSPO on 21/07/2014 14:34:49
What's the strongest positive force field? A proton carries 1 positive charge, you put a test charge neat it to measure it. The force f=1/r^2, isn't the strongest positive force field is near the surface of a proton?

Strong force holds protons and neutrons together, therefore it should be negative charged in nature. Agree?

Also, inside nucleus, gravity plays a big deal due to f=m1m2/r^2, compare the radius of the atom and the nucleus.

There is no charge associated with the strong force. Just as with gravity. Charge is irrelevant to these forces. Just as mass is irrelevant to the electrostatic force.

The gravitational field of a hydrogen or helium nucleus is pretty close to nothing as far as their electrons are concerned. However, for much more massive atoms, (where the electrons are also much closer to the nucleus due to electrostatic attraction), like gold or mercury there is actually a significant effect. But it's not the gravitational attraction (electrons are really light), it is from the time dilation near the nucleus.
Title: Re: Why don't an atom's electrons fall into the nucleus and stick to the protons?
Post by: PmbPhy on 21/07/2014 15:03:01
Quote from: jccc
What's the strongest positive force field?
[/quoplte A proton carries 1 positive charge, you put a test charge neat it to measure it. The force f=1/r^2, isn't the strongest positive force field is near the surface of a proton?
jccc - In my last post asked you several questions. I asked you those questions because I need to know the answer in order to answer your question. In this case I need to know what force you're talking about. When someone uses the term "positive force" they're often referring to a force which is directed away from the source of the force. In this context it means a repulsive force. The strongest repulsive force is the electric force. The closer to the particle the stronger the force.

With those assumptions we can say that the force is the same near any positive charge because all positive charges are the same. The only exception is the quark but they don't appear by themselves outside the nucleon.

However if the particle is an alpha particle (which has two protons in it) then the force is stronger near the surface of the alpha particle. And the force is the same near any positively charged particle. So using the proton is misleading because any positive charge will have the same force pushing it away. And it keeps going up with the number of protons near the nucleus of the atom, so long as electrons aren't screening them.

Quote from: jccc
Strong force holds protons and neutrons together, therefore it should be negative charged in nature. Agree?
No.
In theory the strong force also holds neutrons to neutrons. Just because it holds two charged particles together it doesn't mean that it's doing so by canceling out the positive charge.

Quote from: jccc
Also, inside nucleus, gravity plays a big deal due to f=m1m2/r^2, compare the radius of the atom and the nucleus.
The gravitational force inside the nucleus is so small that it's ignored in nuclear physics.
Title: Re: Why don't an atom's electrons fall into the nucleus and stick to the protons?
Post by: JP on 21/07/2014 16:22:02
Don't put out books and numbers, if you cannot explain just say it, say we don't know yet is better answer.

On the contrary, its been explained numerous times.  If you can't understand the explanations, just say it and we'll try to explain it in a different way.
Title: Re: Why don't an atom's electrons fall into the nucleus and stick to the protons?
Post by: jccc on 21/07/2014 16:28:58
Don't put out books and numbers, if you cannot explain just say it, say we don't know yet is better answer.

On the contrary, its been explained numerous times.  If you can't understand the explanations, just say it and we'll try to explain it in a different way.

Yes, please explain it in a way that is logically sound. This is so important! THANKS!
Title: Re: Why don't an atom's electrons fall into the nucleus and stick to the protons?
Post by: PmbPhy on 21/07/2014 17:17:59
Quote from: jccc
You can say anything you like, but the logic just don't sound. Why is electrons in gold atoms very close to nucleus   not contact with it? Isn't strong attraction force at work?
In order to understand that you need to study quantum mechanics. It's not possible to answer any and all questions about nature to someone who doesn't understand physics. Likewise it's not possible to explain what happens in an atom to someone who has never studied quantum mechanics. The best that can be said to someone who's never studied it is to say that in the microscopic world electrons don't behave like ordinary things in our macroscopic world. I.e. they can't be described as having a particular shape of being at a particular position. What we can also say is that electrons around an atom are in a way somewhat like standing waves and those standing waves can only be so close to the nucleus.

Quote from: jccc
7 pages, no one has an answer to the Op question that is sounding.
Actually it has been explained to the satisfaction of the person who asked the question. Just because you didn't understand it it doesn't mean the description was wrong. And as I said, you need a certain background to be able to grasp it. People don't just walk into a quantum mechanics class, ask a question and can expect to walk out of the class understanding the answer.

Quote from: jccc
What's the mechanism?
We talked about the fact that science is not about providing mechanisms. Did you not listen. Read carefully  http://math.ucr.edu/home/baez/crackpot.html
Quote
10 points for arguing that while a current well-established theory predicts phenomena correctly, it doesn't explain "why" they occur, or fails to provide a "mechanism".
:)
Title: Re: Why don't an atom's electrons fall into the nucleus and stick to the protons?
Post by: jccc on 21/07/2014 17:51:06
Electrons move in straight line in cloud chamber. Bent by EM field. I don't see standing wave, cloud or shell. I see a charged particle.

Why in atoms electron becomes wave? Is the wave negative charged? Is it attracted by nucleus? How the wave moves?

In H2O, electrons are bounded in fixed position, how is it a wave?
Title: Re: Why don't an atom's electrons fall into the nucleus and stick to the protons?
Post by: lightarrow on 21/07/2014 18:02:32
Don't put out books and numbers, if you cannot explain just say it, say we don't know yet is better answer.

On the contrary, its been explained numerous times.  If you can't understand the explanations, just say it and we'll try to explain it in a different way.

Yes, please explain it in a way that is logically sound. This is so important! THANKS!
NO! It's not important at all! Physics *don't have* to be "logically sound", and not even "logical". It have to be consistent with its postulates/theorems/definitions/rules and with the experimental results.

--
lightarrow
Title: Re: Why don't an atom's electrons fall into the nucleus and stick to the protons?
Post by: jccc on 21/07/2014 23:18:20
Electrons move in straight line in cloud chamber. Bent by EM field. I don't see standing wave, cloud or shell. I see a charged particle.

Why in atoms electron becomes wave? Is the wave negative charged? Is it attracted by nucleus? How the wave moves?

In H2O, electrons are bounded in fixed position, how is it a wave?
No comment? Or as confused as me?
Title: Re: Why don't an atom's electrons fall into the nucleus and stick to the protons?
Post by: lightarrow on 22/07/2014 00:58:42
NO! It's not important at all! Physics *don't have* to be "logically sound", and not even "logical". It have to be consistent with its postulates/theorems/definitions/rules and with the experimental results.
Please explain the theorems and experiment results about how a hydrogen atom is formed.
You received a lot of answers but you don't believe us...
You only have to study quantum mechanics and when you have seriously done it you come here again and you tell us what's wrong with this theory.
Regards.

--
lightarrow
Title: Re: Why don't an atom's electrons fall into the nucleus and stick to the protons?
Post by: PmbPhy on 22/07/2014 05:09:12
Quote from: jccc
Electrons move in straight line in cloud chamber.
Actually that's not true. What you're seeing is an approximation of a straight line. At the subatomic level it can't even be said to be a trajectory in the literal sense.

Quote from: jccc
I don't see standing wave, cloud or shell.
Please recall what I wrote, i.e.

Quote from: jccc
The best that can be said to someone who's never studied it is to say that in the microscopic world electrons don't behave like ordinary things in our macroscopic world.
Note: When I use the term microscopic level, I'm being a bit sloppy. To be precise it really applies to the subatomic level.

Notice that I was talking about the subatomic world while you keep thinking about the macroscopic world. Did you ever really believe that if you looked hard or with a strong enough microscope that you'd be able to see electrons orbiting a nucleus? It's down at atomic sizes that this wave size for electrons is apparent. Thompson's double slit experiment used photons to demonstrate the wave nature of photons. So you really can't compare the two when it comes to wavelength.

Quote from: jccc
I see a charged particle.
That too is incorrect. Have you ever studied how a cloud chamber works? If not then please see
http://en.wikipedia.org/wiki/Cloud_chamber#Structure_and_operation

Quote from: jccc
The result is a supersaturated environment. The alcohol vapour condenses around ion trails left behind by the travelling ionizing particles. The result is cloud formation, seen in the cloud chamber by the presence of droplets falling down to the condenser. As particles pass through they leave ionization trails and because the alcohol vapour is supersaturated it condenses onto these trails. Since the tracks are emitted radially out from the source, their point of origin can easily be determined
When you look at "trajectories" in a cloud chamber and you think you see a moving charge, what you're really seeing is a trail of vapour. And this is at the macroscopic level, not the microscopic level.

You can't see things at the subatomic level with your eyes or even with the most powerful microscope that can theoretically be built.

Quote from: jccc
Why in atoms electron becomes wave? Is the wave negative charged? Is it attracted by nucleus? How the wave moves?
Electrons never become waves. They have wave "properties" at some times and particle properties at other times.

Quote from: jccc
In H2O, electrons are bounded in fixed position, how is it a wave?
Again, you didn't read close enough to my last post. I.e.

Quote from: jccc
What we can also say is that electrons around an atom are in a way somewhat like standing waves and those standing waves can only be so close to the nucleus.
Why do you think I added that part that says  in a way somewhat like? It's because it's too difficult to explain the exact nature to someone who hasn't studied the subject. If I told you about eigenfunctions and spherical harmonics would you be able to understand what I was talking about? The functions which describe the probability density of the electron a solution to Schrodinger's equation.

To see what the solutions look like for hydrogen go to Google and search using the phrase "diagrams of the solutions to Schrodinger equation for hydrogen". You'll see that the first result comes up with pictures that look like clouds (which is why they're called "electron clouds"). Take a look at it and study it.
Title: Re: Why don't an atom's electrons fall into the nucleus and stick to the protons?
Post by: jccc on 23/07/2014 17:18:00
It is raining, I see everything without a leg/support falling down, a little gravity but nothing can escape from it.

What's the leg support 10^34 g attraction force between nucleus and electrons?

QM laws? Seriously my friends.
Title: Re: Why don't an atom's electrons fall into the nucleus and stick to the protons?
Post by: chiralSPO on 23/07/2014 19:59:23
your "logic" fails on the subatomic scale. Many of your assumptions are based on macroscopic phenomena, and lead to false and paradoxical statements.

On this subatomic scale, it is entirely reasonable, and actually necessary, for something to "be in multiple places at once"
On this subatomic scale, it is entirely impossible for something to stop moving
On this subatomic scale, it is entirely possible for something to go from one place to another without traveling through any points in between.

On a macroscopic scale, these claims sound ridiculous, and anything behaving in this way would certainly seem magical. However this is how really small things are. It has been demonstrated experimentally, we have mathematical models for it, and these models make good predictions that can be tested against experiment (many of them).

Please stop telling the world that it is not behaving the way you think is logical (it won't listen)
Title: Re: Why don't an atom's electrons fall into the nucleus and stick to the protons?
Post by: JP on 23/07/2014 23:30:20
I've split a few posts off this thread.  jccc, please keep your posts on topic and don't promote new theories here.  If you want to propose alternative explanations to standard quantum mechanics, the place to do so is in the New Theories forum.

Thanks,
The Mods
Title: Re: Why don't an atom's electrons fall into the nucleus and stick to the protons?
Post by: Ethos_ on 24/07/2014 04:26:08

On a macroscopic scale, these claims sound ridiculous, and anything behaving in this way would certainly seem magical. However this is how really small things are. It has been demonstrated experimentally, we have mathematical models for it, and these models make good predictions that can be tested against experiment (many of them).



What experiments and predictions? Please point out, appreciate.

Believe me, if I can understand QM, my sleep will be much sweeter.
To understand QM, I'm sure if you were to take the advice which several members here have offered, you might not only understand QM, you would sleep much sweeter as well as the rest of us.
Title: Re: Why don't an atom's electrons fall into the nucleus and stick to the protons?
Post by: jccc on 24/07/2014 05:20:46
My great great great grandpa was a scientist. He left a note book, he theorised/ predicted the universe is expending, light speed is constant in all sources and directions, spiral galaxies create magnetic field at center disk.

If he was published his theories/predictions in mainstream science field 100 years ago, would you never forget his name?
Title: Re: Why don't an atom's electrons fall into the nucleus and stick to the protons?
Post by: JSS on 24/07/2014 22:46:02
The reason that an electron cannot meet the proton is simply one of "impedance matching". A small mass charge cannot enter the domain of a large mass charge, even of the opposite charge. Note that as long as the masses of the particles are nearly equal, they do collide, because they are more impedance matched. The truly strongest force in nature is impedance mismatching (most notably between a large and small mass with opposite charges.

I could go into extreme detail of exactly why and how that works and what mass has to do with impedance matching, but it is a very, very long story.

And interestingly to space travel buffs, if you have a very large, heavy space ship with a very strong positive charge on it, then fire a small missile with an equally strong negative charge on it at the ship, the missile will blow up before reaching the ship. The impedance mismatch is a harder surface than the ship.

That is how you get the "impenetrable force shield" depicted in sci-fi films. The trick is getting the strong charge on the missile.  [8D]
Title: Re: Why don't an atom's electrons fall into the nucleus and stick to the protons?
Post by: JSS on 24/07/2014 23:34:21
So, electrons are for real orbiting the nucleus?

Charged particle accelerating in EM field will release energy, how long can electron keep orbiting before crash into nucleus?
The electron is constantly maintaining its size by absorbing energy (in the form of infinitesimal EMR pulses). All particles are merely a concentrated clump of infinitesimal EMR noise, constantly replenishing. Thus an electron can orbit (merely shifting the center of its noise), release energy, and also absorb an equal amount of energy.

What we call "empty space" is actually filled with infinitesimal EMR pulses (much as Krauss theorized, although having nothing to do with quantum foam). Without such subtle affectance, no particle could ever exist.
Title: Re: Why don't an atom's electrons fall into the nucleus and stick to the protons?
Post by: Ethos_ on 24/07/2014 23:39:01



What we call "empty space" is actually filled with infinitesimal EMR pulses (much as Krauss theorized, although having nothing to do with quantum foam). Without such subtle affectance, no particle could ever exist.
Precisely................There is no "space/time" empty of field.
Title: Re: Why don't an atom's electrons fall into the nucleus and stick to the protons?
Post by: evan_au on 25/07/2014 10:55:59
Quote
So, electrons are for real orbiting the nucleus?

Not really - it's an idea dating from the early days of studying the internals of the atom, where they imagined the atom like a tiny solar system.

This is still sometimes used as an analogy to introduce students to the structure of the atom, but the wave nature of planets is too small to be noticeable, so it's fairly limited as an analogy.

Quote
Charged particle accelerating in EM field will release energy, how long can electron keep orbiting before crash into nucleus?
About 16 picoseconds.

Clearly, the atoms in our bodies survive longer than this, so there is a paradox here.

This exact problem was identified by Niels Bohr, who proposed a solution in 1913 (101 years ago, now).

To catch up, see: http://en.wikipedia.org/wiki/Bohr_model#Origin

Later developments in quantum theory from 1925 onwards made these ideas more precise (but often more shrouded in mathematics).
Title: Re: Why don't an atom's electrons fall into the nucleus and stick to the protons?
Post by: jeffreyH on 25/07/2014 18:12:02
I don't mean to be contrary. [:)] I just need to explore every possibility that might offer experimental evidence that my vision of a photon is not reality. As far as I can determine the double slit experiment supports the vision. If I did not have the photon defined so that it must produce the observed results by cause and effect, I might fantasize some magical wave-particle duality.

The anatomy of a photon: A photon consists of two half cycles of electric and magnetic fields that drive points of maxima through space. The fields exist in a spatial area around the points. The changing amplitude of the fields drive the points and determine their path through space. Photon interaction happens at the points of maxima. So any observation will see the points. Edit: It is not my definition; it is Maxwell's definition.

What perplexes me is that folks don't seem to understand that. Is it that I just can't put the right words together?

Here's a schematic of the vision. It looks just like those that were in the text books when I studied electronics and nuclear instrumentation back in the 50's.

(https://www.thenakedscientists.com/forum/proxy.php?request=http%3A%2F%2Fphotontheory.com%2Fphoton-01.jpg&hash=fdb8df20e4b0531ffc4c59573374c8c0)

I know exactly what you mean.  [8D] I am putting together a model the details of which are in new theories. The Pauli Exclusion Principle has a physical mechanism that can be described along with the difference in energies of electrons. I like your diagram BTW. Although you haven't described the path of the waveform correctly. Yes there are waves!
Title: Re: Why don't an atom's electrons fall into the nucleus and stick to the protons?
Post by: PmbPhy on 25/07/2014 20:06:58
Quote from: jccc
Bingo!

This kind of comment is just cool as it can be.
What? That's what I've been telling you all this time!
Title: Re: Why don't an atom's electrons fall into the nucleus and stick to the protons?
Post by: jccc on 25/07/2014 21:17:25
Quote from: jccc
Bingo!

This kind of comment is just cool as it can be.
What? That's what I've been telling you all this time!

Feel like his wording has more momentum. Yours more temperature.
Title: Re: Why don't an atom's electrons fall into the nucleus and stick to the protons?
Post by: jccc on 26/07/2014 04:03:14
In fusion reaction, two atoms impact so hard, even two nucleus crashed into one, how come electron escaped from the crash?

Let's say somehow QM is correct, atoms are stable due to some mystery laws. When we putting pressure on matter, to a point, atoms/orbital/shell/wave will crash. But this never happened.

Wonder, confuse, another sleepless night, have a nice weekend!
Title: Re: Why don't an atom's electrons fall into the nucleus and stick to the protons?
Post by: PmbPhy on 26/07/2014 04:13:34
Quote from: jccc
In fusion reaction, two atoms impact so hard, even two nucleus crashed into one, how come electron escaped from the crash?
The atoms become ionized during fusion and loose electrons due to such an energetic impact knocking it out, the atom would later capture another electron and have a full set of electrons again.

Quote from: jccc
Let's say somehow QM is correct, atoms are stable due to some mystery laws.
Those "mystery laws" are called the postulates of quantum mechanics.

Quote from: jccc
When we putting pressure on matter, to a point, atoms/orbital/shell/wave will crash. But this never happened.
Since when? Atoms due combine to form molecules you know.
Title: Re: Why don't an atom's electrons fall into the nucleus and stick to the protons?
Post by: PmbPhy on 26/07/2014 05:47:01
Quote from: jccc
Pete, I don't get the part two positive nucleus able to impact into one but opposite charges not able to impact into one.
I don't know what you mean by "impact." Please explain.

The proton and electron are opposite charges and they form a hydrogen atom. The electron and positron are opposite charges and form positronium.  See http://en.wikipedia.org/wiki/Positronium

Two positive charges also have the strong force acting on them in order to bind them together. Without the strong force nuclei with more the one proton couldn't exist.

Quote from: jccc
You talk about electron capture, capture by attraction force right? Why is nucleus only capture electrons half way and put them at radius?
What do you mean by "put them at radius"? Electron capture is the following process

cc4eadd7c2cf0fbaa6cddb455a2ab5e0.gif

where n is a neutron and 66698fb025b35665217b25df186b0e93.gif is an electron neutrino.

Quote from: jccc
When we put matter into liquid nitrogen, atom's orbital/shell/wave should crash like a glass doom under high pressure, that never happened as I know.
Now we're getting into quantum chemistry. A field I haven't thought about since I was a sophomore in college so many decades ago. Also you're using terms which I don't know what they mean such as crash like a glass doom.
Title: Re: Why don't an atom's electrons fall into the nucleus and stick to the protons?
Post by: lightarrow on 26/07/2014 12:20:58
When we put matter into liquid nitrogen, atom's orbital/shell/wave should crash like a glass doom under high pressure,
Forget it. The interaction between proton and electron in an atom has nothing to do with thermal excitement.
Anyone made you believe it, has studied physics on comics...

--
lightarrow
Title: Re: Why don't an atom's electrons fall into the nucleus and stick to the protons?
Post by: jccc on 26/07/2014 15:03:08
When we put matter into liquid nitrogen, atom's orbital/shell/wave should crash like a glass doom under high pressure,
Forget it. The interaction between proton and electron in an atom has nothing to do with thermal excitement.
Anyone made you believe it, has studied physics on comics...

--
lightarrow

The strong attraction force fade away under QM laws seems more comic to me and lot others.

So far we don't even sure if the electron is orbiting, is that a fact?

Title: Re: Why don't an atom's electrons fall into the nucleus and stick to the protons?
Post by: jccc on 26/07/2014 17:11:17
At 0 degree k, all particles stop motion, certainly there would be no orbiting or standing wave happening, how could  two opposite charges in atom keep a distance?

Strong force bond positive and negative quarks together, why not bond electron into nucleus?

Thinking about it often times got me very emotional, why why why? Pull my hairs off, pull more. I hate science.
Title: Re: Why don't an atom's electrons fall into the nucleus and stick to the protons?
Post by: PmbPhy on 26/07/2014 18:13:28
Quote from: jccc
The strong attraction force fade away under QM laws seems more comic to me and lot others.
What is "strong attraction force"? And what do you mean "fade away under QM laws"? If they are what I think they are then any comic appearance is merely ignorance of the subject matter.

Quote from: jccc
So far we don't even sure if the electron is orbiting, is that a fact?
No. That's not a fact. We are very sure about what's going on. Electrons in atomic orbitals have a very specific meaning in quantum mechanics. "orbit" is just doesn't have the same meaning I QM than it does in classical mechanics. I've already explained many times what it means so I won't repeat myself.

Quote from: jccc
At 0 degree k, all particles stop motion,...
That's not true. First off it's not phrased as "0 degrees K". That's a common misunderstanding. It's phrased as "0 K". There's no "degrees" when it comes to Kelvin. 0 K only means that all atomic and molecular motion ceases. It doesn't refer to subatomic particles like electrons.

Quote from: jccc
... certainly there would be no orbiting or standing wave happening, how could  two opposite charges in atom keep a distance?
This is a mistake based on your misunderstanding of what 0 K means.

Quote from: jccc
Strong force bond positive and negative quarks together, why not bond electron into nucleus?
Because the strong force only acts between nucleons. That's because gluons mediate the strong force and gluons only mediate the strong force between quarks and not between electrons and nucleons/a nucleus.

I wish you'd consider a serious study of physics rather then being all over the place haphazardly. You're going to get frustrated like this as I see you getting now. If you learn it like us physicists do then you'd learn a lot more.
Title: Re: Why don't an atom's electrons fall into the nucleus and stick to the protons?
Post by: jccc on 26/07/2014 19:57:35
The explanation for why an electron does not fall into the nucleus comes from a fundamental concept in quantum mechanics: the Heisenberg uncertainty principle. Put simply, it states that you cannot know the position and momentum of a particle simultaneously. More rigorously stated, the product of the uncertainty of the position of a particle (Δx) and the uncertainty of its momentum (Δp) must be greater than a specified value:

ΔxΔp≥ℏ2


Now, as the electron approaches the nucleus, it's uncertainty in position decreases (if the electron is 10nm away from the nucleus, it could be anywhere within a spherical shell of radius 10nm, but if the electron is only 0.1nm away from the nucleus, that area is greatly reduced). According to the Heisenberg uncertainty principle, if you decrease the uncertainty of the electrons position, the uncertainty in its momentum must increase. This increased momentum uncertainty means that the electron will be moving away from the nucleus faster, on average.

Put another way, if we do know that at one instant, that the electron is right on top of the nucleus, we lose all information about where the electron will be at the next instant. It could stay at the nucleus, it could be slightly to the left or to the right, or it could very likely be very far away from the nucleus. Therefore, because of the the uncertainty principle it is impossible for the electron to fall into the nucleus and stay in the nucleus.

In essence, the uncertainty principle causes a sort of quantum repulsion, that keeps electrons from being too tightly localized near the nucleus.

Quantum repulsion? My head is exploding.
Title: Re: Why don't an atom's electrons fall into the nucleus and stick to the protons?
Post by: jccc on 26/07/2014 23:00:16
Why is moving object slow down and stop? Because it get's tired.

Why don't an atom's electrons fall into the nucleus and stick to the protons? Because there is quantum repulsion at work.

See the similarity?

In quantum world, black is white, white is black. Don't ask me why, it just be that way.
Title: Re: Why don't an atom's electrons fall into the nucleus and stick to the protons?
Post by: PmbPhy on 26/07/2014 23:37:04
Joey,

Before I respond I want to ask you a question. Your response will determine if and how I help you again. I only say "if" because I ask these questions of you so that I'm able to properly respond to your many questions. Okay?

Question: Why don't you ever answer any of the many questions that I ask you? I only ask them so I can better respond to the questions you ask me. So when you don't respond you're only hurting yourself.

Quote from: jccc
Is there any attraction force stronger than proton and electron at atom radius distance?
No.

Quote from: jccc
Yes, when the distance decreased.
That's incorrect because in your question you wrote at atom radius distance which means at a particular fixed distance. When you said "when the distance decreased" you changed the question making it an unfair question.

Quote from: jccc
Electrons orbiting or not? Is there always acceleration between nucleus and electrons?
You should know the answer to this by now. Why do you keep asking it? I already told you that the concepts of velocity and acceleration at distances so small have no meaning whatsoever. I'm spending my summer doing a review of quantum mechanics using the text Introduction to Quantum Mechanics - Second Edition by David J. Griffiths. After the author derives the expression between d<x>/dt and the wave function on page 16 the author writes
Quote from: David J. Griffiths
What are we to make of this result? Note that we're talking about the "velocity" of the expectation value of x, which is not the same thing as the velocity of the particle. Nothing we have seen so far would enable us to calculate the velocity of a particle. It's not even clear what velocity means in quantum mechanics: If the particle doesn't have a determinate position (prior to measurement), neither does it have a well-defined velocity.

Since you keep asking questions such as Electrons orbiting or not? I can't see an end to it. Therefore I won't be responding to them again.
Title: Re: Why don't an atom's electrons fall into the nucleus and stick to the protons?
Post by: jccc on 27/07/2014 00:16:37
Pete, I went to college study physics and organic chemistry for 3 years. Bad bad score.

Don't remember your other questions, if I had the answers, should be replied.

All my confusion is very simple, my mind cannot imagine how subatomic world functioning according to what I read.

Is there a book less math more explanation about atomic structure? Everything I read online helped me little.
Such as quantum repulsion stuff. Not one place has a straight/decent answer to OP's question. How do you think I found this forum? I googled the OP"s question. So many people are like me, very simple and basic question.



 


 
Title: Re: Why don't an atom's electrons fall into the nucleus and stick to the protons?
Post by: PmbPhy on 27/07/2014 00:43:35
Quote from: jccc
Pete, I went to college study physics and organic chemistry for 3 years. Bad bad score.
Then you know math and physics then. That helps me a great deal.

Quote from: jccc
All my confusion is very simple, my mind cannot imagine how subatomic world functioning according to what I read.
It's not just you, Joe. Nobody understands it. That's how the Shut up and calculate rule came to be created.

Quote from: jccc
Is there a book less math more explanation about atomic structure? Everything I read online helped me little.
Such as quantum repulsion stuff.
What are you talking about when you write quantum repulsion stuff? This is the kind of question I've been asking all this time that never gets answered.

Quote from: jccc
Not one place has a straight/decent answer to OP's question.
That's because everybody thinks differently.

Quote from: jccc
Why is moving object slow down and stop? Because it get's tired.
No object in existence is like that. E.g. if you had an object in a vacuum in flat spacetime in an inertial frame and it had a constant velocity to start then it's velocity would remain at that velocity until something brought it to a stop. It would require doing work on the body to bring it to a halt.
Title: Re: Why don't an atom's electrons fall into the nucleus and stick to the protons?
Post by: JSS on 27/07/2014 01:16:22
Quantum Mechanics has absolutely nothing to do with WHY things happen. It is strictly a mathematical method for predicting things by assuming everything to have quantifiable properties, statistically found to be in certain ranges. Why any of it works is NOT the business of QM.
Title: Re: Why don't an atom's electrons fall into the nucleus and stick to the protons?
Post by: jccc on 27/07/2014 01:40:45
The explanation for why an electron does not fall into the nucleus comes from a fundamental concept in quantum mechanics: the Heisenberg uncertainty principle. Put simply, it states that you cannot know the position and momentum of a particle simultaneously. More rigorously stated, the product of the uncertainty of the position of a particle (Δx) and the uncertainty of its momentum (Δp) must be greater than a specified value:

ΔxΔp≥ℏ2


Now, as the electron approaches the nucleus, it's uncertainty in position decreases (if the electron is 10nm away from the nucleus, it could be anywhere within a spherical shell of radius 10nm, but if the electron is only 0.1nm away from the nucleus, that area is greatly reduced). According to the Heisenberg uncertainty principle, if you decrease the uncertainty of the electrons position, the uncertainty in its momentum must increase. This increased momentum uncertainty means that the electron will be moving away from the nucleus faster, on average.

Put another way, if we do know that at one instant, that the electron is right on top of the nucleus, we lose all information about where the electron will be at the next instant. It could stay at the nucleus, it could be slightly to the left or to the right, or it could very likely be very far away from the nucleus. Therefore, because of the the uncertainty principle it is impossible for the electron to fall into the nucleus and stay in the nucleus.

In essence, the uncertainty principle causes a sort of quantum repulsion, that keeps electrons from being too tightly localized near the nucleus.

Quantum repulsion? My head is exploding.

Pete,

I googled those answers, not my words.
Title: Re: Why don't an atom's electrons fall into the nucleus and stick to the protons?
Post by: PmbPhy on 27/07/2014 01:45:13
Quote from: JSS
Quantum Mechanics has absolutely nothing to do with WHY things happen. It is strictly a mathematical method for predicting things by assuming everything to have quantifiable properties, statistically found to be in certain ranges. Why any of it works is NOT the business of QM.
Predicting things is what is meant by explaining why. I used to believe the same thing that you just said but have been tilting the other way recently. For example; using quantum mechanics one can explain the nature of why alpha decay occurs and I recall that it can also predict the lifetime of radioactive nuclei that decay by alpha decay. It'd say that's a why question which has an answer.

You can also say that relativity is a mathematical model to. That doesn't mean you can't explain why moving clocks run slower than stationary ones.
Title: Re: Why don't an atom's electrons fall into the nucleus and stick to the protons?
Post by: jccc on 27/07/2014 02:04:23
May God have mercy, put a quantum grenade in my head ASAP.
Title: Re: Why don't an atom's electrons fall into the nucleus and stick to the protons?
Post by: evan_au on 27/07/2014 05:14:12
Quote from: jccc
In fusion reaction, two atoms impact so hard, even two nucleus crashed into one, how come electron escaped from the crash?
Before nuclear fusion is initiated, the Deuterium* gas is heated to somewhere around a million degrees (F, C or K, whatever you prefer...).

At these temperatures, the atoms are completely ionised, so the electrons have already escaped from the nuclei.

When two Deuterium nuclei run into each other with sufficient velocity to overcome their electrostatic repulsion they can approach within 1 femtometer. At these distances, the attraction of the strong nuclear force is greater than the electrostatic repulsion. The two deuterium nuclei can then fuse to form a Helium nucleus.

The electrons have not escaped from this reaction, as they were not present around the Deuterium nuclei at the start of the fusion reaction.

If you cool down the reaction products to room temperature, the plasma will cool into a mixture of neutral Helium atoms and Hydrogen molecules, with the electrons in a cloud around the positive nuclei.

*Sometimes a mix of Tritium & Deuterium is used, but it's easier to explain the D+D=He reaction.
Title: Re: Why don't an atom's electrons fall into the nucleus and stick to the protons?
Post by: jccc on 27/07/2014 06:46:01
Quote from: jccc
In fusion reaction, two atoms impact so hard, even two nucleus crashed into one, how come electron escaped from the crash?
Before nuclear fusion is initiated, the Deuterium* gas is heated to somewhere around a million degrees (F, C or K, whatever you prefer...).

At these temperatures, the atoms are completely ionised, so the electrons have already escaped from the nuclei.

When two Deuterium nuclei run into each other with sufficient velocity to overcome their electrostatic repulsion they can approach within 1 femtometer. At these distances, the attraction of the strong nuclear force is greater than the electrostatic repulsion. The two deuterium nuclei can then fuse to form a Helium nucleus.

The electrons have not escaped from this reaction, as they were not present around the Deuterium nuclei at the start of the fusion reaction.

If you cool down the reaction products to room temperature, the plasma will cool into a mixture of neutral Helium atoms and Hydrogen molecules, with the electrons in a cloud around the positive nuclei.

*Sometimes a mix of Tritium & Deuterium is used, but it's easier to explain the D+D=He reaction.

Thank you AU. You sound like work closely in this field.

At that high temperature, electron escape away from nucleus attraction force field? Why not escape into it? Is there a force field around nucleus that keeps electrons at certain distance? So no matter how energize electrons can only move outward from nucleus?

Isn't strong force quite mystery? Why gluons glue positive and negative quarks together not electron? Is quark really carries -1/3 or +2/3 charge? Why is add 3 quarks mass is not equal to proton mass? How to measure quarks mass?

What is quantum repulsion? Are you going to hate me? Please don't, it's my head's fault.
Title: Uncertainty
Post by: Insentient on 05/01/2015 19:17:47
The uncertainty can be explained as follows...We live in a certain dimension of our own consciousness, and our consciousness has a certain limit to what we are able to observe...We know that the universe is not governed by our consciousness, but we are unable to grasp the fact that we are governed by the universe...When we see an electron behave as a particle, it is because that is what the universe needs us to observe, but this does not mean that's what is actually happening. It just means that we asked a question, and had to get a lie for an answer, because if we don't get an answer, it's a lot worse than getting lied to...Every cause needs to have an effect, otherwise we wont be able to function in our consciousness of time...In the grand scheme of things, there are secrets that the universe needs to keep...We are the uncertainty  [;)]
Title: Re: Uncertainty
Post by: PmbPhy on 05/01/2015 19:41:40
Quote from: Insentient
The uncertainty can be explained as follows...
First off, welcome to the forum. It's always nice to see a new member join. In the second place, that is not correct. You're using the term uncertainty incorrectly. The uncertainly in a quantity is really the standard deviation in a physical observable and that's not what you were describing. See:
http://en.wikipedia.org/wiki/Uncertainty_principle

The uncertainty of an observable depends on the state that the quantum mechanical state that the system is in. If uncertainty was what you described then that wouldn't be the case.

Again, welcome to the forum! :)
Title: Re: Why don't an atom's electrons fall into the nucleus and stick to the protons?
Post by: Insentient on 08/01/2015 07:44:43
Thanks for the welcome! [:)] and thanks for the link also.., but the observer effect still holds the position of uncertainty (The position, and velocity of an object can not be quantified at exactly the same time)... well actually they can... It's just the wave length we determine, is actually our wave length, no matter what 'instruments' we use...We are stuck in a dimension 'like fish are stuck in a fish tank'.., and all the Latin symbols only get you so far before the realization occurs that we have enough information to build a 'spaceship' to get us the hell out of here!..Then we find out how the real math works ;)
Title: Re: Why don't an atom's electrons fall into the nucleus and stick to the protons?
Post by: lightarrow on 08/01/2015 22:49:31
At that high temperature, electron escape away from nucleus attraction force field? Why not escape into it? Is there a force field around nucleus that keeps electrons at certain distance? So no matter how energize electrons can only move outward from nucleus?
The more energy the electrons have, the less they are bound to the nucleus. It's not a force field but Schrodinger equation (solved  [:)]).

They can however interact with a nucleus in the same way as in LHC or other collider: high energy electrons are shooted against nuclei to explore their inside; the more energetic the electron is, the less its (wavefunction) wavelenght is, so it can explore and reveal smaller parts of the nucleus.

So, if you heat up hydrogen atoms at very high temperatures, you will ionize them, the electrons will move even faster, colliding with fast moving nuclei, and at very very high temperatures the electrons will begin to explore the nuclei's inside and to reveal quarks, and to generate new particles, even more exotic and massive and in even more number, in the collisions.

--
lightarrow
Title: Re: Why don't an atom's electrons fall into the nucleus and stick to the protons?
Post by: chiralSPO on 05/02/2015 18:48:21
jccc, I'm happy to discuss this topic further, but let's do it on this thread only.
Title: Re: Why don't an atom's electrons fall into the nucleus and stick to the protons?
Post by: jccc on 05/02/2015 20:17:08
If electrons stick to the protons, atoms will become neutral particles. Therefore matter will not form. Nor the universe.

The nature is God. Its creation beyond wonder.

As human, we wonder why? Why is electron and proton, the strongest force source in nature, not stick together within an atom?

I heard of two theories, QM, which I can't understand. The other one, you pretend you don't understand.
Title: Re: Why don't an atom's electrons fall into the nucleus and stick to the protons?
Post by: chiralSPO on 05/02/2015 20:36:10
Let's say for the moment that there is some negatively charged fluid other than electrons that was attracted to positively charged nuclei. A few questions immediately arise:

1) Why wouldn't the nucleus attract enough of the negatively charged fluid that it would become neutral, and no electrons are needed?

2) Is the entire universe negatively charged, or is there another as-yet-unobserved positively charged something that balances this out?

3) How can this fluid be observed?

If we assume that quantum mechanics is not a good model of the way things work on atomic/molecular scale, then we need to find new solutions for mysteries such as:

1) How do atoms interact to make molecules?

2) What is responsible for the bright line spectra of all the elements? (http://en.wikipedia.org/wiki/Emission_spectrum)

3) When the double slit experiment is performed with beams of electrons, why does it generate an interference pattern?
Title: Re: Why don't an atom's electrons fall into the nucleus and stick to the protons?
Post by: jccc on 05/02/2015 21:10:10
Let's say for the moment that there is some negatively charged fluid other than electrons that was attracted to positively charged nuclei. A few questions immediately arise:

1) Why wouldn't the nucleus attract enough of the negatively charged fluid that it would become neutral, and no electrons are needed?

2) Is the entire universe negatively charged, or is there another as-yet-unobserved positively charged something that balances this out?

3) How can this fluid be observed?

If we assume that quantum mechanics is not a good model of the way things work on atomic/molecular scale, then we need to find new solutions for mysteries such as:

1) How do atoms interact to make molecules?

2) What is responsible for the bright line spectra of all the elements? (http://en.wikipedia.org/wiki/Emission_spectrum)

3) When the double slit experiment is performed with beams of electrons, why does it generate an interference pattern?
1. The Earth can carry as much sea water as you can supply. The electron's force density/strength is equal to the   fluid density at atom radius.
2. The space is negative charged compare to matter/planet.
3. EM wave needs the fluid to exist.

1. Electron grabbing war is on going all the time.
2. Light is produced by electrostatic force vibrating in the space fluid. Every element has its own set of force density/strength due to their charges.
3. I am not sure yet, but I am sure electron is particle, not wave.
Title: Re: Why don't an atom's electrons fall into the nucleus and stick to the protons?
Post by: chiralSPO on 07/02/2015 19:52:54
1. The Earth can carry as much sea water as you can supply. The electron's force density/strength is equal to the   fluid density at atom radius.
But water doesn't repel itself. If you think this negative fluid gathers around the nucleus, presumably the effective charge of the nucleus would be less than a naked nucleus in the absence of negative fluid. How would this "electrons floating on sea of negatively charged space" work for heavier atoms, where there is a multiply charged nucleus and many electrons? How much negative space gets pulled close to the nucleus as it is bigger and more positively charged?

2. The space is negative charged compare to matter/planet.
So space is all negative, and matter is 50% positive 50% negative? How much negative charge is there in the space? This doesn't make much sense to me...

3. EM wave needs the fluid to exist.
So all of the experiments that couldn't find the aether are wrong?

1. Electron grabbing war is on going all the time.


2. Light is produced by electrostatic force vibrating in the space fluid. Every element has its own set of force density/strength due to their charges.
That might work as a qualitative description, but it breaks down when we consider the quantitative description. If it were just an issue of force constants (like springs) we wouldn't see the same harmonics as are observed (http://en.wikipedia.org/wiki/Hydrogen_spectral_series)

3. I am not sure yet, but I am sure electron is particle, not wave.

Glad to hear you are able to admit you are unsure about some things, but why are you so sure the electron is not a wave?
Title: Re: Why don't an atom's electrons fall into the nucleus and stick to the protons?
Post by: jccc on 07/02/2015 21:00:43
1. If there's only one positive change and 10 negative charges in the universe, will the negative charges gather around the positive charge?  Heavier nucleus have more positive charge, form denser space fluid balls, electrons are float at atom radius.

2.Who told you that the universe has same amount positive and negative charges?

3. Could be all wrong, they don't understand that the ether is charged, light speed is not constant, temperature and medium density involved.


1.

2. ??

3. Electron is a charged particle. It turns in EM field. It has mass. EM wave/light is moving force, no mass, no charge. 
Title: Re: Why don't an atom's electrons fall into the nucleus and stick to the protons?
Post by: Bill S on 07/02/2015 21:53:47
Jccc, somewhere Pete suggested you should read the Haifa Lectures. If you have done that you have probably seen the following:

“A trouble with Bohr’s atomic model is that, according to classical, electromagnetic theory, an accelerating electric charge, as is the case of the orbiting electric charge, must radiate energy away. In this case, the electron could not stay in a fixed orbit; it would spiral into the nucleus of the atom. Thus, such an atom could not be stable. Bohr resolved the problem by postulating that the angular momentum of the orbiting electron must be ‘quantized,’ in units of h/2π(The hint for this quantization may come from the fact that the units of Planck’s constant h are the same as those of angular momentum.) Thus, the classical orbital angular momentum, mvr, of the electron was taken by Bohr to be nh/2πwhere n = 0, 1, 2,…, and where v is the speed of the orbiting electron, and r is its radial distance from the nucleus. Thus, with Bohr’s model of the atom, the electron in a fixed orbit is in a particular energy level of the atom, En, until it would (acausally) ‘jump’ to a different energy level. When this ‘jump’ happens, to a lower energy level, the energy lost by the electron (and therefore the host atom), when it descends to the lower energy level Em, is taken up in the creation of a photon with frequency fnm”.

Does that not answer your question about how the electron avoids spiralling into the nucleus?
Title: Re: Why don't an atom's electrons fall into the nucleus and stick to the protons?
Post by: jccc on 07/02/2015 23:45:43
Not at all. Orbiting is totally BS.
Title: Re: Why don't an atom's electrons fall into the nucleus and stick to the protons?
Post by: PmbPhy on 08/02/2015 09:08:15
I think the following quote from QED by Richard Feynman is worth remembering. From page 15
Quote
I want to emphasize that light comes in this form - particles. It is very important to know that light behaves like particles, especially for those of you who have gone to school, where you were probably told something about light behaving like waves. I'm telling you the way it does behave - like particles.
Feynman talks about the wave aspects of the photon in his lecture series.
Title: Re: Why don't an atom's electrons fall into the nucleus and stick to the protons?
Post by: jeffreyH on 08/02/2015 16:19:32
The 'orbit' of an electron must be distributed in all directions. It is still an orbit however. If you think of the solar system model of an atom then an electron approaching the polar region of the proton WILL be able to 'crash' into the proton. This would be happening all the time which it doesn't because matter would not be stable. jccc, you neglect the distribution of the electron charge and the fact that it would not naturally combine with the proton. For a start the wavelengths of the proton and the electron will be different. Only when certain conditions are met can they combine to form a neutron. Some things just have to be accepted because the theory matches experimental evidence. How many people have to tell you this before you decide to believe them.
Title: Re: Why don't an atom's electrons fall into the nucleus and stick to the protons?
Post by: jccc on 08/02/2015 16:48:59
I'll never believe anything not logical to my mind.

Solar systems are light years apart. Do they share planets?

Those who believe orbiting atom structure should also believe magic.

Still think about photon? Particle wavelength? There's no such thing beside foolish theories.
Title: Re: Why don't an atom's electrons fall into the nucleus and stick to the protons?
Post by: Ethos_ on 08/02/2015 17:34:06
I'll never believe anything not logical to my mind.

Unless you start believing in experimental results, you'll never be practicing good science either!
Title: Re: Why don't an atom's electrons fall into the nucleus and stick to the protons?
Post by: PmbPhy on 08/02/2015 17:57:52
Quote from: jccc
I'll never believe anything not logical to my mind.
That's your problem. You seem to think that we want too convince you that something is logical. There will never ever be a reason to assume that you'll ever understand the logic. That simply might be beyond your understanding. The only thing that we're going to do, and that physics can do is to formulate laws of physics whose results can be tested and which we can use to make predictions which are also testable, the results being consistent with what's observed.

Quote from: jccc
Those who believe orbiting atom structure should also believe magic.
Not at all. Especially since this sentence is quite meaningless.

Quote from: jccc
Still think about photon? Particle wavelength? There's no such thing beside foolish minds.
Actually it's you who is the fool because you talk about things that you don't understand.

The wavelength associated with photons is the wavelength that a wave would have in order to describe things like interference patterns and to determine probability distributions.

This is what we've been telling you for years now and you refuse to read to really understand it. That's why you remain ignorant and make foolish comments. If you actually studied quantum mechanics you'd then learn what all of these things mean and what led physicists to these laws. Since you refuse to learn then you'll keep making silly comments like this.
Title: Re: Why don't an atom's electrons fall into the nucleus and stick to the protons?
Post by: jeffreyH on 08/02/2015 18:00:33
A free neutron decays in less than 15 minutes. Being the combination of a proton and electron this shows that this is unstable and the natural state is for them to be separate. This is experimentally verified. What does this tell you? Think about it for fifteen minutes.
Title: Re: Why don't an atom's electrons fall into the nucleus and stick to the protons?
Post by: PmbPhy on 08/02/2015 18:02:04
Quote from: jeffreyH
The 'orbit' of an electron must be distributed in all directions. It is still an orbit however.
Wrong. You've still got a way to go Jeff. It this was true then the electron would radiate. Since it doesn't then we know that the electron doesn't even move on a classical trajectory, regardless of its shape. Don't confuse orbits with orbitals.

See http://en.wikipedia.org/wiki/Atomic_orbital

Notice all the diagrams in that page and notice that there's nothing there that looks like an orbit. There are even orbitals where the electron is found in the nucleus.
Title: Re: Why don't an atom's electrons fall into the nucleus and stick to the protons?
Post by: jccc on 08/02/2015 18:50:28
Lit a candle, say it produces N photons per second. 

Those photons fly away at every direction at c speed.

At N meters away, you have a detector, its lens is 1 meter wide.

How many photons will you detect per second?
Title: Re: Why don't an atom's electrons fall into the nucleus and stick to the protons?
Post by: Bill S on 08/02/2015 19:12:02
We may suck as physics and maths and the likes,
But we husband our erudite goals.
We’d all do much better to get on our bikes
Than to stick around here feeding trolls!
Title: Re: Why don't an atom's electrons fall into the nucleus and stick to the protons?
Post by: jeffreyH on 08/02/2015 20:48:53
Quote from: jeffreyH
The 'orbit' of an electron must be distributed in all directions. It is still an orbit however.
Wrong. You've still got a way to go Jeff. It this was true then the electron would radiate. Since it doesn't then we know that the electron doesn't even move on a classical trajectory, regardless of its shape. Don't confuse orbits with orbitals.

See http://en.wikipedia.org/wiki/Atomic_orbital

Notice all the diagrams in that page and notice that there's nothing there that looks like an orbit. There are even orbitals where the electron is found in the nucleus.

We can consider these orbitals as standing waves. These standing waves do not revolve around the equator of a proton. They must be distributed all around the proton. There is no way in which an equator can be said to exist anyway as proton spin can not be viewed as in the rotation of a planet. It is more complex than that. The uncertainty involves the direction of this standing wave as well as the direction of the electron spin in the z direction. This can be spin up or spin down. The whole reason we say there is no radiation is the quantization of angular momentum. Do you think jccc understands ANY of this. Is it going to help him by making it so complicated straight away?
Title: Re: Why don't an atom's electrons fall into the nucleus and stick to the protons?
Post by: jeffreyH on 08/02/2015 21:12:21
Just to make this clear.

http://chemwiki.ucdavis.edu/Physical_Chemistry/Quantum_Mechanics/09._The_Hydrogen_Atom/Atomic_Theory/Electrons_in_Atoms/Electronic_Orbitals

"Visualizing Electron Orbitals

As discussed in the previous section, the magnetic quantum number (ml ) can range from –l to +l. The number of possible values is the number of lobes (orbitals) there are in the s, p, d, and f subshells. As shown in the chart above, the s subshell has one lobe, the p subshell has three lobes, the d subshell has five lobes, and the f subshell has seven lobes. Each of these lobes is labeled differently and is named depending on which plane the lobe is resting in. If the lobe lies along the x plane, then it is labeled with an x, as in 3px. If the lobe lies along the xy plane, then it is labeled with an x and a y such as dxy. Electrons are found within the lobes. The plane (or planes) that the orbitals do not fill are called nodes. These are regions in which there is a 0 probability density of finding electrons. For example, in the dyx orbital, there are nodes on planes xz and yz. This can be seen in the diagrams below."

You can see from the illustration that the s orbital is considered spherical. Being so the distribution is spherical. Other orbitals have lobes. This is complicated further by valence bonding and conduction bands.
Title: Re: Why don't an atom's electrons fall into the nucleus and stick to the protons?
Post by: PmbPhy on 08/02/2015 23:13:07
Quote from: jeffreyH
We can consider these orbitals as standing waves.
You're thinking about the Bohr model, aren't you? In the Bohr model, yes. You can view them as such. See
http://hyperphysics.phy-astr.gsu.edu/hbase/ewav.html

However the Bohr model is not a truly valid model. For example; it gives the impression that no matter what the wavelength of the electron there will always be orbital angular momentum and that's not true. In the completed version of quantum mechanics the ground state allows for zero angular momentum.
Title: Re: Why don't an atom's electrons fall into the nucleus and stick to the protons?
Post by: jeffreyH on 08/02/2015 23:52:07
Quote from: jeffreyH
We can consider these orbitals as standing waves.
You're thinking about the Bohr model, aren't you? In the Bohr model, yes. You can view them as such. See
http://hyperphysics.phy-astr.gsu.edu/hbase/ewav.html

However the Bohr model is not a truly valid model. For example; it gives the impression that no matter what the wavelength of the electron there will always be orbital angular momentum and that's not true. In the completed version of quantum mechanics the ground state allows for zero angular momentum.

That's very interesting. Do you have any links or reading material on the subject.
Title: Re: Why don't an atom's electrons fall into the nucleus and stick to the protons?
Post by: PmbPhy on 09/02/2015 01:18:53
Quote from: jeffreyH
That's very interesting. Do you have any links or reading material on the subject.
You can read all about in the very text; Quantum Mechanics by A.P. French and E.F. Taylor, pages 43-44.

I scanned it in and put it on my website for you. See
http://www.newenglandphysics.org/Science_Literature/Journal_Articles/other.htm

The part you want is Quantum Mechanics by A.P. French and E.F. Taylor, pages 43-44 and 2025-226
Title: Re: Why don't an atom's electrons fall into the nucleus and stick to the protons?
Post by: chiralSPO on 09/02/2015 20:27:23
Lit a candle, say it produces N photons per second. 

Those photons fly away at every direction at c speed.

At N meters away, you have a detector, its lens is 1 meter wide.

How many photons will you detect per second?

Still counting?

Assuming that no photons are blocked by the candle itself, the number of photons leaving the flame per second should be the same as the number passing through the sphere around the candle of radius N (did you mean for N photons per second and distance N to be the same N?) I will call this radius "R"

N photons per second equally distributed through a sphere of radius R meters means N/(4πR2)  photons per second per square meter. A circular lens with a diameter of 1 meter has an area of π/4 square meters, so it would capture N/(16R2) photons per second. (if this number is not a whole number, that does not imply that fractions of photons are involved, it is just an average rate--one half photon per second means one photon every two seconds)

What did this have to do with anything???
Title: Re: Why don't an atom's electrons fall into the nucleus and stick to the protons?
Post by: jccc on 09/02/2015 21:03:26
1. If there's only one positive change and 10 negative charges in the universe, will the negative charges gather around the positive charge?  Heavier nucleus have more positive charge, form denser space fluid balls, electrons are float at atom radius.

2.Who told you that the universe has same amount positive and negative charges?

3. Could be all wrong, they don't understand that the ether is charged, light speed is not constant, temperature and medium density involved.


1.

2. ??

3. Electron is a charged particle. It turns in EM field. It has mass. EM wave/light is moving force, no mass, no charge.

Still waiting for your feedback, chiralSPO.
Title: Re: Why don't an atom's electrons fall into the nucleus and stick to the protons?
Post by: Colin2B on 09/02/2015 22:56:59
[^2=1/16N photons per second. That means every 16N seconds detect 1 photon. I'll never able to take a picture of the flame. Does that gives you doubt that light is not particle but wave of energy?

Forget the photograph, patent the control mechanism.
You genuinely have a means of controlling the candle output so that when the detector is 1 meter away you can reduce the candle output to 1photon/sec?
Wow, respect.
Or am I missing something?

Title: Re: Why don't an atom's electrons fall into the nucleus and stick to the protons?
Post by: jccc on 09/02/2015 23:02:43
[^2=1/16N photons per second. That means every 16N seconds detect 1 photon. I'll never able to take a picture of the flame. Does that gives you doubt that light is not particle but wave of energy?

Forget the photograph, patent the control mechanism.
You genuinely have a means of controlling the candle output so that when the detector is 1 meter away you can reduce the candle output to 1photon/sec?
Wow, respect.
Or am I missing something?

There is no photon particle. Maybe. I hope your comment on my ideas about atomic structure, gravity and magnetism.
Title: Re: Why don't an atom's electrons fall into the nucleus and stick to the protons?
Post by: Ethos_ on 09/02/2015 23:17:57


There is no photon particle. Maybe.
The photon can represent itself in either manner, wave or particle. Traveling thru space the photon is transmitted as a wave but becomes a particle when the wave function collapses. If you are suggesting that the photon can't become a particle, give us your source as evidence.
Title: Re: Why don't an atom's electrons fall into the nucleus and stick to the protons?
Post by: jccc on 09/02/2015 23:39:18


There is no photon particle. Maybe.
The photon can represent itself in either manner, wave or particle. Traveling thru space the photon is transmitted as a wave but becomes a particle when the wave function collapses. If you are suggesting that the photon can't become a particle, give us your source as evidence.

How the photon transmitted to wave and become a particle? What's the mechanism? How electron emits photon? By imagination?
Title: Re: Why don't an atom's electrons fall into the nucleus and stick to the protons?
Post by: Ethos_ on 09/02/2015 23:47:41


There is no photon particle. Maybe.
The photon can represent itself in either manner, wave or particle. Traveling thru space the photon is transmitted as a wave but becomes a particle when the wave function collapses. If you are suggesting that the photon can't become a particle, give us your source as evidence.

How the photon transmitted to wave and become a particle? What's the mechanism? How electron emits photon? By imagination?
You will never have any luck answering a question with another question my friend. If you have no evidence, just say so and we can choose to contribute to this thread or simply ignore it.
Title: Re: Why don't an atom's electrons fall into the nucleus and stick to the protons?
Post by: jccc on 10/02/2015 00:08:24


There is no photon particle. Maybe.
The photon can represent itself in either manner, wave or particle. Traveling thru space the photon is transmitted as a wave but becomes a particle when the wave function collapses. If you are suggesting that the photon can't become a particle, give us your source as evidence.

How the photon transmitted to wave and become a particle? What's the mechanism? How electron emits photon? By imagination?
You will never have any luck answering a question with another question my friend. If you have no evidence, just say so and we can choose to contribute to this thread or simply ignore it.
You want truth or win an argument? 
Title: Re: Why don't an atom's electrons fall into the nucleus and stick to the protons?
Post by: Ethos_ on 10/02/2015 00:10:18


There is no photon particle. Maybe.
The photon can represent itself in either manner, wave or particle. Traveling thru space the photon is transmitted as a wave but becomes a particle when the wave function collapses. If you are suggesting that the photon can't become a particle, give us your source as evidence.

How the photon transmitted to wave and become a particle? What's the mechanism? How electron emits photon? By imagination?
You will never have any luck answering a question with another question my friend. If you have no evidence, just say so and we can choose to contribute to this thread or simply ignore it.
You want truth or win an argument?
I haven't heard any truth yet, but I'm still waiting.
Title: Re: Why don't an atom's electrons fall into the nucleus and stick to the protons?
Post by: jccc on 10/02/2015 00:23:07
Answer those questions, you'll closer to truth. Don't you agree?

How the photon transmitted to wave and become a particle? What's the mechanism? How electron emits photon?
Title: Re: Why don't an atom's electrons fall into the nucleus and stick to the protons?
Post by: Ethos_ on 10/02/2015 00:37:56
Answer those questions, you'll closer to truth. Don't you agree?

How the photon transmitted to wave and become a particle? What's the mechanism? How electron emits photon?
I believe I asked you first to explain why a photon can't become a particle. To respond with another question without answering mine is acting like mine was unworthy of an answer. Nevertheless, I will be waiting for your answer when and if you have one. If you don't have an answer, I will simply move on.
Title: Re: Why don't an atom's electrons fall into the nucleus and stick to the protons?
Post by: jccc on 10/02/2015 01:50:57
My theory is the space is charged fluid. All things are within it.

Positive charged nucleus attract the negative charged fluid to form atoms. The density of the fluid is Df=1/r^3. Electrons also attracted by positive charged nucleus and stable at atom radius where the attracting force is equal to the repel force. A demo
The electrons around the atom is like bond by a spring, need force to push in or pull out from the nucleus. Now if a force is applied, the electron will vibrating and produce pressure/EM wave across the space around it.

Every element has certain charge and bonding strength, therefore unique spectrum.

I don't understand the text book, how electron emits photon, be appreciate if you can explain the detail/logic.   
Title: Re: Why don't an atom's electrons fall into the nucleus and stick to the protons?
Post by: PmbPhy on 10/02/2015 02:22:01
Quote from: jccc
My theory is the space is charged fluid.
That's not a theory. It's a speculation/hypothesis. You should learn the difference between the two. See:
http://en.wikipedia.org/wiki/Theory

Quote from: jccc
Positive charged nucleus attract the negative charged fluid to form atoms.
What holds this charged particle together? If it's composed of charge and all like charges repel each other, what's holding them together?

Quote from: jccc
The electrons around the atom is like bond by a spring, need force to push in or pull out from the nucleus.
Why? What is responsible for the repulsive force? Since your electron is moving around inside the atom its accelerating. Since accelerating charge radiates energy why doesn't the energy radiate away and the electron spirals into the nucleus. Since we don't observe that it contradicts your theory. Why?
Title: Re: Why don't an atom's electrons fall into the nucleus and stick to the protons?
Post by: PmbPhy on 10/02/2015 02:24:43
Quote from: Ethos_
I believe I asked you first to explain why a photon can't become a particle.
It seems to me that he was saying that a photon "isn't" a particle. It's obvious why a photon can't transform into a particle with non-zero rest mass if that's what you had in mind. There'd be a violation of the conservation of momentum in one or more frames of reference.
Title: Re: Why don't an atom's electrons fall into the nucleus and stick to the protons?
Post by: jccc on 10/02/2015 02:32:06
Quote from: jccc
My theory is the space is charged fluid.
That's not a theory. It's a speculation/hypothesis. You should learn the difference between the two. See:
http://en.wikipedia.org/wiki/Theory

Quote from: jccc
Positive charged nucleus attract the negative charged fluid to form atoms.
What holds this charged particle together? If it's composed of charge and all like charges repel each other, what's holding them together?

Quote from: jccc
The electrons around the atom is like bond by a spring, need force to push in or pull out from the nucleus.
Why? What is responsible for the repulsive force? Since your electron is moving around inside the atom its accelerating. Since accelerating charge radiates energy why doesn't the energy radiate away and the electron spirals into the nucleus. Since we don't observe that it contradicts your theory. Why?
1. OK, just my theory.

2. Nucleus is positive charged, electron and space fluid are negative charged.

3. In my model, electrons are not moving around but bonded by nucleus attraction force and the space fluid balls repel/float force. Did you watch that video? Seems you still don't know what I am trying to picture, sorry about my poor English.
Title: Re: Why don't an atom's electrons fall into the nucleus and stick to the protons?
Post by: jccc on 10/02/2015 05:30:06
If Coulombs's law stands universally, we should assume that every atom or charged particle are connected by their force field across the whole space.

An atoms force field does not end at atom radius, but extend to infinity. In whole, an atom or planet maybe electrically neutral, but Every charge within has its own force field beyond distance, those forces overlapped to produce chemical bonding, magnetism and gravity.

Title: Re: Why don't an atom's electrons fall into the nucleus and stick to the protons?
Post by: jccc on 10/02/2015 07:55:14
If proton is in fact built by U and D quarks, then maybe the proton is like one small woman sleep between two fat man. It happens in real life. At least, the 3 quarks could electrically form into a group that we called proton.

We should assume all nucleus have some degree of polarity according their unique charges carried and the structure of all quarks stick together.

An iron atom maybe is a small magnet, the positive pole of the nucleus attract dense space fluid to form a force field that its density/strength drop off at 1/r^3, that matches the observation, and fits Coulombs law.



Title: Re: Why don't an atom's electrons fall into the nucleus and stick to the protons?
Post by: evan_au on 10/02/2015 11:21:09
My calculation will be N/16N^2=1/16N photons per second. That means every 16N seconds detect 1 photon.

You can do some quick "sanity checks" on the answer:
A) Double the number of photons emitted per second (eg by putting a second candle in the center of the sphere), and you should double the number of photons detected. However, with the proposed answer of 1/16N, the number of photons will halve. This does not compute.
B)  Reduce the radius of the detector. The number of photons detected should reduce. However, with the proposed answer of 1/16N, the number of photons does not change. This does not compute.
C) Increase the radius of the sphere, while leaving the detector size unchanged. The number of photons should reduce. In the proposed answer, the number of photons does not change. This does not compute.

So I conclude that the proposed answer of 1/16N fails all 3 sanity checks.

The alternative answer from chiralSPO is N/(16R2). This passes tests A & C.  2 out of 3 is a much better score.
(It does not pass test B, because the radius of the detector is assumed fixed at 1 meter, rather than being parametrized with a variable like radius r, and area πr2. There is also an approximation here that is more accurate if r is much smaller than R.)

Title: Re: Why don't an atom's electrons fall into the nucleus and stick to the protons?
Post by: Colin2B on 10/02/2015 15:15:00
My calculation will be N/16N^2=1/16N photons per second. That means every 16N seconds detect 1 photon. I'll never able to take a picture of the flame. Does that gives you doubt that light is not particle but wave of energy?

Jcc
I appreciate you are searching for understanding, but there are some gaps in your understanding of basic maths etc that you need to address.

[^2=1/16N photons per second. That means every 16N seconds detect 1 photon. I'll never able to take a picture of the flame. Does that gives you doubt that light is not particle but wave of energy?

Forget the photograph, patent the control mechanism.
You genuinely have a means of controlling the candle output so that when the detector is 1 meter away you can reduce the candle output to 1photon/sec?
Wow, respect.
Or am I missing something?

I apologise for my lighthearted answer, I should have been more direct. To add to the points made by evan:
The assumptions you use for your formula include an embedded definition of the number of photons/sec coming out of the candle. Thus with the detector at 1m you have 1photon/s, at 2m you have 2photons/s, etc. this clearly cannot be. You have built a feedback loop into your assumptions.
If you assume a more reasonable number of photons say 10^15 your formula says you have to move your detector to 10^15m and at that distance you are unlikely to see a candle let alone photograph it. In this case you and evan will no longer be at loggerheads. I was brought up on a sliderule so like evan I try to use sanity checks as often as possible.
Until you address the problem of your misunderstanding of maths and formulation of ideas you will meet only with frustration and misunderstanding in these discussions.

Thank you for your invitation to read your posts on atomic structure, gravity and magnetism. If you agree to address your misunderstanding, as above, I will read them.
Thank you
Title: Re: Why don't an atom's electrons fall into the nucleus and stick to the protons?
Post by: jccc on 10/02/2015 19:06:14
There is a picture of elephant right in front of your faces.

Seems you only see the tail of it. Good eyes!
Title: Re: Why don't an atom's electrons fall into the nucleus and stick to the protons?
Post by: Bill S on 10/02/2015 20:15:26
Quote from: Ethos
The photon can represent itself in either manner, wave or particle. Traveling thru space the photon is transmitted as a wave but becomes a particle when the wave function collapses. If you are suggesting that the photon can't become a particle, give us your source as evidence.

I think this is an oversimplification that risks letting in the sort of argument that jccc seem to be having trouble with.

If you regard a photon not as something that can change between a wave and a particle, you can run into trouble that might not be there if you see it as something that is neither a wave nor a particle, but is something we do not (yet) fully understand which can appear as one or the other, depending on how we observe it.
Title: Re: Why don't an atom's electrons fall into the nucleus and stick to the protons?
Post by: Ethos_ on 10/02/2015 22:22:22
Quote from: Ethos
The photon can represent itself in either manner, wave or particle. Traveling thru space the photon is transmitted as a wave but becomes a particle when the wave function collapses. If you are suggesting that the photon can't become a particle, give us your source as evidence.

I think this is an oversimplification that risks letting in the sort of argument that jccc seem to be having trouble with.


That's a valid point Bill, describing the photon wave/particle duality is the question here. Because we have yet to fully understand the process by which the photon can represent itself as either particular  entity at any given moment in time or circumstance, we are left with an incomplete description of the phenomenon.

Why and how the wave function collapses is something I've yet to understand myself. I'm not sure if anyone does but this collapse of the wave function is the prominent view today and until a better model comes along, I view it as the best example of reality.
Title: Re: Why don't an atom's electrons fall into the nucleus and stick to the protons?
Post by: Bill S on 10/02/2015 22:35:12
Quote
Why and how the wave function collapses is something I've yet to understand myself. I'm not sure if anyone does but this collapse of the wave function is the prominent view today and until a better model comes along, I view it as the best example of reality.

It seems as though "wave function collapse" is becoming an outdated term in some circles. When I used it in another forum, I was told to "get up-to-date and think decoherence".  [:)]
Title: Re: Why don't an atom's electrons fall into the nucleus and stick to the protons?
Post by: Ethos_ on 10/02/2015 22:52:14
Quote
Why and how the wave function collapses is something I've yet to understand myself. I'm not sure if anyone does but this collapse of the wave function is the prominent view today and until a better model comes along, I view it as the best example of reality.

It seems as though "wave function collapse" is becoming an outdated term in some circles. When I used it in another forum, I was told to "get up-to-date and think decoherence".  [:)]
When I think about the definition for the word "coherence" and it's antithesis "decoherence", I concede that this definition may be a little more accurate. However, using the word "collapse" is not that far from the same meaning. Where collapse refers to a falling apart, decoherence suggests more of a disconnection or more correctly; "a less connected state". So I would agree that "a less connected state of being" might be a better definition. And the term "decoherence" or "scattered" would be a fair description.
Title: Re: Why don't an atom's electrons fall into the nucleus and stick to the protons?
Post by: JohnDuffield on 10/02/2015 23:28:03
Why and how the wave function collapses is something I've yet to understand myself. I'm not sure if anyone does but this collapse of the wave function is the prominent view today and until a better model comes along, I view it as the best example of reality.
Check out Jeff Lundeen (http://www.photonicquantum.info/) who says wavefunction is real (http://www.photonicquantum.info/Research/SemiTechnical_Wavefunction.html), and is something that right there in the lab. Going on from that, I think wavefunction collapse is real too. Conceptiually, I think it's something like the optical Fourier transform:

(https://www.thenakedscientists.com/forum/proxy.php?request=http%3A%2F%2Fcns-alumni.bu.edu%2F%7Eslehar%2Ffourier%2Ffourier3.gif&hash=780cc0ec017e65e77263550fdf39e15b)
http://cns-alumni.bu.edu/~slehar/fourier/fourier.html

Sorry if I've said this before, but think of a photon as something like a subterranean seismic wave. It travels  through space rather than through rock, but it takes "many paths". So it goes through both slits of the dual-slit experiment. But when you detect it at one slit you do something like an optical Fourier transform on it and turn it into a dot. So it goes through that slit only. And when you detect it at the screen you do something like the optical Fourier transform to it and turn it into a dot. Try to imagine feeling a seismic wave with a stick the size of a mountain range. If you absorbed the whole seismic wave and if your stick was pointy, you might think the seismic wave was pointlike. Weak measurement work by Jeff Lundeen et al is something like mapping out the seismic wave with a lot of little sticks. His PhD supervisor was Aephraim Steinberg who heads up another team. See his webpage too (http://www.physics.utoronto.ca/~aephraim/) and especially this depiction of a photon going through both slits:

(https://www.thenakedscientists.com/forum/proxy.php?request=http%3A%2F%2Fwww.physics.utoronto.ca%2F%7Eaephraim%2Fpics%2F3D.jpg&hash=0b5bb35528c03d2be8235f9b697c023e)
Title: Re: Why don't an atom's electrons fall into the nucleus and stick to the protons?
Post by: jccc on 11/02/2015 01:40:00
A particle, should be either at rest or at speed v without extra force apply on it.

If it is moving, it moves along straight line. If it hits something, it may pass through/halfway, reflect or deflect. It will lose momentum/speed anyways.

Does any light or EM wave ever slowdown?

If light is particle, how a particle hits mirror and reflect back? What bounces it back? The electron? The nucleus? The empty space within silver atoms?

May truth sets our minds free sooner.

Title: Re: Why don't an atom's electrons fall into the nucleus and stick to the protons?
Post by: PmbPhy on 11/02/2015 20:32:48
Quote from: jccc
A particle, should be either at rest or at speed v without extra force apply on it.
You don't know that. What we experience is only what our senses are able to tell us and they're only able to give us an average of what is really going on. Particles don't really have a position as a function of time. What we observe to be like that is merely our senses averaging what they're detecting.

Still not willing to learn quantum mechanics, huh?
Title: Re: Why don't an atom's electrons fall into the nucleus and stick to the protons?
Post by: jccc on 11/02/2015 20:45:47
Quote from: jccc
A particle, should be either at rest or at speed v without extra force apply on it.
You don't know that. What we experience is only what our senses are able to tell us and they're only able to give us an average of what is really going on. Particles don't really have a position as a function of time. What we observe to be like that is merely our senses averaging what they're detecting.

Still not willing to learn quantum mechanics, huh?

If you can tell me how an electron emits photon, I can learn faster. Agree?
Title: Re: Why don't an atom's electrons fall into the nucleus and stick to the protons?
Post by: PmbPhy on 11/02/2015 20:53:56
Quote from: jccc
If you can tell me how an electron emits photon, I can learn faster. Agree?
That's not the way life works. If we could learn quantum mechanics that way then there'd be no reason to read textbooks or take courses on the subject. That's a fact that you've never been able to grasp.

Anyway, see http://www.newton.dep.anl.gov/askasci/phy05/phy05070.htm   

It might help you.
Title: Re: Why don't an atom's electrons fall into the nucleus and stick to the protons?
Post by: jccc on 11/02/2015 22:15:16
Pete,

This thread is the most asked question in every science forum. I searched them all and QM is the only answer I found. I also puzzled all my life about gravity and magnetism.

I believe you already read my hypothesis about all 3 in the past few pages. I am seriously eager to learn your thoughts about them.

I said you are my teacher and always will be, please take my word.

Title: Re: Why don't an atom's electrons fall into the nucleus and stick to the protons?
Post by: Ethos_ on 11/02/2015 22:17:02
Quote from: jccc
If you can tell me how an electron emits photon, I can learn faster. Agree?
That's not the way life works. If we could learn quantum mechanics that way then there'd be no reason to read textbooks or take courses on the subject. That's a fact that you've never been able to grasp.

Anyway, see http://www.newton.dep.anl.gov/askasci/phy05/phy05070.htm   

It might help you.
Does it helps a pest?
You don't consider yourself a pest do you? How on earth could you come to such a conclusion.............tongue in cheek! [;)] [;D]
Title: Re: Why don't an atom's electrons fall into the nucleus and stick to the protons?
Post by: Ethos_ on 11/02/2015 22:30:20
Quote from: jccc
If you can tell me how an electron emits photon, I can learn faster. Agree?
That's not the way life works. If we could learn quantum mechanics that way then there'd be no reason to read textbooks or take courses on the subject. That's a fact that you've never been able to grasp.

Anyway, see http://www.newton.dep.anl.gov/askasci/phy05/phy05070.htm   

It might help you.
Does it helps a pest?
You don't consider yourself a pest do you? How on earth could you come to such a conclusion.............tongue in cheek! [;)] [;D]

Pete says so, I trust him a lot besides QM.
If you trust him, and you should for he is quite knowledgeable, then take his advice and read a few of the links he's provided. If you give QM a chance, you might learn enough some day to have one of those extraordinary WOW moments of scientific revelation. Give it a chance.
Title: Re: Why don't an atom's electrons fall into the nucleus and stick to the protons?
Post by: jccc on 11/02/2015 22:38:33
Share me your WOW please, don't be a teaser.
Title: Re: Why don't an atom's electrons fall into the nucleus and stick to the protons?
Post by: Ethos_ on 11/02/2015 22:51:56
Share me your WOW please, don't be a teaser.
Those WOW moments don't come cheaply, one must involve themselves deeply in research and study. Neither Pete nor I can just snap our fingers and reveal anything to a student that is unwilling to put forth the time and effort. A word to the wise is sufficient.
Title: Re: Why don't an atom's electrons fall into the nucleus and stick to the protons?
Post by: PmbPhy on 12/02/2015 00:43:56
Quote from: jccc
Share me your WOW please, don't be a teaser.
Some of those texts will teach you about physics and that means learning what the job of physics is. It will tell you what physics can do and what it can't do. For example; I know that when an electron transitions from an higher quantum level in an atom to a lower one it will spit out a photon. How it does that I don't know. But it's not the job of physics to tell us that. We seek to learn such things but in the end we know that we won't get all the answers. Therefore learning the philosophy of physics will help you a great deal. The reason you're not getting the answers that you're looking for is because you're weak on knowing what physics can and can't do. It will also help you understand things such as questions like "What does it mean to say that someone is measuring the time interval between two events?" etc.
Title: Re: Why don't an atom's electrons fall into the nucleus and stick to the protons?
Post by: jccc on 12/02/2015 01:04:28
Pete,

What is quantum level? How electrons change level? Is it a voltage thing?
Title: Re: Why don't an atom's electrons fall into the nucleus and stick to the protons?
Post by: chiralSPO on 12/02/2015 01:12:39
Electrons in an atom can be described by quantum numbers (n, l, ml and ms) No two electrons in the same atom can have the same numbers. You can think of it as a kind of address. One can also calculate the energy of the electron based on its address (quantum numbers). see more here: http://en.wikipedia.org/wiki/Quantum_number
Title: Re: Why don't an atom's electrons fall into the nucleus and stick to the protons?
Post by: jeffreyH on 12/02/2015 01:40:46
jccc Think of the electrons like cog wheels. When two are interlinked then as one rotates clockwise the other will have to rotate anti-clockwise. This is like spin up and spin down in a very simplified analogy. Pairs of electrons have to be in this configuration due to the Pauli exclusion principle.
Title: Re: Why don't an atom's electrons fall into the nucleus and stick to the protons?
Post by: jccc on 12/02/2015 05:03:15
Pete,

This thread is the most asked question in every science forum. I searched them all and QM is the only answer I found. I also puzzled all my life about gravity and magnetism.

I believe you already read my hypothesis about all 3 in the past few pages. I am seriously eager to learn your thoughts about them.

I said you are my teacher and always will be, please take my word.

Dear Pete, find any mistakes?
Title: Re: Why don't an atom's electrons fall into the nucleus and stick to the protons?
Post by: chiralSPO on 12/02/2015 14:27:35
jccc, I know we all sound like broken records by now, but I will repeat yet again:

Your approach is completely bass ackwards--with few exceptions, scientific progress is made by an iterative process of making observations, constructing a model based on those observations and current scientific understanding, checking the model against future observations, and finally discarding or modifying the model, repeating the cycle until there is excellent agreement between observation and theory.

Contrary to common belief, science does not tell us how the universe works, it lets us imagine systems that imitate (closely or exactly) what we observe of the universe. We will never find "truth" only good predictive models. We often use models that we know are wrong because they still make good predictions and are simple to use. As long as we know the limitations of the theories, when assumptions can be made, and how the real world is likely to deviate from the simplistic models, using "wrong" models is very practical.

You have proposed a model, but instead of checking it against observations, you are trying to "reason" that it is useful based on how much sense it makes to you. This model may be useful for making some types of predictions, but that doesn't mean that it is a correct worldview, and it doesn't imply anything about how the world works.

You claim to have a theory, but it sounds more like a "Just So" story (http://en.wikipedia.org/wiki/Just_So_Stories) than scientific thought.


If you really care to know more about these three questions you have, take Pete up on his offers and actually read some of the material he provides links to. You have to suspend disbelief at the begining and just try to understand the reasoning behind quantum theories. Once you learn enough about it, you will realize the ways in which it is self consistent and how it can predict/describe what goes on on molecular, atomic and subatomic scales. You will also learn where the model fails. Remember all scientific models are wrong! Some just happen to be useful anyway.
Title: Re: Why don't an atom's electrons fall into the nucleus and stick to the protons?
Post by: PmbPhy on 12/02/2015 15:09:33
Pete,

What is quantum level? How electrons change level? Is it a voltage thing?
I thought I made it clear to you that I will not teach you what you can find it a text. There's a huge amount of information to be digested that you're not willing to work to do.
Title: Re: Why don't an atom's electrons fall into the nucleus and stick to the protons?
Post by: jccc on 12/02/2015 17:41:25
Pete,

What is quantum level? How electrons change level? Is it a voltage thing?
I thought I made it clear to you that I will not teach you what you can find it a text. There's a huge amount of information to be digested that you're not willing to work to do.

Did you find any mistake in my pest theories?

Have you missed any chance to show off your science knowledge?
Title: Re: Why don't an atom's electrons fall into the nucleus and stick to the protons?
Post by: Ethos_ on 13/02/2015 04:21:50
Still don't love me?

I am going to share your pm soon.
I think you'll find many here consider such action inappropriate.
Title: Re: Why don't an atom's electrons fall into the nucleus and stick to the protons?
Post by: PmbPhy on 13/02/2015 14:34:37
Still don't love me?

I am going to share your pm soon.
If you share any PM I've ever sent you then I promise never to PM you again.
Title: Re: Why don't an atom's electrons fall into the nucleus and stick to the protons?
Post by: PmbPhy on 13/02/2015 16:59:37
Quote from: jccc
If you share any PM I've ever sent you then I promise never to PM you again.
I'm not gay from cripes sake so I don't love you.  I can barely stand you. :)

Please stop with this childish nonsense and act your age. I have no time for this childish behavior.
Title: Re: Why don't an atom's electrons fall into the nucleus and stick to the protons?
Post by: jccc on 13/02/2015 17:56:51
Well, let's get back to biz.

How come you find no bones in my ideas about gravity, magnetism and atom?

What's your take?
Title: Re: Why don't an atom's electrons fall into the nucleus and stick to the protons?
Post by: PmbPhy on 13/02/2015 18:25:02
Quote from: jccc
How come you find no bones in my ideas about gravity, magnetism and atom?
Because I've been ignoring you. I've been waiting until the day when you decide to get serious and stop being lazy about all of this and are ready to do the work necessary to understand physics.

What are these ideas you have?
Title: Re: Why don't an atom's electrons fall into the nucleus and stick to the protons?
Post by: jccc on 13/02/2015 23:04:24
let me share my wow moment.

yesterday, if i can trade my life to understand any of the 3, i will.

today i found them all, at least i believe.

how do i feel?

i want to transplant into a butterfly.




Title: Re: Why don't an atom's electrons fall into the nucleus and stick to the protons?
Post by: Ethos_ on 13/02/2015 23:16:36
let me share my wow moment.

yesterday, if i can trade my life to understand any of the 3, i will.

today i found them all, at least i believe.

how do i feel?

i want to transplant into a butterfly.
Continue with your present behavior jccc and all you can rightfully expect from everyone here is for us to start ignoring you.

As for myself, the time has come for me to do just that. Unless you start making some sense, and show real interest in learning and or sharing, you can expect no further replies from yours truly......................Bye, bye
Title: Re: Why don't an atom's electrons fall into the nucleus and stick to the protons?
Post by: PmbPhy on 14/02/2015 02:49:29
let me share my wow moment.

yesterday, if i can trade my life to understand any of the 3, i will.

today i found them all, at least i believe.

how do i feel?

i want to transplant into a butterfly.
Continue with your present behavior jccc and all you can rightfully expect from everyone here is for us to start ignoring you.

As for myself, the time has come for me to do just that. Unless you start making some sense, and show real interest in learning and or sharing, you can expect no further replies from yours truly......................Bye, bye
I'm with my dear friend Ethos_ . In fact I declare a boycott on jccc.
Title: Re: Why don't an atom's electrons fall into the nucleus and stick to the protons?
Post by: PmbPhy on 14/02/2015 03:15:33
I found your greatest science achievement is artificial skin.
What in the world are you talking about now? It's dumb stuff like this that makes us want to ignore you.

I'm going to take a last shot and helping you understand orbitals and that's IT!!

See http://www.chem1.com/acad/webtut/atomic/WhyTheElectron.html
Title: Re: Why don't an atom's electrons fall into the nucleus and stick to the protons?
Post by: jccc on 14/02/2015 05:25:52
Atom, how is it build? If you only have two charged particles, 1 proton and 1 electron.

According to physics  law, they attract each other, the closer the stronger. f=q1q2/r^2. they become a little dot with positive force field on one side and negative the other. That's not the atom we see.

You have two choices, add more charge/matter or change your law. You forgot you have more matter, so you changed your law, one for logic, one for wish.

You forgot only have 1 law, the truth.

Do you really need to change your law? If you remembered the other matter that is the charged space itself.

See if you can debunk the above, Pete.

Title: Re: Why don't an atom's electrons fall into the nucleus and stick to the protons?
Post by: jccc on 14/02/2015 05:29:20
And this:

A particle, should be either at rest or at speed v without extra force apply on it.

If it is moving, it moves along straight line. If it hits something, it may pass through/halfway, reflect or deflect. It will lose momentum/speed anyways.

Does any light or EM wave ever slowdown?

If light is particle, how a particle hits mirror and reflect back? What bounces it back? The electron? The nucleus? The empty space within silver atoms?
Title: Re: Why don't an atom's electrons fall into the nucleus and stick to the protons?
Post by: PmbPhy on 14/02/2015 05:46:53
Quote from: jccc
See if you can debunk the above, Pete.
This is another of the serious problems that you have. You're unable to grasp the fact that I just said
Quote
I'm going to take a last shot and helping you understand orbitals and that's IT!!
When someone says "that's IT!!" it means no more conversation. And yet you keep acting like I'm going to respond to the crap you post. Sheesh! This is exactly what it means to be a troll so please STOP trolling.
Title: Re: Why don't an atom's electrons fall into the nucleus and stick to the protons?
Post by: jccc on 14/02/2015 07:18:38
My theory is the space is negative charged elastic fluid. All things are within it.

Positive charged nucleus attracts the negative charged fluid to form atom. The density of the fluid is Df=1/r^3. Electrons also attracted by positive charged nucleus and stable at atom radius where the attracting force is equal to the repel force. A demo
The electrons around the atom is like bond by a spring, need force to push in or pull out from the nucleus. Now if a force is applied, the electron will vibrating and produce pressure/EM wave across the space around it. Every element has certain charge and bonding strength, therefore unique spectrum.

If Coulombs's law stands universally, we should assume that every atom or charged particle are connected by their force field across the whole space.

An atoms force field does not end at atom radius, but extend to infinity. In whole, an atom or planet maybe electrically neutral, but Every charge within has its own force field beyond distance, those forces overlapped to produce chemical bonding, magnetism and gravity. Ever wonder why is Fe=q1q2/r^2, Fg=m1m2/r^2, and mass proportional to proton numbers within it?

If proton is in fact built by U and D quarks, then maybe the proton is like one small woman sleep between two fat man. It happens in real life. At least, the 3 quarks could electrically form into a group that we called proton.

We should assume all nucleus have some degree of polarity according their unique charges carried and the structure of all quarks stick together.

An iron atom maybe is a small magnet, the positive pole of the nucleus attract dense space fluid to form a force field that its density/strength drop off at 1/r^3, that matches the observation, and fits Coulombs law.
Title: Re: Why don't an atom's electrons fall into the nucleus and stick to the protons?
Post by: syhprum on 14/02/2015 07:55:37
To transmogrify into a butterfly you must undergo a metamorphosis.

on a more serious note how do electrons survive floating in this negative charged eather
Title: Re: Why don't an atom's electrons fall into the nucleus and stick to the protons?
Post by: jccc on 14/02/2015 08:03:19
How about just the soul part?
Title: Re: Why don't an atom's electrons fall into the nucleus and stick to the protons?
Post by: jccc on 14/02/2015 08:10:47
To transmogrify into a butterfly you must undergo a metamorphosis.

on a more serious note how do electrons survive floating in this negative charged eather

how you survive all the enemies in the world around you? You have no where to go but within them.

image the electron carries -1, the fluid carries - 1/10^20/cm^3. something like that.

the electron surrounded by negative fluid, seeks any positive charges in sight, fly to the closest proton. getting closer, the dense negative fluid ball around the proton keeps it not too close. The attraction force is f=pxe/r^2, the repel force form the negative field strength is f=1/r^3, the 2 has to balance at atom radius.
Title: Re: Why don't an atom's electrons fall into the nucleus and stick to the protons?
Post by: jccc on 14/02/2015 08:17:17
if this theory has merit, we may assume space is bounded by itself, therefore infinity.

Infinity space/negative charged elastic fluid, with any amount of protons and electrons able to build universe. Centered by positive nucleus and space fluid ball and electron outer made atoms.

No matter if proton number or charge equal to electron number or charge. As long opposite charges exist, groups will from. Any forces will act relatively.
Title: Re: Why don't an atom's electrons fall into the nucleus and stick to the protons?
Post by: jccc on 14/02/2015 10:06:31
Who says the universe has to have same amount positive and negative charges? If there are only 100 protons and 100000 electrons, would matter/atom able to form?

According to Coulombs law, the universe should be a little ball of matter sounded by empty space. The matter part is the two kinds of particles mixture.

But it might not look like matter we know. Without negative charged space fluid, matter will be tooo dense to support life.
Title: Re: Why don't an atom's electrons fall into the nucleus and stick to the protons?
Post by: jccc on 14/02/2015 10:45:09
QM atom structure suggests that atoms are like different sized ping pong balls.

Negative electron orbital shell, empty space and center positive nucleus.

3 questions, how electron not stick with nucleus?

Why atoms deep in Planet center not crash under high pressure?

How 2 O atoms form into 1 O2 since the shell of the 2 ping pong balls are all negative charged?

 
Title: Re: Why don't an atom's electrons fall into the nucleus and stick to the protons?
Post by: jccc on 14/02/2015 12:29:07
What is space? Anywhere force able to reach is space.

Electrostatic forces has no boundary, therefore space is infinity. Even there is only one charge exist it has boundless reach.

From geometry view, 1,2,3 dimension all infinity.

?
Title: Re: Why don't an atom's electrons fall into the nucleus and stick to the protons?
Post by: jccc on 14/02/2015 13:21:10
if there are only two electrons in empty space, they will along a straight line fly away forever.

if 1 e 1 p, no matter how far away, they will along a straight line fly together. if they are solid balls, the atom/universe will be 2 connected balls with opposite force fields on each side.

if the space is filled with positive charged elastic fluid, it will condensed to the electron to form nucleus and proton will be float at atom radius. an anti hydrogen.

see, no matter how many electron or proton, no matter how big is space, no matter how much charge they each carries, if you got all 3, as long as forces exist between them, you can build universe.

 
Title: Re: Why don't an atom's electrons fall into the nucleus and stick to the protons?
Post by: chiralSPO on 14/02/2015 17:43:43

An atoms force field does not end at atom radius, but extend to infinity. In whole, an atom or planet maybe electrically neutral, but Every charge within has its own force field beyond distance, those forces overlapped to produce chemical bonding, magnetism and gravity. Ever wonder why is Fe=q1q2/r^2, Fg=m1m2/r^2, and mass proportional to proton numbers within it?


Yes the electrostatic force field of a proton is unbounded, and the electrostatic force of an electron is unbounded. But in a neutral atom those forces effectively cancel out at any mesoscopic or macroscopic distance. Use the equation you provide (Fe=q1q2/r^2) and calculate the net force on a particle of charge +1 if acted on by a proton of charge +1, at a distance of 1 mm and an electron of charge –1 at a distance of 0.99999995 mm. It's a pretty small number. Also since the electron's AVERAGE location is the same as the proton's AVERAGE location, these net AVERAGE force is ZERO.

The inverse square law is the same for electrostatic and gravitational attraction because we live in a 3D world.

You also forgot about the masses of the neutrons and electrons which should be included in the mass of a planet (the Earth has about 2 × 1021 kg of electrons)
Title: Re: Why don't an atom's electrons fall into the nucleus and stick to the protons?
Post by: jccc on 14/02/2015 19:28:17
To transmogrify into a butterfly you must undergo a metamorphosis.

on a more serious note how do electrons survive floating in this negative charged eather

so far so good? thank you for inspiring me putting ideas out.

if atomic structure theory was wrong, all theories about matter could be wrong. what is mass if matter carries no charge? if matter has no charge/force, how you measure it?

maybe proton charge is not +1, the proton and fluid ball combined net charge is +1. we can never see a single proton or neutron, they all surrounded by fluid ball like a solid rock.

maybe the size of proton and neutron are 1800 electron size, when they passing mass spectrometer, the space fluid inside the tube puts resistance on them, the bigger ball curves more. just like shoot two beach balls horizontally, the bigger ball drops faster. ???

mass equal to matter's force field strength.  a gold ball contains more charges therefore heavier than a silver ball. when the ball moving in space, the resistance following speed, the faster you go the heavier you are.

need sleep, later.
Title: Re: Why don't an atom's electrons fall into the nucleus and stick to the protons?
Post by: evan_au on 15/02/2015 10:04:27
Quote from: jccc
Why atoms deep in Planet center not crash under high pressure?
That would be because the electrons of adjacent atoms repel each other, at close range (you can get more precise by including quantum effects).

This is because in something of the scale of the Earth, the electromagnetic force is far stronger than the gravitational force.

Take it up a notch, and take a white dwarf 1.5 times mass of the Sun. As it cools, it shrinks, and the force of gravity increases. The electrostatic force cannot hold the atoms apart, and it will collapse into a neutron star (http://en.wikipedia.org/wiki/Neutron_star). But this effect does not occur for objects the mass of the Earth.
Title: Re: Why don't an atom's electrons fall into the nucleus and stick to the protons?
Post by: jccc on 18/02/2015 18:34:58
happy now?  i hope so.

light is atom's gravity wave   

Title: Re: Why don't an atom's electrons fall into the nucleus and stick to the protons?
Post by: chiralSPO on 18/02/2015 18:59:19
happy now?  i hope so.

light is atom's gravity wave

That makes no sense whatsoever. Light is an electromagnetic wave. The energy of of an atom's gravitational field is miniscule compared to the energy carried by photons of light emitted by atoms.
Title: Re: Why don't an atom's electrons fall into the nucleus and stick to the protons?
Post by: PmbPhy on 19/02/2015 16:57:34
happy now?  i hope so.

light is atom's gravity wave

That makes no sense whatsoever.
Of course. Consider the source.
Title: Re: Why don't an atom's electrons fall into the nucleus and stick to the protons?
Post by: jccc on 27/02/2015 13:36:02
an atom has mass m, vibrates at f times per second, it will seed mfc^2/t energy/force out as gravity wave. the nature of the force wave is electrostatic force.

light is not wave nor particle, light is gravitational pause produced by exited atoms.

Science has never been absolute, what makes you think today's science is all correct? In fact, science is so fucked up. Present theories of atomic structure, gravity and light are all wrong.

 
Title: Re: Why don't an atom's electrons fall into the nucleus and stick to the protons?
Post by: jeffreyH on 27/02/2015 14:34:42
an atom has mass m, vibrates at f times per second, it will seed mfc^2/t energy/force out as gravity wave. the nature of the force wave is electrostatic force.

light is not wave nor particle, light is gravitational pause produced by exited atoms.

Science has never been absolute, what makes you think today's science is all correct? In fact, science is so f***ed up. Present theories of atomic structure, gravity and light are all wrong.

What you are trying to say here with this mfc^2/t is that energy multiplied by the frequency of phonon vibrations over time causes gravity. In the case where the phonon vibrations are not regular your energy source will be pulsating and some of this energy is released as gravity. The problems are 1) your mass would be gone very quickly and 2) you would need to prove that all phonon vibrations are irregular. I know energy is a difficult concept but I have never heard of anyone using phonons in this way. What may be possible is that the vibrational energy causes ripples (waves) in the gravitational field without loss of energy. The problem with this is tying it to a force that is all attractive. I have no idea where you would even start on that one so good luck I await your results. I may be waiting an awfully long time.
Title: Re: Why don't an atom's electrons fall into the nucleus and stick to the protons?
Post by: jccc on 27/02/2015 14:51:18
we need to start from the light source. if atoms are like qm suggested, 99% empty space, why is water/matter not compressible? why there is no discharge between proton and electron? obviously the empty space is not empty.

Title: Re: Why don't an atom's electrons fall into the nucleus and stick to the protons?
Post by: PmbPhy on 27/02/2015 15:21:28
Quote from: jccc
we need to start from the light source. if atoms are like qm suggested, 99% empty space, why is water/matter not compressible?
It is compressible. See:
http://en.wikipedia.org/wiki/Properties_of_water#Compressibility
Title: Re: Why don't an atom's electrons fall into the nucleus and stick to the protons?
Post by: jccc on 27/02/2015 15:43:14
Quote from: jccc
we need to start from the light source. if atoms are like qm suggested, 99% empty space, why is water/matter not compressible?
It is compressible. See:
http://en.wikipedia.org/wiki/Properties_of_water#Compressibility
water's compressibility is about 10 ^-10, sounds like 99% empty space to you?

how about the discharge? is the empty space such a good insulator?
Title: Re: Why don't an atom's electrons fall into the nucleus and stick to the protons?
Post by: PmbPhy on 27/02/2015 16:49:40
Quote from: jccc
water's compressibility is about 10 ^-10, sounds like 99% empty space to you?

how about the discharge? is the empty space such a good insulator?
I answered your question about the compressibility of water, not about how much empty space there is. They are not directly related. And the amount of space taken up can be thought of in two ways; one in which one only considers the volume of the particles and one in which one considers the atom as a whole, i.e. the space in which the electron cloud exists. In the first case the volume of the particles is a great deal less than 1%.
Title: Re: Why don't an atom's electrons fall into the nucleus and stick to the protons?
Post by: jccc on 27/02/2015 17:03:27
Quote from: jccc
we need to start from the light source. if atoms are like qm suggested, 99% empty space, why is water/matter not compressible?
It is compressible. See:
http://en.wikipedia.org/wiki/Properties_of_water#Compressibility
water's compressibility is about 10 ^-10, sounds like 99% empty space to you?

how about the discharge? is the empty space such a good insulator?
what's your answer?
Title: Re: Why don't an atom's electrons fall into the nucleus and stick to the protons?
Post by: PmbPhy on 28/02/2015 02:36:57
Quote from: jccc
what's your answer?
I don't have an answer. Who ever said I know everything!
Title: Re: Why don't an atom's electrons fall into the nucleus and stick to the protons?
Post by: jccc on 28/02/2015 02:43:18
thank you Pete!

all my respect to you.

Title: Re: Why don't an atom's electrons fall into the nucleus and stick to the protons?
Post by: PmbPhy on 28/02/2015 03:17:57
thank you Pete!

all my respect to you.
You're welcome.  [^]
Title: Re: Re: The Pseudosciences
Post by: alancalverd on 28/02/2015 15:27:14
We do know all those things. You just don't understand the mathematics.

It's not unusual to find incomprehension masquerading as superior knowledge - that is, after all, the basis of many religions.
Title: Re: Re: The Pseudosciences
Post by: jccc on 28/02/2015 15:30:23
We do know all those things. You just don't understand the mathematics.

It's not unusual to find incomprehension masquerading as superior knowledge - that is, after all, the basis of many religions.

what thing do you know? why water is not compressible if there is 99% empty space in atoms?
Title: Re: Re: The Pseudosciences
Post by: PmbPhy on 28/02/2015 15:51:40
Quote from: jccc
you don't even understand how atoms are formed, .you don't know why is electron not stick to proton, you don't know why water is not compressible, you don;t know why there is no discharge in atoms,
When I said that I don't know what "how about the discharge? is the empty space such a good insulator?" means, no physicists has all that kind of data in their minds. When we need to know something we either calculate it or look it up. Which ever is the most convenient.

Title: Re: Why don't an atom's electrons fall into the nucleus and stick to the protons?
Post by: jccc on 28/02/2015 18:38:19
a matter/mass/charge has force/gravitational field, if the matter moves, the field follows. if the matter vibrates, the field follows.

if the sun has a big ejection/quake, we should be able to detect it 500 seconds later. we might even have a earthquake.

atoms, each has mass, when atoms exited, their fields exited, within that field, you feel light.
Title: Re: Re: The Pseudosciences
Post by: David Cooper on 28/02/2015 19:12:56
what thing do you know? why water is not compressible if there is 99% empty space in atoms?

Water is compressible - it just takes an extremely high force to compress it to a significant degree.
Title: Re: Re: The Pseudosciences
Post by: jccc on 28/02/2015 19:16:49
what thing do you know? why water is not compressible if there is 99% empty space in atoms?

Water is compressible - it just takes an extremely high force to compress it to a significant degree.

right, 10^-10. why? if it is 99% empty. what force canceled pressure?
Title: Re: Re: The Pseudosciences
Post by: David Cooper on 28/02/2015 20:33:57
It isn't just water. Think about rock, metal, wood, etc.

With a gas, it's easy to comress it because the molecules start far apart, and the forces keeping them apart come from impact collision energy (temperature). In water you don't have that situation because the molecules are packed tightly together already, so to push them closer together you have to push against the forces that hold atoms apart. I don't understand how atoms are held apart at the distance they are, but a simple way of understanding how two things can in principle maintain a fixed separation would be to imagine two forces, one pulling two items together and the other pushing the two items apart, but with the one pushing apart decaying more quickly over distance. This results in strong repulsion when they get close together, but there will be a distance apart where the two items attract and repel each other equally, and that's where they will naturally sit. It's easier to pull them apart from there than it is to push them closer together, so breaking them apart can be done with relatively low force while trying to compress them is extremely hard and gets enormously harder for each extra bit of compression.

[Note: I have not posted to this thread at all, but my posts (along with some others around them) were moved here from a different thread as they were part of an off-topic conversation.]
Title: Re: Re: The Pseudosciences
Post by: alancalverd on 28/02/2015 22:36:13
what thing do you know? why water is not compressible if there is 99% empty space in atoms?

Water is compressible - it just takes an extremely high force to compress it to a significant degree.

right, 10^-10. why? if it is 99% empty. what force canceled pressure?

What makes you think it's 99% empty? In my glass, it's 100% water. Your failure to understand molecular orbital energetics is sad, but by no means unique. That's what distinguishes scientists from the ineducable.
Title: Re: Re: The Pseudosciences
Post by: jccc on 28/02/2015 22:56:13
what thing do you know? why water is not compressible if there is 99% empty space in atoms?

Water is compressible - it just takes an extremely high force to compress it to a significant degree.

right, 10^-10. why? if it is 99% empty. what force canceled pressure?

What makes you think it's 99% empty? In my glass, it's 100% water. Your failure to understand molecular orbital energetics is sad, but by no means unique. That's what distinguishes scientists from the ineducable.

what is water? h2o, 3 atoms together, each atom has a nucleus, has electron orbitals, and 99.99% empty space. isn't that's the standard model?

https://www.google.com/
search?q=atom+standard+model&espv=2&biw=1242&bih=606&tbm=isch&tbo=u&source=univ&sa=X&ei=
Title: Re: Re: The Pseudosciences
Post by: jeffreyH on 28/02/2015 23:08:13
Pseudoscience is disputing established science with claims that do not stand up to scrutiny. I think it's on topic. It exemplifies the lack of understanding that leads to pseudoscience. It can also be tied to a lack of understanding of the history and philosophy of science. This is a typical case. If jccc thinks it is extraordinary that so much space could exist in atomic structure then the orbital distance between the sun and neptune must also be wrong.
Title: Re: Re: The Pseudosciences
Post by: jccc on 28/02/2015 23:17:03
Pseudoscience is disputing established science with claims that do not stand up to scrutiny. I think it's on topic. It exemplifies the lack of understanding that leads to pseudoscience. It can also be tied to a lack of understanding of the history and philosophy of science. This is a typical case. If jccc thinks it is extraordinary that so much space could exist in atomic structure then the orbital distance between the sun and neptune must also be wrong.

any solar systems share planets? solar systems are light years apart. atoms are packed together. solar system works on gravity. atom system works on electrostatic force, 10^40g attraction between proton and electrons, what force keeps them apart?
Title: Re: Re: The Pseudosciences
Post by: alancalverd on 01/03/2015 00:58:28
If you have any pretensions to being educable, the first step to understanding quantum physics is to realise that whilst you can derive classical physics from quantum mechanics, you can't derive quantum mechanics from classical physics. You have to set aside all your preconceptions and accept the universe as it is, not as you would like it to be.   

Wisdom grows from humilty, not arrogance.
Title: Re: Re: The Pseudosciences
Post by: jccc on 01/03/2015 01:35:04
qm what? you can't even answer any questions i asked.

isn't standard model says there is 99.99% empty space within atoms? how empty space stands any pressure?

how many volts is in between proton and electron in a hydrogen atom? why there is no discharge? is the empty space such a good insulator?
Title: Re: Re: The Pseudosciences
Post by: alancalverd on 01/03/2015 08:02:01

isn't standard model says there is 99.99% empty space within atoms? how empty space stands any pressure?


I have a tent which is 99.9% empty space but still stands up in the wind. You need to start from observation and develop your model to fit the truth. Or you could buy into quantum mechanics and use the model that others have developed over the last century or so.

And don't forget Eddington's dicta:

The student of physics must become accustomed to having his common sense violated five times before breakfast.

If a physicist fell through the floor and rematerialised downstairs, he would consider it no more than an extremely improbable event.

....which would be merely amusing if it didn't actually happen, which makes it interesting and useful.

Title: Re: Re: The Pseudosciences
Post by: jccc on 01/03/2015 08:14:17

isn't standard model says there is 99.99% empty space within atoms? how empty space stands any pressure?


I have a tent which is 99.9% empty space but still stands up in the wind. You need to start from observation and develop your model to fit the truth. Or you could buy into quantum mechanics and use the model that others have developed over the last century or so.

And don't forget Eddington's dicta:

The student of physics must become accustomed to having his common sense violated five times before breakfast.

If a physicist fell through the floor and rematerialised downstairs, he would consider it no more than an extremely improbable event.

....which would be merely amusing if it didn't actually happen, which makes it interesting and useful.

your tent has frame to support it. how much pressure can a tent support?
Title: Re: Re: The Pseudosciences
Post by: alancalverd on 01/03/2015 08:33:36
You are getting there. Abandon the Bohr atom (most of us did so in the 1950s) and think orbitals. 
Title: Re: Re: The Pseudosciences
Post by: evan_au on 01/03/2015 10:24:17
Quote
isn't standard model says there is 99.99% empty space within atoms?
I think this is being quoted backwards.

The standard model says that the nucleus is about 1/100,000 of the radius of the atom (eg: Hydrogen radius: 120pm; proton radius: 0.87 fm).

But the electron is not a point particle; it is not even a particle with a fairly well-defined radius (like the proton). It does have some wave-like or probabilistic properties, which smears it out over the entire volume of the atom.

So the atom volume is not empty - it is filled with electron(s).

Quote
how empty space stands any pressure?
The volume of an atom is filled with the negative electron(s), and this electron will repel the negative electron(s) of any atom approaching closer than a specified distance. This distance may be empirically determined by X-Ray diffraction, deviations from the Ideal Gas rule at high pressures, etc.

In principle, this length could also be calculated from the Schrodinger wave equation, but it gets horribly complex for molecules larger than a Hydrogen atom (ie all of them).

Electrostatic repulsion withstands considerable pressure.
Title: Re: Why don't an atom's electrons fall into the nucleus and stick to the protons?
Post by: jccc on 01/03/2015 12:06:23
Electrostatic repulsion withstands considerable pressure.

what repulsion is between nucleus and electrons? i only can see attraction there.
Title: Re: Why don't an atom's electrons fall into the nucleus and stick to the protons?
Post by: alancalverd on 01/03/2015 16:43:43
You can't make an egg from an omelette, and you can't predict quantum phenomena from classical electrostatics.

You have to start with the fact that atoms are finite, construct mathematical models consistent with that observation, and see if they can predict other observed phenomena, including classical (continuum, mesoscopic) physics. To everyone's delight, quantum mechanics does this very well, which is why we use it.
Title: Re: Why don't an atom's electrons fall into the nucleus and stick to the protons?
Post by: jccc on 01/03/2015 16:56:05
you use it because you have nothing better to use.

so far you can't use it to answer any of my questions on the last 2 pages.

i explained gravity, light and atomic structure, what's your debunk?

can you proof that light is not gravitational wave produced by exited atoms?

Title: Re: Re: The Pseudosciences
Post by: JohnDuffield on 01/03/2015 17:27:02
why? if it is 99% empty. what force canceled pressure?
See Einstein's Leyden address (http://www-history.mcs.st-and.ac.uk/Extras/Einstein_ether.html) where you can read this: "recognition of the fact that 'empty space' in its physical relation is neither homogeneous nor isotropic, compelling us to describe its state by ten functions (the gravitation potentials gmn), has, I think, finally disposed of the view that space is physically empty". Space isn't empty. You can curve it and stress it. When you compress a spring, the energy is stored in the bonds between the atoms. It's effectively stored in the space between the atoms. And mindful of what evan said, when you dig down deep and look at an electron, it's "just field". It's the same for everything else. That's why it's quantum field theory. Yes the proton has this three-part central structure, but its electromagnetic field is part of what it is. And you can read Einstein describing a field as a state of space (http://www.rain.org/~karpeles/einsteindis.html). So atoms aren't 99% empty space. They're 100% space! 
Title: Re: Re: The Pseudosciences
Post by: JohnDuffield on 01/03/2015 17:39:47
is his fart sweet to you?
No. It's just that I'm something of a "relativist", I've read a lot of Einstein's material, there's some interesting stuff in there. Like space is this ghostly gin-clear elastic that you can curve and stress. Hence the stress-energy-momentum tensor (http://en.wikipedia.org/wiki/Stress%E2%80%93energy_tensor) "describes the density and flux of energy and momentum in spacetime". Note the shear stress term. 

did he said if you can't explain it to a six years old, you don't understand it?
He is supposed to have said "'You do not really understand something unless you can explain it to your grandmother"
Title: Re: Why don't an atom's electrons fall into the nucleus and stick to the protons?
Post by: jccc on 01/03/2015 19:26:11
i wondered about why laser is pointing for so long. now if laser is gravitational wave produced by all atoms vibrate at same direction, of cause it is pointing. expends a little due to medium/air on the way. the force is not pushing or puling but vibrating, energy able to transfer without contact.

i want to take the time thank this awesome forum, and every dear member. when i joined i have little clue, now i learned a lot science. everyone is my teacher in some way. Thank You! forever.
Title: Re: Why don't an atom's electrons fall into the nucleus and stick to the protons?
Post by: evan_au on 01/03/2015 20:29:43
Quote from: jccc
can you proof that light is not gravitational wave produced by excited atoms?
Maxwell (http://en.wikipedia.org/wiki/Maxwell%27s_equations)* showed that light is a transverse electromagnetic wave. We see this confirmed in practice, as we see that the speed of light is affected by εr (which affects the electric field) and μr (which affects the magnetic field), when it passes through some material like glass.

At the scale of a single atom (electrons + nucleus), the electric field is far stronger than the gravitational forces. So changes in electron orbitals can store and release far more electromagnetic energy than they can store and release gravitational energy. The energy of light (as measured by the photo-electric effect) agrees with the energy of photons being electromagnetic, rather than gravitational.

We have good astronomical evidence that gravitational waves do exist, but to be big enough to measure, they require extreme conditions like two closely orbiting neutron stars (http://en.wikipedia.org/wiki/PSR_B1913%2B16). (Much more massive than an orbiting electron & nucleus!) Scientists have spent over $US300M on making detectors that are sensitive enough to detect the existence of gravitational waves on Earth (eg see LIGO (http://en.wikipedia.org/wiki/LIGO)). So far, they have not been sensitive enough.

So the solar cells on many peoples roofs, costing $USthousands, and generating useful amounts of power are a clear demonstration that light is an electromagnetic phenomenon, and not a gravitational phenomenon.

*By accident, I discovered a statue of James Clerk Maxwell, while wandering the streets of Edinburgh. The base is inscribed with his famous equations - but hidden around the back, presumably because they may scare the general public?
See: http://upload.wikimedia.org/wikipedia/commons/9/9c/James_Clerk_Maxwell_statue,_George_Street_Edinburgh.jpg
Title: Re: Re: The Pseudosciences
Post by: PmbPhy on 01/03/2015 20:30:13
Quote from: JohnDuffield
He is supposed to have said "'You do not really understand something unless you can explain it to your grandmother"
Wrong. I checked with an Einstein historian and was told that it's a myth.
Title: Re: Why don't an atom's electrons fall into the nucleus and stick to the protons?
Post by: PmbPhy on 01/03/2015 20:37:36
Quote from: jccc
can you proof that light is not gravitational wave produced by exited atoms?
That's easy. First of all if it was then the gravitational wave detector LIGO would be detecting gravitational waves all the time. Light is an electromagnetic wave which means that it's not a gravitational wave. An electromagnetic wave interacts differently with matter than does gravitational waves to. We can see light but we can't see gravitational waves. Light waves have different sources than gravitational waves do. If the source of a gravitational wave was increased but not that for light then the light intensity wouldn't change but that for the gravitational wave would. etc.

Everything that goes into the theory of electrodynamics is different than that which goes into gravitation and/or GR so if you knew both of these theories like Evan and myself then there'd be no question about it.
Title: Re: Why don't an atom's electrons fall into the nucleus and stick to the protons?
Post by: jccc on 01/03/2015 21:25:37
are you sure? our eyes detect gravity wave all the time. the source/atom has to vibrate at high f to produce visible light. only atoms able to vibrate at high f. a little ball weights 1g will take a lot energy to vibrate at such high speed.

if you put a bell in vacuum, put gravity wave detector in the center, knock the bell and you should detect gravity wave.

the wave is electrostatic force in nature, so we call it em wave?
Title: Re: Why don't an atom's electrons fall into the nucleus and stick to the protons?
Post by: jeffreyH on 01/03/2015 22:33:51
jccc why do you have this fixation on electromagnetism being a source of gravity? Show us some real evidence. Amaze us! That is exactly what would happen if you could demonstrate the validity of such an idea. However the bar is high and any theory would need peer review. It's a lot of work. Are you capable of providing such evidence?
Title: Re: Why don't an atom's electrons fall into the nucleus and stick to the protons?
Post by: jccc on 01/03/2015 22:39:34
did you read page 12 and 13? have any questions?

start from atomic structure, if H atom is build by electron and proton only with 99% empty space, why there is no discharge? why matter is not compressible?

Title: Re: Why don't an atom's electrons fall into the nucleus and stick to the protons?
Post by: alancalverd on 01/03/2015 22:41:07


can you proof that light is not gravitational wave produced by exited atoms?



A gravitational wave would be detectable on the exit side of a thin sheet of metal.
Title: Re: Why don't an atom's electrons fall into the nucleus and stick to the protons?
Post by: alancalverd on 01/03/2015 22:45:29
did you read page 12 and 13? have any questions?

start from atomic structure, if H atom is build by electron and proton only with 99% empty space, why there is no discharge? why matter is not compressible?



A big "if", and wrong. As you say, you have to start with atomic structure and develop a physical model that explains it. Pretending that it can't happen isn't allowed, because it does happen. So your chosen model is wrong because it doesn't explain the obvious facts. You can't derive quantum mechanics from classical electrostatics.   
Title: Re: Why don't an atom's electrons fall into the nucleus and stick to the protons?
Post by: jccc on 01/03/2015 22:52:05


can you proof that light is not gravitational wave produced by exited atoms?



A gravitational wave would be detectable on the exit side of a thin sheet of metal.

solar cells transfer gravity wave into current all the time. green leaves transfer gravity wave into food.
Title: Re: Why don't an atom's electrons fall into the nucleus and stick to the protons?
Post by: jeffreyH on 01/03/2015 22:58:10
If opposite charges are meant to combine then why does the neutron decay? I have deja vu here.
Title: Re: Why don't an atom's electrons fall into the nucleus and stick to the protons?
Post by: jccc on 01/03/2015 23:04:01
If opposite charges are meant to combine then why does the neutron decay? I have deja vu here.

why people got married? never seen divorce?
Title: Re: Why don't an atom's electrons fall into the nucleus and stick to the protons?
Post by: jeffreyH on 01/03/2015 23:09:47
If opposite charges are meant to combine then why does the neutron decay? I have deja vu here.

why people got married? never seen divorce?

You aren't here to learn.
Title: Re: Why don't an atom's electrons fall into the nucleus and stick to the protons?
Post by: jccc on 02/03/2015 01:27:24
the most beautiful thing in the universe is not mystery, but your mind.

without mind, there's nothing.
Title: Re: Why don't an atom's electrons fall into the nucleus and stick to the protons?
Post by: alancalverd on 02/03/2015 08:52:35


solar cells transfer gravity wave into current all the time. green leaves transfer gravity wave into food.

Repeating an obvious untruth won't make it true, or even amusing.
Title: Re: Why don't an atom's electrons fall into the nucleus and stick to the protons?
Post by: jccc on 02/03/2015 11:07:39


solar cells transfer gravity wave into current all the time. green leaves transfer gravity wave into food.

Repeating an obvious untruth won't make it true, or even amusing.

are you talking about qm? graviton or photon? I told you long ago.
Title: Re: Re: The Pseudosciences
Post by: JohnDuffield on 02/03/2015 14:42:42
Quote from: JohnDuffield
He is supposed to have said "'You do not really understand something unless you can explain it to your grandmother"
Wrong. I checked with an Einstein historian and was told that it's a myth.
That's why I said supposed.
Title: Re: Why don't an atom's electrons fall into the nucleus and stick to the protons?
Post by: Ophiolite on 03/03/2015 08:14:44
You can only claim an atom is >99% empty space if you insist that electrons each exist at a single point. They don't. They exist as a probability cloud. Those clouds interact in various ways and are responsible for the lack of compression you think should be present.

Title: Re: Why don't an atom's electrons fall into the nucleus and stick to the protons?
Post by: jccc on 03/03/2015 09:43:06
i claim nothing.

i ask if standard atomic model is correct, why matter is not compressible? why there is no discharge within atoms?

obviously atom is not 99% empty space. the space between nucleus and electrons must have a force field that balances the attraction so electron able to stay floating.

haven't see any legit answer, do you have 1?
Title: Re: Why don't an atom's electrons fall into the nucleus and stick to the protons?
Post by: alancalverd on 03/03/2015 12:05:14
You are still asking the question backwards. Here is the modern formulation:

The atom has a finite diameter. From what we know of its structure, can we derive classical electrostatics?

The answer is yes. Quantum mechanics (which provides an accurate description of atomic structure) degenerates to classical electrostatics (an adequate approximation of the behaviour of charged bodies) when the separation between charged bodies is large. 

Beware of using outdated models in science. Not everyone understands them anymore, and some may think you are being obtuse..
Title: Re: Why don't an atom's electrons fall into the nucleus and stick to the protons?
Post by: jccc on 03/03/2015 13:35:22
you are at it again.

why is matter not compressible? why is no discharge within atoms? if stand model is correct?
Title: Re: Why don't an atom's electrons fall into the nucleus and stick to the protons?
Post by: jccc on 03/03/2015 14:06:15
Pete is the only 1 has the merit to admit he has no answer to my questions.

you act like a kid of science. Please remember everything you posted in this thread will be reading by many.

ever thought how your students will think about your postings?
Title: Re: Why don't an atom's electrons fall into the nucleus and stick to the protons?
Post by: David Cooper on 03/03/2015 17:42:00
why is matter not compressible?

It is compressible - it just puts up a very good fight, though it doesn't do so well in neutron stars, and it loses comprehensively in black holes.
Title: Re: Why don't an atom's electrons fall into the nucleus and stick to the protons?
Post by: jccc on 03/03/2015 17:48:43
if atom is like alan's tent, give 100 atm alan will become a meat ball.
Title: Re: Why don't an atom's electrons fall into the nucleus and stick to the protons?
Post by: alancalverd on 03/03/2015 18:02:52
if stand model is correct?

For the third and final time, what you call the "standard model" is WRONG for the very reason you give. Please take a deep breath and give your brain a chance to recover!

If a hypothesis does not fit the facts, the hypothesis is wrong. In the case of the atom, everyone else, from the dumbest schoolboy to several Nobel prizewinners,  recognised this about 100 years ago which is why we have quantum mechanics instead of classical electron orbits.

This is a science board. Scientists abandon models that don't work, so don't expect anyone here to play with your outdated and broken toys. I'm sure there are boards where people happily repeat nonsense ad infintum in the hope that someone will take them seriously, and I'm sure you would be a welcome guest.
Title: Re: Why don't an atom's electrons fall into the nucleus and stick to the protons?
Post by: jccc on 03/03/2015 18:22:53
you are kidding yourself again.

can you debunk any thing i posted about gravity, atomic structure and light? what are you waiting for?
Title: Re: Why don't an atom's electrons fall into the nucleus and stick to the protons?
Post by: PmbPhy on 03/03/2015 20:15:29
Quote from: alancalverd
For the third and final time, what you call the "standard model" is WRONG for the very reason you give.
I missed what he said and your response. Are you saying that the "standard model" as he thinks of it is wrong or are you saying that the actual standard model is wrong and if so, why?

By "Standard Model" I mean the one defined here: http://en.wikipedia.org/wiki/Standard_Model
Title: Re: Why don't an atom's electrons fall into the nucleus and stick to the protons?
Post by: Ethos_ on 03/03/2015 21:00:54
you are kidding yourself again.

can you debunk any thing i posted about gravity, atomic structure and light? what are you waiting for?
It's not our responsibility to prove you wrong, it's up to you to show us evidence in support of your theory. So far, little if any has been presented.

It isn't good enough to simply challenge someone to prove a negative. If you seek acceptance, you will be required to provide your own evidence. Saying "prove me wrong" will get you nowhere.
Title: Re: Why don't an atom's electrons fall into the nucleus and stick to the protons?
Post by: jccc on 03/03/2015 22:33:30
you are kidding yourself again.

can you debunk any thing i posted about gravity, atomic structure and light? what are you waiting for?
It's not our responsibility to prove you wrong, it's up to you to show us evidence in support of your theory. So far, little if any has been presented.

It isn't good enough to simply challenge someone to prove a negative. If you seek acceptance, you will be required to provide your own evidence. Saying "prove me wrong" will get you nowhere.

did we had encounter at page 10? read it again.
Title: Re: Why don't an atom's electrons fall into the nucleus and stick to the protons?
Post by: alancalverd on 03/03/2015 23:12:57
Quote from: alancalverd
For the third and final time, what you call the "standard model" is WRONG for the very reason you give.
I missed what he said and your response. Are you saying that the "standard model" as he thinks of it is wrong or are you saying that the actual standard model is wrong and if so, why?

By "Standard Model" I mean the one defined here: http://en.wikipedia.org/wiki/Standard_Model

I have no idea what he calls the standard model, but if it predicts that the electrons will fall into the nucleus, it's obviously wrong because they don't. Or if it doesn't but he thinks it does, he's an idiot. Either way, there's little point in discussing the matter with him.
Title: Re: Why don't an atom's electrons fall into the nucleus and stick to the protons?
Post by: PmbPhy on 03/03/2015 23:30:22
Quote from: alancalverd
I have no idea what he calls the standard model, ...
The standard model has less to do with atoms and more to do with interactions. Essentially the Standard Model is the theory that describes all of the known elementary particle interactions except gravity.

Quote from: alancalverd
... but if it predicts that the electrons will fall into the nucleus, it's obviously wrong because they don't.
For the most part you're correct. However that is not always the case. For example; In one of the quantum states of the hydrogen atom, the electron has a finite probability of being found inside the nucleus. See
http://physicspages.com/2013/01/23/hydrogen-atom-probability-of-finding-electron-inside-the-nucleus/

This is possible because is some of the quantum states that hydrogen can be in the wave function is not zero at r = 0.

Don't mention this to jccc because he'll just start all over again with his puerile talk of magic. Lol.
Title: Re: Why don't an atom's electrons fall into the nucleus and stick to the protons?
Post by: alancalverd on 03/03/2015 23:39:16


can you debunk any thing i posted about gravity, atomic structure and light? what are you waiting for?

See #326 above.
Title: Re: Why don't an atom's electrons fall into the nucleus and stick to the protons?
Post by: jccc on 03/03/2015 23:49:34
that's cool.

for the sake of science, i became pest, amoeba, virus, idiot, whatever you like.

i understood gravity, light and atomic structure, and magnetism.

my dreams are completed.

Thank You All!
Title: Re: Why don't an atom's electrons fall into the nucleus and stick to the protons?
Post by: jccc on 06/03/2015 00:59:57
To transmogrify into a butterfly you must undergo a metamorphosis.

on a more serious note how do electrons survive floating in this negative charged eather

hit and run?
Title: Re: Why don't an atom's electrons fall into the nucleus and stick to the protons?
Post by: Ethos_ on 06/03/2015 16:31:32
this forum deleted some of my postings
Appropriate action considering the content.
Quote from: jccc
maybe soon will ban my account
Might also be an appropriate action considering your behavior.
Quote from: jccc
i recorded everything i posted
That shouldn't require more than a bit or two.
Quote from: jccc
find true science at fuckedscience.com
LOL
Quote from: jccc
if you see me here no more
Please.....................
Title: Re: Why don't an atom's electrons fall into the nucleus and stick to the protons?
Post by: jccc on 07/03/2015 02:35:32
To transmogrify into a butterfly you must undergo a metamorphosis.

on a more serious note how do electrons survive floating in this negative charged eather

so far so good? thank you for inspiring me putting ideas out.

if atomic structure theory was wrong, all theories about matter could be wrong. what is mass if matter carries no charge? if matter has no charge/force, how you measure it?

maybe proton charge is not +1, the proton and fluid ball combined net charge is +1. we can never see a single proton or neutron, they all surrounded by fluid ball like a solid rock.

maybe the size of proton and neutron are 1800 electron size, when they passing mass spectrometer, the space fluid inside the tube puts resistance on them, the bigger ball curves more. just like shoot two beach balls horizontally, the bigger ball drops faster. ???

mass equal to matter's force field strength.  a gold ball contains more charges therefore heavier than a silver ball. when the ball moving in space, the resistance following speed, the faster you go the heavier you are.

need sleep, later.

here's the missing link.

the negative charged elastic fluid attracted by positive nucleus, form a negative field around the nucleus. density =1/r^3. away from the nucleus to a point, the density no longer decay. that's the background charge of the space. electron has a force radius f=1/r^2, off that radius, electron has the same negative charge density as the space around it.

proton may carries +1800, attracted -1799 fluid to form hydrogen nucleus, add 1 electron on the radius to become hydrogen atom.

the space fluid maybe used up, maybe a little left in space. i haven't thought it through.
Title: Re: Why don't an atom's electrons fall into the nucleus and stick to the protons?
Post by: jccc on 15/03/2015 08:31:27
maybe proton carries 900+, attracted 899- fluid to form nucleus, add 1 electron to form hydrogen.

the rest fluid maybe the source of dm/de?

atomic structure has to be 100% correct, otherwise whole science is doubtful.

any thoughts?
Title: Re: Why don't an atom's electrons fall into the nucleus and stick to the protons?
Post by: alancalverd on 15/03/2015 10:03:34
Atomic structure is 100% correct, otherwise we wouldn't be here. Your deliberate misunderstanding of it is at fault.
Title: Re: Why don't an atom's electrons fall into the nucleus and stick to the protons?
Post by: jccc on 15/03/2015 13:07:48
Atomic structure is 100% correct, otherwise we wouldn't be here. Your deliberate misunderstanding of it is at fault.

isn't standard model says there is 99.99% empty space within atoms? how empty space stands any pressure?

how many volts is in between proton and electron in a hydrogen atom? why there is no discharge? is the empty space such a good insulator?
Title: Re: Why don't an atom's electrons fall into the nucleus and stick to the protons?
Post by: jccc on 15/03/2015 14:41:34
maybe proton carries 900+, attracted 899- fluid to form nucleus, add 1 electron to form hydrogen.

the rest fluid maybe the source of dm/de?

atomic structure has to be 100% correct, otherwise whole science is doubtful.

any thoughts?

i am thinking when they collide proton beams in the lab, they mistake thinking proton is a point particle, but in fact they are colliding proton with fluid ball beams. the fluid explode, so they detect all kinds of new particles.

it's a reasonable doubt, no?

have a great new week! 
Title: Re: Why don't an atom's electrons fall into the nucleus and stick to the protons?
Post by: PmbPhy on 15/03/2015 16:05:11
Quote from: jccc
i am thinking when they collide proton beams in the lab, they mistake thinking proton is a point particle, but in fact they are colliding proton with fluid ball beams. the fluid explode, so they detect all kinds of new particles.

it's a reasonable doubt, no?
No. It's not a reasonable doubt. These people are significant experts at what they do. They've devoted their entire lives to these endeavors. What makes you think that you, with no education or experience, could think you caught something that the entire particle physics community missed? Particle physicists have probed the interior of the proton using deep elastic scattering. They found that it's composed of three lumps of charge. Want to guess why?
Title: Re: Why don't an atom's electrons fall into the nucleus and stick to the protons?
Post by: jccc on 15/03/2015 16:14:20
Quote from: jccc
we need to start from the light source. if atoms are like qm suggested, 99% empty space, why is water/matter not compressible?
It is compressible. See:
http://en.wikipedia.org/wiki/Properties_of_water#Compressibility
water's compressibility is about 10 ^-10, sounds like 99% empty space to you?

how about the discharge? is the empty space such a good insulator?
what's your answer?
[/quote]
Quote from: jccc
what's your answer?
I don't have an answer. Who ever said I know everything!
[/quote]

do you have answer now? Pete

do you think people work at lhc have the answer?
Title: Re: Why don't an atom's electrons fall into the nucleus and stick to the protons?
Post by: jccc on 15/03/2015 16:17:37
talking about expert, how about einstein? is photon a real thing? is light a particle?

if photon is none existence, isn't his nobel prize a joke?
Title: Re: Why don't an atom's electrons fall into the nucleus and stick to the protons?
Post by: chiralSPO on 15/03/2015 16:24:52


isn't standard model says there is 99.99% empty space within atoms? how empty space stands any pressure?

No. The the current widely accepted model is that the nucleus takes up ~0.01% of the volume of the atom, and the electrons take up the remaining 99.99%. We explain over and over--the electrons behave as waves, and they are spread out through the entire space of the atom.

Even if you want to use a model in which the electrons are point particles whizzing about within the atom, they would still exert/withstand significant pressure. The air around us (or any gas at reasonable pressures) is about 90% "empty space" (the volume of the molecules is a really tiny fraction of the total volume of the gas--for instance a nitrogen molecule has a van der Waals volume of about 65 Å3 or 6.5 x 10–27 m3. One mole of nitrogen molecules would have a collective volume of 6.02 x 1023 x 6.5 x 10–27 or 3.9 x 10–3 m3; but at standard atmospheric pressure and temperature, the gas fills a volume of 2.2 x 10–2 m3--therefore, if we assume air is only nitrogen, it would be about 82% "empty".

The key here is that the molecules are whizzing around really quickly, so *on average* every point of space is filled with .18 molecules, even though at any given instant only 18% of points have a nitrogen molecule and 82% have none.

how many volts is in between proton and electron in a hydrogen atom? why there is no discharge? is the empty space such a good insulator?

Again, you are confusing macroscopic and microscopic phenomena. "Discharge" as commonly understood is flow of electrons or flow of charged ions. There couldn't possibly be discharge from an electron--it would have to be the electron itself moving. But! As we have already pointed out many, many, many times in this thread: the electron is already centered about the nucleus, and cannot "get any closer" because it naturally spreads out to take up the whole volume of the atom.


We understand that this is a very tricky and counter-intuitive subject, but the experimental and theoretical evidence is very clear--electrons in atoms and molecules are best described as waves, and once you accept that premise, everything else falls into place very nicely.
Title: Re: Why don't an atom's electrons fall into the nucleus and stick to the protons?
Post by: jccc on 15/03/2015 16:37:13
electrons in atoms and molecules are best described as waves, and once you accept that premise, everything else falls into place very nicely.

what wave? how electron waves? standing wave? is the waving electron still carry negative charge? if so why is it not stick to the proton?

there is no premise, no logic. no?
Title: Re: Why don't an atom's electrons fall into the nucleus and stick to the protons?
Post by: chiralSPO on 15/03/2015 16:47:49
The electron is still negative. It's just easier to think of it as a wave than a particle in these cases.

I would argue that an atom is an example of an electron stuck to the nucleus--it's just that the electron takes up far more space than the nucleus. It takes energy to remove the electron from the atom, energy is released when the electron is captured by the positive ion (for example both H+ + e → H or Au+ + e → Au release energy)
Title: Re: Why don't an atom's electrons fall into the nucleus and stick to the protons?
Post by: jccc on 15/03/2015 17:10:30
because electron and proton are attracting each other, so need energy to apart them.

but according to C's law and entropy, electron will release energy and stick to the proton.
Title: Re: Why don't an atom's electrons fall into the nucleus and stick to the protons?
Post by: chiralSPO on 15/03/2015 17:17:03
because electron and proton are attracting each other, so need energy to apart them.

but according to C's law and entropy, electron will release energy and stick to the proton.

And why don't you count a hydrogen atom as an electron stuck to a proton?
Title: Re: Why don't an atom's electrons fall into the nucleus and stick to the protons?
Post by: jccc on 15/03/2015 17:34:28
if they stick together, atom will be so small, the universe will be a little ball with all charges sleep together.

you can say the negative charged elastic fluid condensed over proton and form a solid ball and electron stick to that ball to from hydrogen atom.

it all end up add a basic building block to the atomic structure to explain why atoms are not compressible, no discharge, no electron proton marriage.
Title: Re: Why don't an atom's electrons fall into the nucleus and stick to the protons?
Post by: chiralSPO on 15/03/2015 17:40:31
if they stick together, atom will be so small, the universe will be a little ball with all charges sleep together.

you can say the negative charged elastic fluid condensed over proton and form a solid ball and electron stick to that ball to from hydrogen atom.

it all end up add a basic building block to the atomic structure to explain why atoms are not compressible, no discharge, no electron proton marriage.

I really don't see what we gain from having an additional negative substance.

Why doesn't this imaginary negative liquid stick so close to the proton that the whole universe is a "small ball"?

How does this negative fluid behave differently from electrons?

And why wouldn't enough negative fluid surround the nucleus that it wouldn't also need to attract electrons?
Title: Re: Why don't an atom's electrons fall into the nucleus and stick to the protons?
Post by: jccc on 15/03/2015 18:16:04
the fluid is charged, so it attracted by proton. it is elastic, so its density around proton decay as 1/r^3, under the attraction force f=1/r^2.

if you split electron to zillion pieces, that could be the fluid itself.

electron's charge strength at atom radius is equal to the fluid balls charge strength. 
Title: Re: Why don't an atom's electrons fall into the nucleus and stick to the protons?
Post by: chiralSPO on 15/03/2015 20:38:03

if you split electron to zillion pieces, that could be the fluid itself.

electron's charge strength at atom radius is equal to the fluid balls charge strength.

So you agree that one proton surrounded by a zillion pieces of one electron is a reasonable solution to the question?
Title: Re: Why don't an atom's electrons fall into the nucleus and stick to the protons?
Post by: jccc on 15/03/2015 21:57:40

if you split electron to zillion pieces, that could be the fluid itself.

electron's charge strength at atom radius is equal to the fluid balls charge strength.

So you agree that one proton surrounded by a zillion pieces of one electron is a reasonable solution to the question?

not at all.

i think proton carries 900+ charges, attracted 899 fluid and 1 electron to form hydrogen atom. see above few postings.
Title: Re: Why don't an atom's electrons fall into the nucleus and stick to the protons?
Post by: chiralSPO on 16/03/2015 01:43:01

i think proton carries 900+ charges, attracted 899 fluid and 1 electron to form hydrogen atom. see above few postings.

Interesting theory. Can you cite any experimental evidence to back it up? Or make any testable predictions?

If one proton did carry such a charge, and were surrounded by enough negative fluid to cancel out all but one electron's worth of charge I have a few questions:

1) Does it make sense that a helium nucleus would attract just enough negative charge that it still required exactly 2 electrons? And a bismuth nucleus would attract just enough negative fluid that it needs exactly 83 electrons? (still one electron for one proton)

2) Is there ever a way to separate any of this negative fluid from the proton? The hydride anion (H), for example, has two electrons and one proton. Why doesn't it leak negative fluid? If the attraction between the fluid and the proton is purely electrostatic, it should be possible to rip it off using a sufficiently strong electric field. Why
have we never observed a "proton" with any charge greater than +1, even when exposed to millions of volts? What powerful force prevents this discharge?

3) If every proton attracts exactly the same amount of this negative fluid under all circumstances, and the negative fluid cannot be separated from the proton under any circumstances, can we consider the fluid part of the proton?
Title: Re: Why don't an atom's electrons fall into the nucleus and stick to the protons?
Post by: jccc on 16/03/2015 05:49:02
http://physics.bu.edu/~duffy/PY106/PeriodicTable.html

see if you have any questions on that page?
Title: Re: Why don't an atom's electrons fall into the nucleus and stick to the protons?
Post by: evan_au on 16/03/2015 10:17:44
Quote from: jccc
what wave? how electron waves? standing wave?

You can think of an electron as having wave-like properties.

The Frenchman Louis de Broglie  (http://en.wikipedia.org/wiki/Louis_de_Broglie) got a Nobel Prize for the wave-particle duality model in 1929 after his some of his predictions were demonstrated in the laboratory.

One of the properties of a wave is that it's position is a bit vague (especially if you speak French...). It is not possible to confine this wave within the diameter of a nucleus, and still have an electron. A wave (and an electron) are not single-point particles.

De Broglie's model of the atom is now a little dated. Later mathematical models of the atom by Erwin Shroedinger  (http://en.wikipedia.org/wiki/Wave_function) are now considered an even more useful model of the electrons around a proton.

Quote
is the waving electron still carry negative charge?
Yes

Quote
if so why is it not stick to the proton?
It does stick to the proton - we call it a Hydrogen atom.
You can free an electron from the proton by hitting it with energy in the form of an ultraviolet photon.
This would not be possible if an electron was physically inside the nucleus - the energy to free the electron from the electrostatic field would be much higher than the ultraviolet wavelength we observe. It would require a gamma ray (much higher energy) to free the electron.

If an electron approaches another proton, it may stick to that proton, by emitting an ultraviolet photon (or several photons in the visible range, with the same total energy as the ultraviolet photon).
Title: Re: Why don't an atom's electrons fall into the nucleus and stick to the protons?
Post by: chiralSPO on 16/03/2015 12:46:53
http://physics.bu.edu/~duffy/PY106/PeriodicTable.html

see if you have any questions on that page?

This page states pretty standard stuff. I don't necessarily like the way it's all presented, but I accept it is generally true.

How about you try answering some of my questions?

If one proton did carry such a charge, and were surrounded by enough negative fluid to cancel out all but one electron's worth of charge I have a few questions:

1) Does it make sense that a helium nucleus would attract just enough negative charge that it still required exactly 2 electrons? And a bismuth nucleus would attract just enough negative fluid that it needs exactly 83 electrons? (still one electron for one proton)

2) Is there ever a way to separate any of this negative fluid from the proton? The hydride anion (H), for example, has two electrons and one proton. Why doesn't it leak negative fluid? If the attraction between the fluid and the proton is purely electrostatic, it should be possible to rip it off using a sufficiently strong electric field. Why
have we never observed a "proton" with any charge greater than +1, even when exposed to millions of volts? What powerful force prevents this discharge?

3) If every proton attracts exactly the same amount of this negative fluid under all circumstances, and the negative fluid cannot be separated from the proton under any circumstances, can we consider the fluid part of the proton?
Title: Re: Why don't an atom's electrons fall into the nucleus and stick to the protons?
Post by: chiralSPO on 16/03/2015 19:35:10
jccc, I have thought of another way of describing the hydrogen atom pictorially. It's not a completely accurate model, just an analogy that might help.

Think of the electric potential produced by the proton as a surface--essentially like the gravity wells represented in curved space-time. The proton is very small, so it can essentially be treated as a point particle, or we can use a nonzero radius for the cutoff of the well (finite depth of the well), either way it doesn't matter.

The electron can be thought of as a marble that is free to roll around on this surface. It will naturally roll down into the potential well created by the proton, and it will eventually get stuck in the well. It is centered at the same x-y coordinates as the proton (center of the marble is directly over the center of the well), but because it has a determined diameter, the marble can only go so far down into the well.

I have illustrated a 1-dimensional version of this (two including potential, but only one spatial coordinate: x). The size of the "marble" is determined by how massive the particle is (more massive means smaller marble) (the size of this marble represents the de Broglie wavelength λ = h/p, where p is momentum and h is Planck's constant).

Thus when a negative particle heavier than the electron is modeled, we get a smaller marble. For instance, the muon has the same charge as an electron, but is about 200 times more massive. The exotic atom formed by the interaction of a muon and a proton is exactly the same as a normal hydrogen atom, except the muon is distributed much closer to the proton (this is how muons catalyze fusion). Going even further, an antiproton (1832 times heavier than an electron) would be extremely close to the proton. The antiproton and proton would also interact via the strong force (which the electron and muon would not do) and would fairly quickly annihilate with the proton.
Title: Re: Why don't an atom's electrons fall into the nucleus and stick to the protons?
Post by: jccc on 17/03/2015 23:10:26
chiralSPO, thank you for everything and the diagram!

my idea about atom is on going, nothing to support. haven't thought ways to test it yet.

1) Does it make sense that a helium nucleus would attract just enough negative charge that it still required exactly 2 electrons? And a bismuth nucleus would attract just enough negative fluid that it needs exactly 83 electrons? (still one electron for one proton)

without the fluid, isn't helium just get 2 and bismuth just get 83?

2) Is there ever a way to separate any of this negative fluid from the proton? The hydride anion (H–), for example, has two electrons and one proton. Why doesn't it leak negative fluid? If the attraction between the fluid and the proton is purely electrostatic, it should be possible to rip it off using a sufficiently strong electric field. Why
have we never observed a "proton" with any charge greater than +1, even when exposed to millions of volts? What powerful force prevents this discharge?

try to separate the fluid and proton is like try to pump water out ocean. a proton's force field is much bigger than atom's radius, you can give an hydrogen atom more than 1 electron, as long no other atoms near by to fight for the electron, more electrons can stable with 1 proton.  the voltage between proton and electron is higher than any voltage man made, yet there is no discharge, sound like empty space?

3) If every proton attracts exactly the same amount of this negative fluid under all circumstances, and the negative fluid cannot be separated from the proton under any circumstances, can we consider the fluid part of the proton?

of cause we can think so, but maybe call it nucleus is more likely.

Title: Re: Why don't an atom's electrons fall into the nucleus and stick to the protons?
Post by: alancalverd on 17/03/2015 23:19:04
maybe proton carries 900+, attracted 899- fluid to form nucleus, add 1 electron to form hydrogen.


Alas, an isolated proton has a charge of +1, by experiment.
Title: Re: Why don't an atom's electrons fall into the nucleus and stick to the protons?
Post by: jccc on 17/03/2015 23:42:34
alash. a hydrogen is neutral, by all means. yet you cannot say it contains no charge. you say its net charge is 0.

if proton carries the fluid with it, net charge is +1, will test show +1 charge?


Title: Re: Why don't an atom's electrons fall into the nucleus and stick to the protons?
Post by: Ethos_ on 17/03/2015 23:51:49
alash. a hydrogen is neutral, by all means. yet you cannot say it contains no charge. you say its net charge is 0.

if proton carries the fluid with it, net charge is +1, will test show +1 charge?
Experiment shows that a proton consists of two up quarks with a charge of +2/3 each. And one down quark with a charge of -1/3. Taking 2 times +2/3 equals +4/3 and adding the -1/3 charges leaves us with +3/3 simplified to +1 charge. I really don't know where you are getting some of your figures jccc, they sure aren't coming form a physics book. Where did you ever come up with the proton having 900+ charges?
Title: Re: Why don't an atom's electrons fall into the nucleus and stick to the protons?
Post by: jccc on 18/03/2015 00:05:04
experiment shows photon knocks out electron, is photon a real thing?

i assume proton carries 900+, because i believe real mass is the ratio of force act on charge.

think about, when charge doubled, its force doubled. push 1 charge particle to accelerate at a, need force f, push 3 charges need 3 f to get same a.

test shows hydrogen weights 1800 electron weight, that's why i believe proton carries 900+, plus 899- and 1 electron to become neutral atom.
Title: Re: Why don't an atom's electrons fall into the nucleus and stick to the protons?
Post by: chiralSPO on 18/03/2015 02:00:31
I think a hydrogen atom has a mass about 1837 times that of an electron.

I don't think it's reasonable to assume that the mass and charge scale together. Protons are hadrons and electrons are leptons, so there is no reason to think that their mass to charge ratio should be equal...

And didn't you claim that this negative fluid had no mass before? Now it has the same mass to charge ratio as electrons?
Title: Re: Why don't an atom's electrons fall into the nucleus and stick to the protons?
Post by: jccc on 18/03/2015 02:16:35
i believe without charge, there is no force, therefore no mass.

why don't you use the same attitude to question standard atom models?

why don't you answer my questions on the photon thread? 
Title: Re: Why don't an atom's electrons fall into the nucleus and stick to the protons?
Post by: chiralSPO on 18/03/2015 03:14:20
I have spent years learning about and questioning the standard atomic theories. Most of my questions on the subject have been answered to my satisfaction or acknowledged as current limitations of our understanding.

Your theory is much newer to me and inconsistent with much of what I have learned, and sometimes inconsistent with itself.

Mostly I am asking questions to make you think hard about your theory and how it fits (or doesn't) with our current understanding of the world--hopefully my questions help you refine and communicate your ideas.
Title: Re: Why don't an atom's electrons fall into the nucleus and stick to the protons?
Post by: jccc on 18/03/2015 03:46:02
certainly all your questions helped me to think better and deeper, never can be forgotten.

did you asked yourself why electron and proton not stick together? why no discharge? why matter is not compressible? how electron waving around proton? what is energy level? how is n p s shell carry electrons? what's the mechanism?

is book gives you the correct answers? you are satisfied 100% or 20%?   
Title: Re: Why don't an atom's electrons fall into the nucleus and stick to the protons?
Post by: PmbPhy on 18/03/2015 04:00:04
Quote from: jccc
certainly all your questions helped me to think better and deeper, never can be forgotten.
And yet you show no signs of that because you ask the same exact questions.

Quote from: jccc
did you asked yourself why electron and proton not stick together?
Most of us here know why. You're alone in this except for a few people with no understanding of quantum mechanics.

Quote from: jccc
why matter is not compressible?
False claim since matter is compressible. You've been told this countless times now and like other questions you still act like nobody's explained it to you. Again you're attempting to irritate members of this forum with that childish game of yours.
Title: Re: Why don't an atom's electrons fall into the nucleus and stick to the protons?
Post by: alancalverd on 18/03/2015 18:47:13
Pete, have you not been warned about p***ing into the wind? jccc has his own beliefs about the structure of the hydrogen atom and no amount of observation or commonsense will change his mind, possibly because we are all capitalist lickspittles, commie atheists, or evil alien lizardpeople dedicated to obscuring the Truth that has been Revealed to jccc alone.
Title: Re: Why don't an atom's electrons fall into the nucleus and stick to the protons?
Post by: jccc on 19/03/2015 12:37:15
a more logically sounding theory of atomic structure is discussed in the last 6 pages, 20k new viewers.

i kind disappointed to see some of the comments.

question is, am i the only one?
Title: Re: Why don't an atom's electrons fall into the nucleus and stick to the protons?
Post by: chiralSPO on 19/03/2015 15:07:27
certainly all your questions helped me to think better and deeper, never can be forgotten.
:-)

did you asked yourself why electron and proton not stick together? why no discharge? why matter is not compressible? how electron waving around proton? what is energy level? how is n p s shell carry electrons? what's the mechanism?
Are you not listening? We have said multiple times that an electron stuck to proton is a hydrogen atom!

How could an electron discharge if the smallest charge carrier is an electron?

Matter is compressible! It takes a lot of force, but densities of crystals increase at increased pressure. With crazy amounts of force (neutron star) the electrons can be forced into the nucleus, forming a neutrons.

An electron behaves in a wavelike manner and it is around the proton--this has been shown experimentally in many different ways.

Energy levels can be thought of as the different harmonics of the electron wave in the atom (or molecule).

n is the way we denote energy level. It is one of the principle quantum numbers we use to describe electrons in an atom (like an address for each electron) the others are l (azimuthal quantum number, relates to magnitude of angular momentum), ml (magnetic quantum number, relates to direction of angular momentum) and ms (spin). These are hard concepts to grasp, but essentially, there are only so many ways an electron wave can be stable around a nucleus, and these four numbers are used to describe the different stable solutions.

For n = 1, there are no solutions with angular momentum (l = 0 and ml = 0)
For n = 2, l can be 0 or 1, and when l = 1 ml can be –1, 0 or 1
overall for any n, l can have integer values of 0, 1, 2, .... all the way up to n–1
for any l, ml can have any integer value between –l and l

any orbital with l = 0 is called an s orbital; any orbital with l = 1 is called a p orbital (and remember when l = 1 ml can be –1, 0 or 1) so there are three types of orthogonal p orbitals
any orbital with l = 2 is called a d orbital (there are 5 types of orthogonal d orbitals) etc. etc. etc. this is all just our naming system, the actual equation that I'm dancing around is called the Schrödinger equation, which is used to calculate the possible wavefunctions of electrons in an atom (or molecule).

The mechanism is that that's the way the universe works. We are just describing it.

is book gives you the correct answers? you are satisfied 100% or 20%?

I am about 90% satisfied with book answers. The remaining 10% represents the combination of my own lack of understanding and what scientists everywhere don't yet have good explanations for.
Title: Re: Why don't an atom's electrons fall into the nucleus and stick to the protons?
Post by: chiralSPO on 19/03/2015 15:10:00
a more logically sounding theory of atomic structure is discussed in the last 6 pages, 20k new viewers.

i kind disappointed to see some of the comments.

question is, am i the only one?

Your theory is NOT more logical, and don't think that the number of views this thread gets is any confirmation of your theory. It is probably the result of the title being a commonly held question, coming up in google searches, and forum members interested in hearing/shooting down the next crazy thing you throw up into the thread.
Title: Re: Why don't an atom's electrons fall into the nucleus and stick to the protons?
Post by: jccc on 19/03/2015 16:05:00
a more logically sounding theory of atomic structure is discussed in the last 6 pages, 20k new viewers.

i kind disappointed to see some of the comments.

question is, am i the only one?

Your theory is NOT more logical, and don't think that the number of views this thread gets is any confirmation of your theory. It is probably the result of the title being a commonly held question, coming up in google searches, and forum members interested in hearing/shooting down the next crazy thing you throw up into the thread.

which part is not logical? did i answered every question you asked for?

please point, thank you.
Title: Re: Why don't an atom's electrons fall into the nucleus and stick to the protons?
Post by: jccc on 19/03/2015 18:30:23
We have said multiple times that an electron stuck to proton is a hydrogen atom!

an atom's radius is way bigger than a proton, which part of the electron is stuck to a proton? how they stuck? if they stuck why there is atom radius?

Matter is compressible! It takes a lot of force.

1/10^10 is compreessible? isn't atomis mostly empty space between proton and electron?

How could an electron discharge if the smallest charge carrier is an electron?

electron always discharge into positive charge, what else particle is more dischargeable?

Energy levels can be thought of as the different harmonics of the electron wave in the atom.

who though so? what wave? how a particle waves? what's the mechanism?

The mechanism is that that's the way the universe works. We are just describing it.

you see n, s, p shells? see electrons there? count them? how?
Title: Re: Why don't an atom's electrons fall into the nucleus and stick to the protons?
Post by: jccc on 19/03/2015 19:26:15
think about it, a building, an earth quake can damage it, a bomb can destroy it. if atoms are as science said, electrons are waving around the nucleus, all perfectly balanced, why is compressibility is 10^-10? why is during fission reaction, not even damage any part of it?

can we build anything similar? not in imagination?
Title: Re: Why don't an atom's electrons fall into the nucleus and stick to the protons?
Post by: chiralSPO on 19/03/2015 19:33:31
We have said multiple times that an electron stuck to proton is a hydrogen atom!

an atom's radius is way bigger than a proton, which part of the electron is stuck to a proton? how they stuck? if they stuck why there is atom radius?
The atom's radius is set by the electron, not the proton. They are stuck by electrostatic attraction.

Matter is compressible! It takes a lot of force.

1/10^10 is compreessible? isn't atomis mostly empty space between proton and electron?
I don't know where you're getting that number (and no units, so it's not very meaningful...), but since it's not zero, I would say it probably means that matter is compressible. There isn't really empty space in the atom. The electron(s) is (are) there.

How could an electron discharge if the smallest charge carrier is an electron?

electron always discharge into positive charge, what else particle is more dischargeable?
Not sure what you mean here. The electron is already "at" the proton, so it can't move closer.

Energy levels can be thought of as the different harmonics of the electron wave in the atom.

who though so? what wave? how a particle waves? what's the mechanism?

Louis de Broglie formalized the concept of wave-particle duality. The electron wave (or electron probability density wave, if you prefer)

The mechanism is that that's the way the universe works. We are just describing it.

you see n, s, p shells? see electrons there? count them? how?

I can't see s, p, d shells. But I can measure them, and I can model them.
Title: Re: Why don't an atom's electrons fall into the nucleus and stick to the protons?
Post by: jccc on 19/03/2015 19:50:12
seriously, your above statement is no better than your crashing bird 1.
Title: Re: Why don't an atom's electrons fall into the nucleus and stick to the protons?
Post by: chiralSPO on 19/03/2015 20:30:27
At this point, you're on your own. The answers are all in this thread. If you are actually a truth seeker, you will read more about quantum mechanics. I recommend you look at the history of it, rather than the mathematics of it. You will see how people made their discoveries, how they defended their theories against criticism, and how inventors were able to take advantage of our new-found understanding of atomic and molecular systems.

If you would rather make up your own reality, and you don't care how representative it is of the universe, have fun.
Title: Re: Why don't an atom's electrons fall into the nucleus and stick to the protons?
Post by: jccc on 19/03/2015 20:40:09
you mean you cannot point out what part of my theory is NOT logical?
Title: Re: Why don't an atom's electrons fall into the nucleus and stick to the protons?
Post by: Ethos_ on 19/03/2015 21:05:01
you mean you cannot point out what part of my theory is NOT logical?
How about your post #329? Makes absolutely no sense!
Title: Re: Why don't an atom's electrons fall into the nucleus and stick to the protons?
Post by: jccc on 19/03/2015 21:31:22
maybe proton carries 900+, attracted 899- fluid to form nucleus, add 1 electron to form hydrogen.

the rest fluid maybe the source of dm/de?

atomic structure has to be 100% correct, otherwise whole science is doubtful.

any thoughts?

this 1?
Title: Re: Why don't an atom's electrons fall into the nucleus and stick to the protons?
Post by: Ethos_ on 19/03/2015 21:36:22
maybe proton carries 900+, attracted 899- fluid to form nucleus, add 1 electron to form hydrogen.

the rest fluid maybe the source of dm/de?

atomic structure has to be 100% correct, otherwise whole science is doubtful.

any thoughts?

this 1?
Yep...................
Title: Re: Why don't an atom's electrons fall into the nucleus and stick to the protons?
Post by: jccc on 19/03/2015 21:52:27
please see 353, thanks!
Title: None
Post by: corrupt on 20/03/2015 05:26:36
How can a neutron (which is comprised of two down-quarks and an up-quark) turn into an electron (which is itself a fundamental particle)?
Title: Re: Why don't an atom's electrons fall into the nucleus and stick to the protons?
Post by: lightarrow on 20/03/2015 17:52:13
you mean you cannot point out what part of my theory is NOT logical?
How can a soup of silly things be considered a theory? Or you are kidding us all? You are doing it, say the truth...
But I can't understand those who keep ansering you losing their time ...

--
lightarrow
Title: Re: None
Post by: chiralSPO on 20/03/2015 18:39:26
How can a neutron (which is comprised of two down-quarks and an up-quark) turn into an electron (which is itself a fundamental particle)?

I'm not exactly sure of the mechanism, but neutrons are know to decompose into a proton, an electron and a neutrino. If you crunch the numbers, I bet the mass-energy, charge (this one's easy) and spin (angular momentum) are all conserved.

Any QED experts want to shed light on this one?
Title: Re: Why don't an atom's electrons fall into the nucleus and stick to the protons?
Post by: guest39538 on 21/03/2015 12:39:50
Because the electron field is produced by the Proton?

or by simply being so small the electron is relatively fell into the Proton, how big do you imagine this atom to be?

Title: Re: Why don't an atom's electrons fall into the nucleus and stick to the protons?
Post by: Ethos_ on 21/03/2015 14:00:04

How can a soup of silly things be considered a theory? Or you are kidding us all? You are doing it, say the truth...
But I can't understand those who keep ansering you losing their time ...

--
lightarrow
I agree with you lightarrow, I think these fellows have come here with the single agenda, to sow discourse and cause confusion.
Title: Re: Why don't an atom's electrons fall into the nucleus and stick to the protons?
Post by: guest39538 on 21/03/2015 14:32:36
I understand the question and it leaves room for thought, if an electron is attracted to a Proton then what stops the electron just being connected directly to the Proton?

The answer is simply that atoms are so small, relatively Electrons  are connected directly to the Proton, there is hardly any space in something smaller than a dot?


Title: Re: Why don't an atom's electrons fall into the nucleus and stick to the protons?
Post by: jccc on 21/03/2015 17:55:26
Your time is limited, so don’t waste it living someone else’s life. Don’t be trapped by dogma — which is living with the results of other people’s thinking. Don’t let the noise of others’ opinions drown out your own inner voice. And most important, have the courage to follow your heart and intuition. They somehow already know what you truly want to become. Everything else is secondary.

quote from Steve Jobs
Title: Re: Why don't an atom's electrons fall into the nucleus and stick to the protons?
Post by: PmbPhy on 12/06/2015 02:06:12
Quote from: UltimateTheory
Antiparticle has positive energy. Dirac was mistaken.
That's a misconception. The equation E2 + p2 = m2  has two solutions for E. One is positive and the other is negative. You're statement confuses the inertial energy = rest energy + kinetic energy with negative energy states. As explained here: http://quantummechanics.ucsd.edu/ph130a/130_notes/node490.html
Quote
We cannot discount the ``negative energy'' solutions since the positive energy solutions alone do not form a complete set. An electron which is localized in space, will have components of its wave function which are ``negative energy''.
...
The idea of an infinite sea of ``negative energy'' electrons is a strange one. What about all that charge and negative energy? Why is there an asymmetry in the vacuum between negative and positive energy when Dirac's equation is symmetric? (We could also have said that positrons have positive energy and there is an infinite sea of electrons in negative energy states.) This is probably not the right answer but it has many elements of truth in it. It also gives the right result for some simple calculations. When the Dirac field is quantized, we will no longer need the infinite ``negative energy'' sea, but electrons and positrons will behave as if it were there.
Title: Re: Why don't an atom's electrons fall into the nucleus and stick to the protons?
Post by: jccc on 22/06/2015 19:42:17
we have all kinds of high tech toys, why can scientists make a working model of an atom?

let's brainstorm, see if we can use 1 electron and 1 proton to make an atom.

first, let's think, if they are stick together in the beginning, we got to separate them first to make possible that electron able to orbiting/clouding.

if they are separate at a distance, they will accelerate and get closer, how could impact not happen? lucky electron just fall into orbit?

am i the only 1 so confused? 
Title: Re: Why don't an atom's electrons fall into the nucleus and stick to the protons?
Post by: chiralSPO on 22/06/2015 19:50:09
am i the only 1 so confused?

only you and any others who can't let go of the intuition of classical physics...
Title: Re: Why don't an atom's electrons fall into the nucleus and stick to the protons?
Post by: jccc on 22/06/2015 20:35:17
seems to me the standard model of atomic structure is more confuse.

there is no clear logic to explain many facts we observe daily.

why atoms are not compressible as theory predicted? 99.99% empty space within atom right?

why electron not discharge into proton? any other em field is stronger? any other voltage is higher?

how neutral charged atoms able to form into group?

all legit questions, agree?
Title: Re: Why don't an atom's electrons fall into the nucleus and stick to the protons?
Post by: chiralSPO on 22/06/2015 20:35:52
I would refer you again to my post (much) earlier in this thread (also check out the attachment in the original post):

jccc, I have thought of another way of describing the hydrogen atom pictorially. It's not a completely accurate model, just an analogy that might help.

Think of the electric potential produced by the proton as a surface--essentially like the gravity wells represented in curved space-time. The proton is very small, so it can essentially be treated as a point particle, or we can use a nonzero radius for the cutoff of the well (finite depth of the well), either way it doesn't matter.

The electron can be thought of as a marble that is free to roll around on this surface. It will naturally roll down into the potential well created by the proton, and it will eventually get stuck in the well. It is centered at the same x-y coordinates as the proton (center of the marble is directly over the center of the well), but because it has a determined diameter, the marble can only go so far down into the well.

I have illustrated a 1-dimensional version of this (two including potential, but only one spatial coordinate: x). The size of the "marble" is determined by how massive the particle is (more massive means smaller marble) (the size of this marble represents the de Broglie wavelength λ = h/p, where p is momentum and h is Planck's constant).

Thus when a negative particle heavier than the electron is modeled, we get a smaller marble. For instance, the muon has the same charge as an electron, but is about 200 times more massive. The exotic atom formed by the interaction of a muon and a proton is exactly the same as a normal hydrogen atom, except the muon is distributed much closer to the proton (this is how muons catalyze fusion). Going even further, an antiproton (1832 times heavier than an electron) would be extremely close to the proton. The antiproton and proton would also interact via the strong force (which the electron and muon would not do) and would fairly quickly annihilate with the proton.
Title: Re: Why don't an atom's electrons fall into the nucleus and stick to the protons?
Post by: chiralSPO on 22/06/2015 20:41:54

why atoms are not compressible as theory predicted? 99.99% empty space within atom right?

why electron not discharge into proton? any other em field is stronger? any other voltage is higher?

how neutral charged atoms able to form into group?

all legit questions, agree?

all legit questions, all with legit answers.

For instance: neutral atoms form into groups because it is usually energetically favorable for the electrons to be shared between multiple nuclei. The atomic orbitals we continuously talk about morph into molecular orbitals when there are multiple nuclei to be considered. We have developed excellent ways of modeling the electronic structures of molecules--to the extent that we can say how many bonds atoms will make with each other; whether molecules will be paramagnetic or diamagnetic; what frequencies of light a molecule is likely to absorb; whether molecules will conduct electricity or not; etc. etc. etc.
Title: Re: Why don't an atom's electrons fall into the nucleus and stick to the protons?
Post by: PmbPhy on 22/06/2015 21:42:05
Quote from: chiralSPO
only you and any others who can't let go of the intuition of classical physics...
We've told him that countless times and he ignores it as if we never even said it. He's terrible that way.
Title: Re: Why don't an atom's electrons fall into the nucleus and stick to the protons?
Post by: jccc on 22/06/2015 21:54:19

why atoms are not compressible as theory predicted? 99.99% empty space within atom right?

why electron not discharge into proton? any other em field is stronger? any other voltage is higher?

how neutral charged atoms able to form into group?

all legit questions, agree?

all legit questions, all with legit answers.

For instance: neutral atoms form into groups because it is usually energetically favorable for the electrons to be shared between multiple nuclei.

ok. then if atoms are apart, the same energetically favorable should be for the atoms to share the outer electrons and attract each other that causes gravitation.

am i have a point?
Title: Re: Why don't an atom's electrons fall into the nucleus and stick to the protons?
Post by: chiralSPO on 22/06/2015 22:45:21
ok. then if atoms are apart, the same energetically favorable should be for the atoms to share the outer electrons and attract each other that causes gravitation.

am i have a point?

except that the attraction between atoms to form molecules is only over very short distances (more than 5 Å, or 5x10–10 between nuclei, and there is no substantial effect) because it requires sharing of electrons, and the effect becomes repulsive (antibonding) if there are too many electrons around. Typically atoms come together to form very discrete molecular units (like H2O or C6H6), and once the stable molecule is formed, it becomes very difficult to add more atoms. There are much weaker interactions between molecules that make it favorable for molecules to attract oneanother (this is how gases condense into liquids, how geckos can walk on walls, and how drugs bind to receptors)
Title: Re: Why don't an atom's electrons fall into the nucleus and stick to the protons?
Post by: jccc on 22/06/2015 23:48:37
2 separate atoms each is neutral charged, what attraction force there is?

atom 1 proton attracts atom 2 electron or what? how it exactly works? if not induction?

are the 2 atoms share electrons or clouds? or orbitals? is all this imaginary?
Title: Re: Why don't an atom's electrons fall into the nucleus and stick to the protons?
Post by: chiralSPO on 23/06/2015 00:05:05
https://en.wikipedia.org/wiki/Covalent_bond
https://en.wikipedia.org/wiki/Molecular_orbital_theory
Title: Re: Why don't an atom's electrons fall into the nucleus and stick to the protons?
Post by: jccc on 23/06/2015 00:14:20
https://en.wikipedia.org/wiki/Covalent_bond
https://en.wikipedia.org/wiki/Molecular_orbital_theory

if i believe/understand those theory, i won't be here.

if you understand, please explain in your words. those wiki knowledge to me not like science.
Title: Re: Why don't an atom's electrons fall into the nucleus and stick to the protons?
Post by: PmbPhy on 23/06/2015 04:47:17
https://en.wikipedia.org/wiki/Covalent_bond
https://en.wikipedia.org/wiki/Molecular_orbital_theory

if i believe/understand those theory, i won't be here.

if you understand, please explain in your words. those wiki knowledge to me not like science.
Wiki does explain it all in words. As we keep telling you, and which you keep choosing to ignore, is that since you refuse to learn physics the correct way, rather than asking a zillion questions whose answers you readily forget, you'll never understand it. Why on Earth do you think physicists have to study so hard. Don't you think that we would have preferred an explanation so simple that even you could understand it. Forget it. It doesn't exist. Nature is a great deal more complicated for a description so simplistic that even you could understand it.
Title: Re: Why don't an atom's electrons fall into the nucleus and stick to the protons?
Post by: jccc on 23/06/2015 05:11:50

Quote from: jccc on 27/02/2015 15:43:14
Quote from: PmbPhy on 27/02/2015 15:21:28
Quote from: jccc
we need to start from the light source. if atoms are like qm suggested, 99% empty space, why is water/matter not compressible?
It is compressible. See:
http://en.wikipedia.org/wiki/Properties_of_water#Compressibility
water's compressibility is about 10 ^-10, sounds like 99% empty space to you?

how about the discharge? is the empty space such a good insulator?
what's your answer?
Modify message
Report to moderator     173.22.244.21
PmbPhy
Hero Member
*****
Posts: 1898
View Profile  Email  Personal Message (Offline)

Re: Why don't an atom's electrons fall into the nucleus and stick to the protons?
« Reply #266 on: 28/02/2015 02:36:57 »
Quote
Quote from: jccc
what's your answer?
I don't have an answer. Who ever said I know everything!
Report to moderator     Logged
jccc
Hero Member
*****
Posts: 851
View Profile  Personal Message (Online)

Re: Why don't an atom's electrons fall into the nucleus and stick to the protons?
« Reply #267 on: 28/02/2015 02:43:18 »
QuoteModifyRemove
thank you Pete!

all my respect to you.

Modify message
Report to moderator     173.22.244.21
PmbPhy
Hero Member
*****
Posts: 1898
View Profile  Email  Personal Message (Offline)

Re: Why don't an atom's electrons fall into the nucleus and stick to the protons?
« Reply #268 on: 28/02/2015 03:17:57 »
Quote
Quote from: jccc on 28/02/2015 02:43:18
thank you Pete!

all my respect to you.
You're welcome.  ^

do you have answers now?
Title: Re: Why don't an atom's electrons fall into the nucleus and stick to the protons?
Post by: PmbPhy on 23/06/2015 05:50:32
Quote from: jccc

Quote from: jccc on 27/02/2015 15:43:14
Quote from: PmbPhy on 27/02/2015 15:21:28
Quote from: jccc
we need to start from the light source. if atoms are like qm suggested, 99% empty space, why is water/matter not compressible?
It is compressible. See:
http://en.wikipedia.org/wiki/Properties_of_water#Compressibility
water's compressibility is about 10 ^-10, sounds like 99% empty space to you?

how about the discharge? is the empty space such a good insulator?
what's your answer?
Modify message
Report to moderator     173.22.244.21
PmbPhy
Hero Member
<snipped garbage>
Well, that was about the most useless post that I've ever seen in any forum that I've ever visited. Then again, nothing that you do surprises me anymore.
Title: Re: Why don't an atom's electrons fall into the nucleus and stick to the protons?
Post by: jccc on 23/06/2015 05:54:24
you don't remember what you said earlier in this thread?
Title: Re: Why don't an atom's electrons fall into the nucleus and stick to the protons?
Post by: PmbPhy on 23/06/2015 06:08:29
Quote from: jccc
you don't remember what you said earlier in this thread?
You nut. There are 396 posts in this forum. I've posted in many threads. Why should I recall what I posted earlier in this thread?

Sheeesh!
Title: Re: Why don't an atom's electrons fall into the nucleus and stick to the protons?
Post by: jccc on 23/06/2015 06:31:56
you do remember your science knowledge right?

you can read right?

do you have answers now?

why is atoms not compressible as 99% empty space?

why electron not discharge into proton? is the space between them such great insulator?
Title: Re: Why don't an atom's electrons fall into the nucleus and stick to the protons?
Post by: PmbPhy on 23/06/2015 08:55:07
Quote from: jccc
you do remember your science knowledge right?

you can read right?

do you have answers now?
Oh my God! What the hell is wrong with you? How many damn times do I have to tell you that I will not make anymore attempts to answer your questions because you don't have the ability to grasp the answers? I even posted this
http://mightylib.mit.edu/Course%20Materials/22.01/Fall%202001/why%20nuclei%20decay.pdf
 the other day and you completely ignored it once again proving to me that you're a waste of our time.

I've said that a half a dozen times yesterday and you still haven't gotten that through your thick skull? If you can't grasp that simple fact then how on Earth do you think that you'd be able to grasp a response that's framed in quantum mechanics (QM)?  You're the one who has chosen not to learn QM. Nobody made that decision for you. And without the ability to understand QM you have no hope of understanding the answers to the questions you're asking. I know because I've already answered those same exact questions a dozen times in this forum and not only were you unable to grasp the answer each time I gave it to you but since then you've made no attempt to learn QM for yourself.

And you have the nerve to think that I'm going to keep repeating myself over and over again every time you ask me the same exact question? Especially since you can always look it up on the internet for yourself and you're just too lazy to do it.
Title: Re: Why don't an atom's electrons fall into the nucleus and stick to the protons?
Post by: jccc on 23/06/2015 08:59:13
you have no God. I am fine.

if you don't want to answer my questions, leave my comments alone. why even bother reply?
Title: Re: Why don't an atom's electrons fall into the nucleus and stick to the protons?
Post by: PmbPhy on 23/06/2015 09:07:46
Quote from: jccc
you have no God. I am fine.
What a dumb comment. When I was a Christian earlier this year did you think I tolerated your asinine comments then either?

Quote from: jccc
if you don't want to answer my questions, leave my comments alone. why even bother reply?
Wow, you're dumb. You're the one who asked me the questions. I didn't prompt you to do so. And since they're not comments but questions I responded to them in a manner I thought appropriate. You need to remember that you've had these questions answered countless times and if you already know that then other members need to know that so they don't waste their time addressing them.

Since you feel free to ask me questions then here's one for you - Why do you ask the same questions over and over again when you've already gotten the answers to them? The correct answer won't change merely because someone else responds. And why do you ignore everyone who asks you questions?
Title: Re: Why don't an atom's electrons fall into the nucleus and stick to the protons?
Post by: Ethos_ on 23/06/2015 15:49:51
you have no God. I am fine.

if you don't want to answer my questions, leave my comments alone. why even bother reply?
I agree with Pete! If you have a problem with the answer you're getting to your questions, quit asking the same question over and over and over again. If you have an answer you prefer more than the ones you're receiving here at TNS, why are you even asking? If you like your answer more than those offered here, please explain why your answers are more appropriate. Otherwise, stuff it!
Title: Re: Why don't an atom's electrons fall into the nucleus and stick to the protons?
Post by: PmbPhy on 23/06/2015 16:44:42
you have no God. I am fine.

if you don't want to answer my questions, leave my comments alone. why even bother reply?
I agree with Pete! If you have a problem with the answer you're getting to your questions, quit asking the same question over and over and over again. If you have an answer you prefer more than the ones you're receiving here at TNS, why are you even asking? If you like your answer more than those offered here, please explain why your answers are more appropriate. Otherwise, stuff it!
Thanks, buddy. I can't tell you how many times I've said that and the worst part about it is that he even refuses to acknowledge that we even reminded him like this. Wow, is it irritating!
Title: Re: Why don't an atom's electrons fall into the nucleus and stick to the protons?
Post by: jccc on 23/06/2015 18:56:53
happy now?  i hope so.

light is atom's gravity wave

That makes no sense whatsoever.
Of course. Consider the source.

still make no sense?

theory is theory, right or wrong, what about source?

cynical?
Title: Re: Why don't an atom's electrons fall into the nucleus and stick to the protons?
Post by: jccc on 28/06/2015 10:03:37
as i said earlier in my theory, 3 building blocks make up atoms.

proton carries 900 positive charges, electron carries 1 negative charge, enertron carries tiny negative charge, but it has more charge to volume ratio than electron.

a proton can never be observed by itself, proton is deep hidden within a dense enertron ball. atom is like earth, proton is the core, enertron is the dirt, electron is giant beach ball.

electron can never get inside of atom radius, that's why atoms are not compressible, no discharge within atom.

proton attracts -899 energtron and 1 electron to become neutral charged atom.

we can not detect enertron because it is too small and attracted by proton more strongly than electron.

when proton beams impact, in fact is proton/enertron balls impact, enertron balls explode, produce all kinds of em phenomena.

without charges, there is no force, there is no mass. cus you cannot measure mass without using force.

1 atomic mass equals to 1800 total charges, no matter the sign of the charge, that's why proton weight 1800 times electron weight but only carries 1 positive charge, the rest 899 positive charge is used up to balance -899 enertron ball.

a hydrogen atom is made of +900 charged proton surrounded by -899 charged enertron ball, add 1 electron on the outer sphere.

the atom has 2 force fields, positive field fp=ke x 900/r^2, negative field fe=ke x -900/r^2.

those positive and negative fields between atoms/matters interact/induction becomes bounding/gravitation.
Title: Why don
Post by: CycleGuy on 07/11/2015 20:59:25
In properly answering this question, one must delve into Quantum Field Theory a bit.

All throughout the universe is a field, called the Quantum Vacuum Zero Point Energy field. It is the energy that has entropied to its lowest energy state, its "ground state". With more energy continually being "generated" from mass (via stars), and thus entropied, there is a continual "filling" of the universe's "bowl" of entropied energy. This Zero Point Energy is where mass came from, and it accounts for why the universe is expanding (QVZPE field density is increasing as more energy entropies, thus field radiation pressure is increasing, thus the universe must either create mass or expand to relieve that field radiation pressure, and it's energetically more conservative at this time to expand. Earlier in the universe's existence, it was energetically more conservative to concretize mass, accounting for that 1 billionth of 1% of all energy coming from the Big Bang being turned into the mass we have today).

According to Quantum Field Theory, everything is a wave... both massive entities and massless. What we see as "matter" is considered a relatively stable wave form in the Quantum Vacuum Zero Point Energy field, and those "virtual particles" that we've been told continually pop in and out of existence are unstable "pulses" in the QVZPE field density. Those "virtual particles" exist, it's just that they exist for such a short time before they settle back into the Zero Point Energy field that we call them "virtual". In fact, researchers at Chalmers University in 2011 "concretized" microwave photons directly from the QVZPE field using what is known as Dynamical Casimir Effect, thereby proving that these particles (which in Quantum Field Theory are considered waves) actually exist... it's just that their wave forms aren't stable enough to remain "concretized" for long. If a large enough differential in ZPE were to come about (ie: a "surge" in the ZPE, much like a wave of entropied energy), mass could be concretized. If the universe were to somehow stop expanding and the QVZPE field radiation pressure built up sufficiently, mass would be concretized.

The atom's nucleus is made up protons and neutrons, held together with the Strong Nuclear Force. The electrons aren't "billiard balls" whizzing about the nucleus, they're actually standing waves "orbiting" the nucleus. Because the electron must have an integer number (ie: round number) of De Broglie waves in its wavelength, this accounts for the Bohr orbit radius. As an electron absorbs energy, when it's absorbed enough energy that it can add additional De Broglie waves to its wavelength, it "jumps outward" in its orbit. When the electron gives off energy (as it is always doing) in trying to reach its lowest energy state, it sheds De Broglie waves in its standing wave orbit, gives off a photon that has the exact same amount of energy as the number of waves shed, and reduces the radius of its "orbit". If an electron's standing wave "orbit" didn't have an integer number of De Broglie waves in its wavelength, destructive interference would occur, and the "orbit" would not be stable.

Now, I said above that the electron is always giving off energy in attempting to reach its ground state. What stops the electron from giving off enough energy that it becomes attracted to the proton? The QVZPE field, of course. It's the "ground state", the lowest a system can go, thus the electrons have a minimum number of De Broglie waves in their standing wave orbits which they can drop to.

This has been known (well, suspected... and proven later) since 1975, when Boyer showed that the hydrogen atom in its ground state (ie: lowest orbit) would be in a state of equilibrium between Larmor radiation and absorption of QVZPE at the correct radius for a classical Rutherford hydrogen atom.

So now you know where our matter came from, why our matter is stable, the basis behind Bohr orbits, and why our universe is expanding. And it's all underpinned by Quantum Vacuum Zero Point Energy.

That said, there are instances when the electron does "crash into" the proton... it's called electron capture decay. It turns the proton into a neutron and causes transmutation. You can cause forced electron capture decay if an atom is placed into a reduced QVZPE field environment sufficient to reduce the number of De Broglie waves in the electron's orbit such that the electron is attracted to the proton.
Title: Re: Why don't an atom's electrons fall into the nucleus and stick to the protons?
Post by: jeffreyH on 07/11/2015 21:21:35
Well that was unexpected and interesting.
Title: Re: Why don't an atom's electrons fall into the nucleus and stick to the protons?
Post by: PmbPhy on 08/11/2015 09:06:55
Well that was unexpected and interesting.
After this post became inactive I found a webpage which explains it wonderfully. It's not as if I don't know the answer to the question but other people are so much better at explaining things to other people. Here's that page:
http://www.chem1.com/acad/webtut/atomic/WhyTheElectron.html

What's on that page you have you have
Title: Re: Why don't an atom's electrons fall into the nucleus and stick to the protons?
Post by: alancalverd on 08/11/2015 13:41:09
A neat reference- particularly as it supports my preferred use of "indeterminacy" for Heisenberg's principle.

I still think it better to start from the observation that atoms have a finite diameter, therefore you can't assume that classical electrostatics will model it.
Title: None
Post by: billy kakes on 02/01/2016 14:43:00
I understand, most answers to this do not sufficiently satisfy.  I have an explanation that will satisfy and makes sense.  The atom is arranged so the forces balance and prevent it from flying apart.  The Nucleus has spin so the electron is attracted to the proton which draws it in but there is a point in the rotation where the neutron is in between the electron and proton during that time the electron is temp no longer drawn in and since it rotates around the nucleus due to centrifugal force it is drawn outward during this position, this rotation continues so there is actually a push and pull force that is balanced keeping it in orbit.   There's a lot more going on in the atom than people realize.
Title: Why don't electrons and neutrons react?
Post by: Monabehi.desu@gmail. on 24/07/2016 00:15:59
Does an electron ever stick to a neutron and what happens then, and if not why?
Title: Re: Why don't an atom's electrons fall into the nucleus and stick to the protons?
Post by: evan_au on 25/07/2016 12:16:20
Quote from: billy kakes
there is actually a push and pull force that is balanced keeping it in orbit.
 
I think you are mentally using the "Solar System" analogy used to introduce the structure of the atom in High School. However, electrons aren't like little planets, they are much more like big dust clouds.

Quote
There's a lot more going on in the atom than people realize.
I think there's a lot more going on in the atom than you realize.

Title: Re: Why don't an atom's electrons fall into the nucleus and stick to the protons?
Post by: evan_au on 25/07/2016 12:17:47
Quote from: Monabehi desu
Does an electron ever stick to a neutron and what happens then, and if not why?
Not usually. Unlike the proton, the neutron has no electric charge that would attract the electron.

But electrons are sometimes used in particle accelerators to study the interior structure of protons and neutrons. High-energy electron collisions can produce a spray of subatomic particles from the quarks inside a proton or neutron.
eg see: https://en.wikipedia.org/wiki/SLAC_National_Accelerator_Laboratory#Accelerator
Title: Re: Why don't an atom's electrons fall into the nucleus and stick to the protons?
Post by: puppypower on 25/07/2016 12:32:39
If you start with a hydrogen atom, as electrons lower energy levels; get closer and closer to the nucleus, photons of energy are given off at each energy level drop. These photons can be used to excite another electron, to the same energy level; energy conservation.

As electrons reach the lowest energy state, but still outside the nucleus, if they were to drop energy levels even further, so they can enter the nucleus space, high energy photons will needed to be released. This typically is reabsorbed by the same electron, bringing it back to where it began. Sometimes, the energy is absorbed by a nucleus proton, allowing the electron to fall into the nucleus. That last burst of energy is then absorbed by a neutron, which then exits the nucleus.